<<

Monthly January 1 Current Affairs of 2019 DECEMBER

Welcome to 's Premier Institute for Banking & Staff Selection Commission. SEN'S SCHOOL OF COMPETITIVE EXAMINATIONS (SSCE) is a leading competitive coaching centre in Kolkata. It provides coaching for Staff Selection Commission ( CGL , CHSL , MTS , SSC STENOGRAPHER , SSC JE etc ) IBPS SEN SCHOOL OF (PO and Clerk) , SBI CLERK , RBI COMPETITIVE Assistant and other Banking Exams, EXAMINATIONS Railway (ASM , GOODS GUARD, TC, ECRC etc) PSC WB , POLICE, POST OFFICE Exams , Insurance and various competitive Examinations. ' SSCE' is the Best Competitive Exam Institute for Banking & SSC in Kolkata as well as Hooghly.

SEN’S SCHOOL OF COMPETIVE EXAMINATIONS SSCE Monthly Current Affairs (December 2018)

Daily Current Affairs 1st Dec 2018

1) Who has been selected as the recipient of 2018 Bharat Ratna Pandit Bhimsen Joshi Lifetime Achievement Award for Classical Music?

Ans:- Pandit Keshav Ginde

Explanation:- Renowned flautist Pandit Keshav Ginde has been selected as the recipient of 2018 Bharat Ratna Pandit Bhimsen Joshi Lifetime Achievement Award for Classical Music. In 1984, Pt. Ginde invented and played his creative innovation, the 'Keshav Venu' flute, a 42- inch long flute which can produce 3.5 octaves compared to the maximum 2.5 octaves by regular flutes. This unique flute has been preserved by the Sangeet Natak Akademi in its national archives. The flute has also been recognized by the Limca Book of Records as the only flute that can produce 3.5 octaves.

2) Bhasha Sangam scheme is introduced to school students with how many Indian languages?

Ans:- 22

Explanation:- The Government launched 'Bhasha Sangam', a unique initiative to introduce school students to 22 Indian languages. Bhasha Sangam launched under Ek Bharat Shreshtha Bharat. It is a programme for schools and educational institutions to offer multilingual exposure to students in Indian languages. The main aim is to improve linguistic tolerance and respect and promote national integration.

3) The theme for the World AIDS Day 2018 is;

Ans:- Know Your Status

Explanation:- World AIDS Day is designated on 1 December every year since 1988. The day provides an opportunity for people worldwide to unite in the fight against HIV, to show support for people living with HIV and to commemorate those who have died from an AIDS-related illness. More than 35 million people have died of HIV or AIDS, making it one of the most destructive pandemics in history. The theme for the World AIDS Day 2018 is

SSCE 8981426494/8296260082 Downloaded from - www.onlinessce.com Page 2

"Know Your Status". This will also be an occasion to celebrate the 30th anniversary of WAD.

4) World's first GST calculator was launched by which of the following company?

Ans:- Casio

Explanation:- Casio India has announced the launch of the world's first GST calculator with two new innovations MJ-120 GST and MJ-12GST dedicated to the Indian market. It will make manual invoicing easy and hassle-free. The key features of the Casio MJ-12 GST and MJ-120GST calculators are In-built GST tabs:All the five (0%, 5%, 12%, 18% and 28%) GST are in-built in MJ-120GST and MJ-12GST and The tax slabs are changeable as per the industry needs. Gross value (net value + Tax),net value and tax paid under different GST slabs stay stored in the GST+0, GST+1, GST+2, GST+3, GST+4 buttons and the overall value in the five slabs stays stored in the GST GT button. Tax- Mode application: a TAX- feature for all the five tax slabs to calculate the base value from MRP was introduced and will help in its application in calculating base value and net profit earned. Multi-industry use: GST+/tax- key can calculate values in multiple formats.

5) Name the Indian shooter who won the won the Women's, Youth Women's and Junior Women's 10m Air Pistol competitions at the National Shooting Championship at Thiruvananthapuram?

Ans:- Esha Singh

Explanation:- India's 13-year-old shooter Esha Singh won the Women's, Youth Women's and Junior Women's 10m Air Pistol competitions at the National Shooting Championship at Thiruvananthapuram. She beat Manu Bhaker and Heena Sidhu to win three gold medals in women's air pistol events at the 62nd National Shooting Championships. The Telangana shooter prodigy shot 241.0 in the women's final to leave behind Bhaker, who finished second with 238.9. Heena Sidhu finished 6th with a score of 154.9.

6) Who has been elected the first woman President of Georgia?

Ans:- Salome Zurabishvili

Explanation:- Salome Zurabishvili, a French-born Georgian politician, has elected the first woman president in the history of Georgia. She will be inaugurated as President of Georgia on 16 December 2018. The President of Georgia is head of state and a guarantor of national independence and unity of the country. He/she is elected on the basis of universal, equal and direct suffrage by secret ballot, for a period of five years. The President ensures the functioning of state bodies in accordance with the Constitution.

7) George HW Bush passed away at the age of 94. He was the former president of ______.

SSCE 8981426494/8296260082 Downloaded from - www.onlinessce.com Page 3

Ans:- US

Explanation:- The 41st President of the US, George HW Bush has passed away on at the age of 94.

8) Home Minister Rajnath Singh inaugurated 19th Hornbill Festival in ______.

Ans:- Nagaland

Explanation:- Home Minister Rajnath Singh inaugurated 19th Hornbill Festival in Nagaland on 55th Nagaland Statehood Day. In 1963, Nagaland attains its statehood and was declared as the 16th state of the Indian Union, by Dr. Sarvapelli Radhakrishnana, the then President of India. The fest aims to display the rich traditions and the vibrant culture of Nagaland through colourful performances, crafts, sports, food fairs, games, and ceremonies. US Ambassador to India Kenneth I Juster was invited as the guest of honour.

9) Who become the first Indian to be honoured with the Blue Cross by the International Shooting Sport Federation (ISSF)?

Ans:- Abhinav Bindra

Explanation:- India's Olympic gold medal-winning shooter Abhinav Bindra became the first Indian to be honoured with the Blue Cross by the International Shooting Sport Federation (ISSF) for his contribution to the sport of shooting. The Blue Cross is the highest laurel for shooters given by the ISSF. Bindra won an Olympic gold (2008), a World Championship gold (2006), 7 CWG medals and 3 Asian Games medals.

10) Which become the first country in the world to recognise 'orphanage trafficking' as a form of modern slavery?

Ans:- Australia

Explanation:- Australia has become the first country in the world to recognise 'orphanage trafficking' as a form of modern slavery. Children from smaller countries are recruited in orphanages in the West for the purpose of profit through foreign aid. According to the US State Department, "many orphanages use the children to raise funds by forcing them to perform shows for or interact and play with potential donors to encourage more donations". As per estimates, 80% of children living in the world's orphanages have at least one living parent.

11) What is the main focus of India Rivers Week 2018?

Ans:- Can India Rejuvenate Ganga?

Explanation:- The India Rivers Week (IRW) 2018, a unique meeting on rivers in India was held at the World Wide Fund for -India (WWF), New Delhi, from November 24 to 26,

SSCE 8981426494/8296260082 Downloaded from - www.onlinessce.com Page 4

2018. The focus of this year's three-day meet was 'Can India Rejuvenate Ganga?' The meeting this year took stock of the current progress on the rejuvenation of the Ganga, the appropriateness or effectiveness of the current road map and on developing a plan for the future that needs to be adopted if the Ganga is ever to be restored to its original state. The meeting this year took stock of the current progress on the rejuvenation of the Ganga, the appropriateness or effectiveness of the current road map and on developing a plan for the future that needs to be adopted if the Ganga is ever to be restored to its original state.

12) The 1st Global Aviation Summit will be held on January 15 at ______.

Ans:-

Explanation:- Supported by ICAO, IATA, ACI and CANSO, the 1st Global Aviation Summit will be held on January 15 and 16, 2019 at The Grand Hyatt in Mumbai. It aims to focus on the celebration of 'Flying for All' and to provide a platform to the aviation fraternity to highlight the challenges of the sector in the newly-developing growth spots.

13) US, Mexico and this country signs USMCA to Replace NAFTA.

Ans:- Canada

Explanation:- US President Donald Trump, Mexican President Enrique Pena Nieto, and Canadian PM Justin Trudeau signed the new US-Mexico-Canada Agreement (USMCA) which will replace the North American Free Trade Agreement (NAFTA). US President Donald Trump, Mexican President Enrique Pena Nieto, and Canadian PM Justin Trudeau signed the new US-Mexico-Canada Agreement (USMCA) which will replace the North American Free Trade Agreement (NAFTA).

14) Who has been sworn as the new Water Resources Minister of Kerala?

Ans:- K. Krishnankutty

Explanation:- K Krishnankutty sworn in as the new Water Resources Minister in the Pinarayi Vijayan cabinet of Kerala. He replaced Matthew T. Thomas who resigned from the position. He replaced Matthew T. Thomas who resigned from the position. He was previously elected to the Kerala Assembly in 1980, 1982 and 1991.

15) The first ever India Indonesia Business forum (IIBF) was held at ______.

Ans:- Port Blair

Explanation:- The first ever India Indonesia Business forum (IIBF) was held at Port Blair in Andaman and Nicobar Islands. The forum was held as part of policies of Delhi and Jakarta to extend their strategic partnership in the Indo-Pacific Region including Indian Ocean Region. It aimed to improve the commercial and people to people linkages between Andaman and Nicobar Islands of India with Aceh and North Sumatera Province of Indonesia. Members of

SSCE 8981426494/8296260082 Downloaded from - www.onlinessce.com Page 5

Andaman Chamber of Commerce and officials of Andaman and Nicobar Administration also participated.

16) Which University has been ranked A+ in the National Assessment and Accreditation Council (NAAC) review 2018?

Ans:- Delhi University

Explanation:- Delhi University has been ranked A+ in the National Assessment and Accreditation Council (NAAC) review held in the last week of October. The grading provided by the council is crucial for funds and grants allotted to a university by the University Grants Commission (UGC). The grading provided by the council is crucial for funds and grants allotted to a varsity by UGC. Last year, the University was ranked A++ in the NAAC review. DU got a Cumulative Grade Point Average (CGPA) of 3.28. The CGPA for A++ accreditation is between 3.51 to 4.

17) The 3rd ASEAN - India Business Summit (AIBS) held in ______.

Ans:- Kuala Lumpur

Explanation:- The 3rd ASEAN (Association of Southeast Asian Nations) India Business Summit was organised by the ASEAN India Business Council (AIBC) Malaysian Chapter in Kuala Lumpur, Malaysia. The Summit consisted of 3 events: ASEAN India Business Conference, ASEAN India Business Excellence Awards and Gala Dinner. It was organized under the guidance of Ministry of International Trade and Industries. The award ceremony was presided by the Malaysian Minister of International Trade and Industry YB Datuk Ignatius Darell Leiking.

18) The National Mission for Clean Ganga organised a two-day international workshop on Developing a Ganga Museum in which city?

Ans:- New Delhi

Explanation:- The National Mission for Clean Ganga organised a two-day international workshop on 'Developing a Ganga Museum Concept: Exchanging Experiences and Ideas between India and Europe' in Delhi. Experts from India and the European Union met and discussed various aspects of the proposed museum. Director General of NMGC, RR Mishra stated that the museum will be part of Namami Gange Programme.

Daily Current Affairs 2nd Dec,2018

1) Who has become the first woman to win the Greg Norman Medal as the best Australian golfer on the international stage?

Ans:- Minjee Lee

SSCE 8981426494/8296260082 Downloaded from - www.onlinessce.com Page 6

Explanation:- 22 Years old Australian Golfer, Minjee Lee became the first woman to win prestigious Greg Norman Medal as the best Australian golfer on the international stage after her brilliant performance in 2018. Minjee Lee who breaks into the Top 10 of the world ranking with the sixth rank joined the Jason Day and Marc Leishman in the list of winners of Australian golf's prestigious individual honour.

2) The Maharashtra assembly has passed the 'Maratha Reservation Bill' 2018. According to the bill, How many percent of reservation given under the socially and educationally backward category?

Ans:- 16%

Explanation:- On 29 November 2018, the Maharashtra assembly has passed the 'Maratha Reservation Bill' 2018. According to the bill, 16% reservation for Marathas will be given under the socially and educationally backward category. The Bill has now been sent to the state Legislative Council. However, there will be no reservation for Marathas in local body elections.

3) Name the software launched by Indian Railways that will enable the Union Railway Minister Piyush Goyal to keep track of punctuality of trains and freight and passenger earnings from anywhere in India?

Ans:- eDrishti

Explanation:- The Indian Railways launched a software named- 'eDrishti', that will enable the Union Railway Minister Shri. Piyush Goyalto keep track of punctuality of trains as well as freight and passenger earnings from anywhere in the country. The software has been developed by the Centre for Railway Information System (CRIS).

4) Name the person who took over as the Revenue Secretary in the Finance Ministry.

Ans:- Dr. Ajay Bhushan Pandey

Explanation:- UIDAI(Unique Identification Authority of India) CEO Dr. Ajay Bhushan Pandey took over as the Revenue Secretary in the Finance Ministry after Hasmukh Adhia retires. Dr. Ajay Bhushan Pandey, a 1984-batch IAS officer of Maharashtra cadre with over 34 years of experience in state and Central governments, was welcomed to his new office by the outgoing Finance and Revenue Secretary Hasmukh Adhia.

5) Border Security Force celebrated its 54th Raising Day on December 1, 2018. It was celebrated with the motto of;

Ans:- Duty Unto Death

Explanation:- The Border Security Force, BSF along with Pakistan and celebrated its 54th Raising Day. The main ceremony was celebrated at Chhawla Camp in

SSCE 8981426494/8296260082 Downloaded from - www.onlinessce.com Page 7

New Delhi. BSF has been India's first wall of defence since 1965. The motto of BSF is: Duty Unto Death.

6) Who was appointed the next Chief Executive Officer (CEO) of the World Gold Council?

Ans:- David Tait

Explanation:- The market development organization for the gold industry, The World Gold Council announced the appointment of David Tait who has a very successful career in the financial services industry as the incoming Chief Executive Officer (CEO). David began his professional career at Goldman Sachs. Before his appointment as the CEO of World Gold Council, he worked as the Global Head of Fixed Income Macro Products at Credit Suisse.

7) Which city is the venue of the South Asia Regional Youth Peace Conference?

Ans:- New Delhi

Explanation:- The South Asia Regional Youth Peace Conference has been organised by Gandhi Smriti and Darshan Samiti (GSDS) in partnership with UNESCO- MGIEP and STEP (Standing Together to Enable Peace) from November 28-30, 2018. The 3-day conference is being organized to mark the beginning of celebration of 150th birth anniversary of Mahatma Gandhi. The conference was inaugurated by ShriKrishna G Kulkarni, great grandson of Mahatma Gandhi, at Gandhi Darshan in New Delhi. About 100 youth leaders from South Asian countries and different parts of India have come together to discuss different dimensions of 'peace'. The conference focused on building a network for young leaders from South Asian countries working on various social issues, to identify avenues for partnership and collaboration, support and show solidarity towards each other's initiatives, nurture relationships and promote global citizenship to work towards achieving the UN Sustainable Development Goals (SDGs).

8) 2019 Global Aviation Summit will be held in ______, India.

Ans:- Mumbai

Explanation:- The two-day 'Global Aviation Summit' will be held in Mumbai in January 2019. The summit will be organised by the Ministry of Civil Aviation and the Airport Authority of India (AAI) in collaboration with the Federation of Indian Chambers of Commerce & Industry (FICCI). The theme of the summit is 'Flying for all - especially for the next 6 billion'.

9) Name of the Chinese firm which unveils plans to provide free worldwide WiFi with 272 satellites.

Ans:- LinkSure Network

SSCE 8981426494/8296260082 Downloaded from - www.onlinessce.com Page 8

Explanation:- A Chinese Internet Technology firm, LinkSure Network which markets itself as a Global Innovative Mobile Internet Company specializing in free internet access, content and location based services unveiled a plan to provide free worldwide WiFi service with a of 272 satellites by 2026. The Company founded in 2013 in Shanghai will launch its first satellite in 2019 from Jiuquan Satellite Launch Centre in Northwest China's Gansu province and by 2020 it will launch 10 satellites. The Company will invest 3 Billion Yuan (USD 431.4 million).

10) Which bank increased interest rates for fixed deposits less than Rs. 1 crore?

Ans:- SBI Bank

Explanation:- State Bank of India (SBI) revised its fixed deposit (FD) interest rates on select maturities by 0.05-0.10 per cent or 5-10 basis points. The hiked interest rates are on deposits below Rs. 1 crore. These changes were done ahead of the fifth bi-monthly monetary policy review outcome on December 5.

11) Name the country that amended the Offensive Weapons Bill 2018 to ensure the right to possession of kirpans by Sikhs.

Ans:- United Kingdom

Explanation:- The United Kingdom government amended the Offensive Weapons Bill 2018 to ensure right to possession of kirpans by Sikhs. The move would maintain status quo in continuing to legally safeguard the sale, possession and use of large kirpans and to not criminalise the Sikh community for the sale or possession of one. The move was led by the first female Sikh MP in the House of Commons, MP Preet Kaur Gill. The bill is aimed at strengthening existing legislative measures on offensive weapons, focusing on corrosive substances, knives and certain types of firearm. It would also ban the sale of corrosive substances to anyone under the age of 18.

12) Who of the following bagged the 2018 World Chess Championship?

Ans:- Magnus Carlsen

Explanation:- World number one Norwegian Magnus Carlsen has retained his World Chess Championship title, beating US opponent Fabiano Caruana in a tie-breaker event. Carlsen's peak rating of 2882, achieved in 2014, is the highest in history. Carlsen became World Champion in 2013 by defeating Viswanathan Anand.

13) European Union has set a goal to become the first major economy to be 'Climate Neural' by ______.

Ans:- 2050

SSCE 8981426494/8296260082 Downloaded from - www.onlinessce.com Page 9

Explanation:- The European Union has set a goal to become the first major economy to be 'Climate Neural' by 2050 after that the emissions of greenhouse gases needs to be offset by planting trees or burying the gases underground. As per European Union, this target can be achieved by using Solar and Wind Energy for electricity generation. Climate Neutrality means the emissions are to be balanced by methods of removing warming gases from the atmosphere by planting new forests or through carbon capturing technologies.

14) British Council sign a MoU with this state government to strengthen educational and cultural cooperation and support the knowledge ambitions.

Ans:- Telangana

Explanation:- The Telangana government and British Council have signed a Memorandum of Understanding (MoU) to strengthen educational and cultural cooperation and support the knowledge ambitions and economic growth of Telangana. The signatories, Alan Gemmell, Director, British Council India, and Mr. Jayesh Ranjan, Principal Secretary, Information Technology, signed the MoU in the presence of Chief Secretary S.K. Joshi. The British Council has aimed at inspiring youngsters from across Telangana and India to build a relationship with the UK for the next 70 years. Mr. Joshi hoped that the MoU will expand the scope of the Council's engagements in developing the skill of young people.

15) Who was awarded the special ASEAN - India Achievement award for the businessman of year, at the 3rd ASEAN India Business Summit?

Ans:- Pradip Batra

Explanation:- The 3rd ASEAN (Association of Southeast Asian Nations) India Business Summit was organised by the ASEAN India Business Council (AIBC) Malaysian Chapter in Kuala Lumpur, Malaysia. Spice Garden group of restaurants owner Pardeep Batra was awarded the special ASEAN - India Achievement award for the businessman of year.

Daily Current Affairs 3rd Dec,2018

1)The 4th edition of Ex Cope India-18, a Bilateral Joint exercise between IAF and USAF was held in Which Indian state?

Ans:- West

Explanation:-Ex Cope India-18 is the fourth edition in the series of Bilateral Joint exercise held between IAF and USAF, which is conducted in India. This is the first time, the exercise is being planned at two Air Force bases, Kalaikunda and Panagarh (West Bengal). USAF is participating with 12 X F15 C/D and 03 X C-130. IAF is participating with the Su-30 MKI, Jaguar, Mirage 2000 , C-130J & AWACS aircraft. The aim of exercise is to provide operational exposure and undertake mutual exchange of best practices towards enhancing operational capability.

SSCE 8981426494/8296260082 Downloaded from - www.onlinessce.com Page 10

2)Shri Ram Nath Kovind, inaugurated the 13th edition of CII Agro Tech India-2018 in ______.

Ans:-

Explanation:-The President of India, Shri Ram Nath Kovind, inaugurated the 13th edition of CII Agro Tech India-2018 in Chandigarh. Speaking on the occasion, the President stated that Indian agriculture needs a renewal of its marriage with contemporary technology; protection against climate change, price fluctuations and demand shocks; and sustained investment by and partnership with business. Together these will enhance agricultural value and competitiveness as well as lead to better incomes.

3)The first Indian who was elected as ISSF Vice President?

Ans:-Raninder Singh

Explanation:-The elections were held during the General Assembly meeting of ISSF in Munich in which Raninder secured 161 votes to make it to the coveted position. The 51- year-old Raninder is currently the president of the National Rifle Association of India (NRAI). The NRAI president was also awarded the ISSF diploma of honor gold medal at the General Assembly by the outgoing longest-serving president of the ISSF, Olegario Vazquez Rana, who was at the helm for 38 years. Vladimir Lisin, of Russia, was elected as the seventh president of ISSF and Alexander Rather was elected as the new secretary general of ISSF in the same General Assembly meeting.

4)What is the theme of 2018 International Day of Persons with Disabilities?

Ans:-Empowering persons with disabilities and ensuring inclusiveness and equality

Explanation:-Since 1992, the United Nations International Day of Persons with Disabilities (IDPD) has been annually observed on 3 December around the world. The theme for this year's IDPD is "Empowering persons with disabilities and ensuring inclusiveness and equality". This theme focuses on the empowering persons with disabilities for the inclusive, equitable and sustainable development envisaged in the 2030 Agenda for Sustainable Development.

5)A.S. Rajeev was appointed as the MD and CEO of this bank rcently.

Ans:-Bank of Maharashtra

Explanation:-Bank of Maharashtra appoints A.S. Rajeev as MD and CEO. The 54-year-old previously served as the Executive Director of Indian Bank and had also been the CFO of Vijaya Bank. He will hold the position for a period of three years or till further orders, whichever is earlier.

6)Where was the Climate Change Conference 2018 held?

SSCE 8981426494/8296260082 Downloaded from - www.onlinessce.com Page 11

Ans:-Poland

Explanation:-The UN climate summits, i.e. so called COP (Conference of the Parties) are global conferences, in the course of which action for climate policy is negotiated. The Climate Change Conference 2018 was held in Katowice, . Previously, Poland hosted them twice - in 2008, in Poznan and in 2013, in Warsaw. For the first time, the climate summit took place in Katowice. This year's summit included: 24th Conference of the Parties to the United Nations Framework Convention on Climate Change (COP24), 14th Meeting of the Parties to the Kyoto Protocol (CMP 14) and the 1st Conference of Signatories to the Paris Agreement (CMA 1).

7)Where the 2018 Global Agriculture and Food Summit was held?

Ans:-Jharkhand

Explanation:-Union Agriculture Minister Radha Mohan Singh inaugurated Global Agriculture and Food Summit 2018 in Ranchi, Jharkhand. Seven countries, including China, Israel and Malaysia are participating in the two-day Summit. The aim of the Summit is to promote Jharkhand as a key player to attract investment in the agriculture sector. Several dignitaries attended the inaugural ceremony including Jharkhand Chief Minister Raghubar Das and NITI Aayog CEO Amitabh Kant. The summit will focus on organic farming in horticulture, dairy, poultry and fisheries, and food processing.

8)According to the report by Bain & Company, which of the following country is the fastest growing e-commerce market?

Ans:-India

Explanation:-According to the report, India's online retail market had a Compound Annual Growth Rate (CAGR) of 53% from 2013 to 2017. This rapid growth is due to heavily discount-driven e-commerce marketplaces, improvement in delivery infrastructure and increased smartphone penetration and data usage. India's total e-commerce retail sales in 2017 were nearly $20 billion. As per the studies, another $50 billion of online e-commerce could be introduced by adding new users and bringing back the currently inactive old users. India was in 8th rank in terms of online retail penetration among 11 countries.

9)Which Ministry announced that 15 helipads will be implemented across the country to improve tourism prospects?

Ans:-Ministry of Civil Aviation

Explanation:-Ministry of Civil Aviation plans to introduce 15 helipads across the country to improve tourism prospects. Union Minister of State for Civil Aviation Jayant Sinha said that the helipads will be established in hilly areas to explore the tourist potential in the north and north-eastern parts of the country. While addressing at STEP conclave organized by Prabudha Bharat in Visvesvaraya Technological University (VTU), Belagavi. He also said

SSCE 8981426494/8296260082 Downloaded from - www.onlinessce.com Page 12

that the government was committed to improving air connectivity at tier-II cities, including Belagavi. The government was also contemplating opening three airports in the State for international destinations.

10)On which date, the World Computer Literacy Day is observed?

Ans:-December 2

Eplanation:-World Computer Literacy Day will be celebrated on December 2 every year.Now the computer has became part of our human life and this day encourage the development of technological skills, particularly among people. It aims to "create awareness and drive digital literacy in underserved communities worldwide". World Computer Literacy Day was founded by Indian computer company NIIT to mark its 20th anniversary in 2001.

11)The 7th India and China joint military exercises - 'Hand in Hand' will commence from ______.

Ans:-December 10

Explanation:-The militaries of China and India would hold a joint 14-day drill from December 10 in southwest China's Chengdu city to improve their capabilities in fighting terrorism and promote mutual understanding. The 7th India and China joint military exercises - 'Hand in Hand' - will focuss on counter-terrorism operations. The exercises will be held after a gap of one year as both the sides were locked in a 73-day standoff in Doklam in the Sikkim sector of the border in 2017.

12) Which of the following state signed pact with Estonian academy on e-governance?

Ans:-Nagaland

Explanation:-The Nagaland government has recently signed pact with Estonian academy to enlighten the state people on the e-governance models prevailing in the world. The main objective of the 5-year agreement with the e-Governance Academy of Estonia is to work together to develop and expand scientific collaboration on setting up e-Governance Academy in Nagaland. The agreement was signed during the current edition of the e-Naga Summit, held with a vision to embark on a process to instill the spirit of innovation and use of technology for impacting positive change in the society.

13)Which of the following rocket is to be launched by NASA?

Ans:-VISIONS-2

Explanation:-NASA is set to launch VISIONS-2 rocket to get a closer look at the how the 's atmosphere is leaking into space. The VISIONS-2 mission, Visualizing Ion Outflow via Neutral Atom Sensing-2, is scheduled to be launched on December 4. Understanding atmospheric escape on earth has applications all over the Universe, from predicting which

SSCE 8981426494/8296260082 Downloaded from - www.onlinessce.com Page 13

planets might be habitable, to piecing together how Mars became a desolate landscape. A sounding rocket makes brief, targeted flights into space before falling back to Earth just a few minutes later. Sounding rockets are unique among scientific spacecraft for their superior dexterity. They can be carted to remote locations, where they are aimed and shot into short- lived events, like the sudden formation of the aurora borealis, at a moment's notice. The aurora borealis is of keen interest to the VISIONS-2 team, but not just for its otherworldly glow. The aurora play are fundamental drivers in the process of atmospheric escape, whereby planets, including Earth, gradually leak their atmosphere into space.

14)Which of the following has been approved under UNESCO's world's most treasured cultural practices?

Ans:-Korean wrestling

Explanation:-UNESCO accepted a joint bid for Korean wrestling to be recognized as one of the world's most treasured cultural practices. This bids a new step in their reconciliation efforts of North and South Korea. The two Koreas had originally filed separate applications for their traditional form of wrestling to be recognized on the UN agency's Intangible Cultural Heritage list. The announcement is the latest in a string of symbolic gestures between Seoul and its nuclear-armed neighbor in recent months. In North Korea, it is known as Ssirum and in South Korea, it is known as Sssireum. The wrestling has been practiced at village festivals for centuries.

15)Which word is declared as the Cambridge Dictionary's Word of the Year 2018?

Ans:-Nomophobia

Explanation:-UK's Cambridge Dictionary has confirmed "nomophobia" the inaugural People's Word of the Year (2018).Cambridge Dictionary has named "nomophobia" which means the "fear or worry at the idea of being without your mobile phone or unable to use it" became word of the year 2018. It defeats the new words such as "gender pay gap", "ecocide" (destruction of the natural environment of an area, or very great damage to it), and "no- platforming" (the practice of refusing someone an opportunity to make their ideas or beliefs known publicly, because you think these beliefs are dangerous or unacceptable).

16)Who will be the Chief Guest of 2019 Republic Day?

Ans:-Cyril Ramaphosa

Explanation:-South African President Cyril Ramaphosa will be the Chief Guest at the 2019 Republic Day celebrations in New Delhi. The anouncement was made by the Prime Minister Narendra Modi. India is celebrating the 150th birth anniversary of Mahatma Gandhi. Mr Modi also stated that Mr Ramaphosa's visit will further cement business and people to people ties between India and South Africa.

17)Who won the Pune International Marathon 2018?

SSCE 8981426494/8296260082 Downloaded from - www.onlinessce.com Page 14

Ans:-Atlawim Debebe

Explanation:-Atlawim Debebe of Ethiopia won the Full Marathon with the timing of 2 hours 17 minutes 17 seconds of Pune International Marathon 2018. Teshome Getachew claimed second and Bekele Asefa third position. Both of them are also from Ethiopia. In women's Full Marathon Pascalia Chepkogei from Kenya stood first with the timing of 2 hour 50 minutes 27 seconds. Belew Mekonnen and Fekede Tilahun of Ethiopia claimed second and third position.

18) Which country to Chair Kimberley Process From January 2019?

Ans:-India

Explanation:-The Geological Survey of India has been made the sole custodian and curator of all meteorites within the Indian boundary. India will chair the Kimberley Process from January 2019. National Projects Construction Corporation (NPCC) Limited has been conferred with the status of 'Miniratna' by the Government of India.

19)Which bank gets RBI approval for subsidiary?

Ans:-State Bank of Mauritius

Explanation:-Mauritius based SBM group granted a license by Reserve Bank of India to operate SBM Bank (India) Limited through Wholly Owned Subsidiary (WOS) mode under the Banking Regulation Act, 1949 with effect from December 1, 2018. Now at present SBM operate its branches in Mumbai, Chennai, Hyderabad, and Ramachandrapuram since 1994. It plans to open six new branches in Delhi, Bangalore, Kolkata, Pune, Ahmedabad, and Jaipur by next year.

Daily Current Affairs 4thDec,2018

1)Who launched a PAiSA portal for processing interest subvention on bank loans to beneficiaries under Deendayal Antyodaya Yojana - National Urban Livelihoods Mission (DAY-NULM)?

Ans:- Durga Shanker Mishra

Explanation:-"PAiSA - Portal for Affordable Credit and Interest Subvention Access", a centralized electronic platform for processing interest subvention on bank loans to beneficiaries under Deendayal Antyodaya Yojana - National Urban Livelihoods Mission (DAY-NULM) was launched by Sh. Durga Shankar Mishra, Secretary, Ministry of Housing and Urban Affairs. The web platform has been designed and developed by Allahabad Bank which is the Nodal bank. All 35 states / UTs & all scheduled commercial banks, RRBs, and Cooperative Banks are expected to be on board the PAiSA portal soon.

SSCE 8981426494/8296260082 Downloaded from - www.onlinessce.com Page 15

2)The Centre and Asian Development Bank (ADB) have signed an 85 million dollar loan to improve skill development eco-system in which of the following state?

Ans:-Odisha

Explanation:-The Centre and Asian Development Bank (ADB) have signed an 85 million dollar loan to improve skill development eco-system in Odisha. An advanced skill training center, the World Skill Center (WSC) will also be established in Bhubaneswar with the help of a loan. The Odisha Skill Development Project agreement was signed by Additional Secretary in the Finance Ministry Sameer Kumar Khare and Country Director of ADB's India Resident Mission Kenichi Yokoyama in New Delhi.

3)Indian Navy Day was observed on;

Ans:-4th December

Explanation:-Navy Day is being celebrated in India on 4th December. The day is celebrated in honour of the maritime force's role during the war with Pakistan in 1971 when Indian warships attacked Karachi port and successfully hurt Pakistani operations in the western coast. The Day is also celebrated to highlight the role the Navy plays in securing the country's marine borders during peacetime and carrying out humanitarian missions. As per the recent Data, since 2008, 70 Indian naval warships have been deployed, which safely escorted over 3440 ships with over 25,000 mariners on board.

4)Vinesh Phogat won a ______medal in the 57 kg of Tata Motors Senior Women's National Championship.

Ans:-Gold

Explanation:-Indian wrestlers Vinesh Phogat and Sakshi Malik successfully emerged champions in the 57 kg and 62 kg title respectively in the Tata Motors Senior Women's National Championship. Vinesh Phogat bagged her sixth National title. She had earlier won five Nationals in a row from 2012 to 2016. Sakshi put up a confident show to win the title in the 62 kg without losing a single point. The event was hosted by Uttar Pradesh.

5)Pawan Singh has become the first Indian to be elected as one of the member of Judges Committee of ______.

Ans:-ISSF

Explanation:-Pawan Singh has become the first Indian to be elected one of seven member's of the Judges Committee of the International Shooting Sports Federation (ISSF). Mr. Singh also holds the post of the Joint Secretary General of the National Rifle Association of India (NRAI). Mr. Singh got elected onto the seven members' Judges Committee for which 22 candidates from all over the world were in the fray. Members of the administrative council of

SSCE 8981426494/8296260082 Downloaded from - www.onlinessce.com Page 16

ISSF voted to select the committee during the elections held at Munich. The ISSF elections take place every four years.

6)Where did Bureau of Police Research and Development organise the 2nd National Conference of Heads of Investigating Agencies?

Ans:-New Delhi

Explanation:-Bureau of Police Research and Development will organize the 2nd National Conference of Heads of Investigating Agencies in New Delhi.The two-day Conference will deliberate on various legal aspects, procedures and use of technology in investigation to improve its quality and secure effective prosecution.The theme of the conference is "Capacity Building of Police Investigators in the Era of New Age Crimes". More than 200 delegates from all States and Union territories will participate in the conference.

7)T.S. Vijayan was appointed as the additional director of which of the following bank?

Ans:- Yes Bank

Explanation:-Private sector lender Yes Bank Ltd appointed former IRDAI chief T.S. Vijayan as an additional director (Independent) for five years with immediate effect. He is already a key member of the search and selection committee (S&SC) set up to identify a suitable successor for CEO Rana Kapoor. Vijayan's appointment would be subject to the approval of Yes Bank shareholders general meeting of the bank. Vijayan has also served as the Chairman of Life Insurance Corporation of India (LIC).

8)Who has been appointed as new Finance Secretary?

Ans:-Ajay Narayan Jha

Explanation:-Ajay Narayan Jha, the Secretary of Department of Expenditure, has been appointed as new Finance Secretary to succeed Hasmukh Adhia who retired on November 30. Fifty-nine-year old Jha is a 1982-batch IAS officer of Manipur Tripura cadre. An alumnus of St Stephens College, he passed with first class in Graduation and Post Graduation in History.

9)Jon Ridgeon was sworn in as the CEO of ______.

Ans:-IAAF

Explanation:-The International Association of Athletics Federations (IAAF) announced the appointment of Jon Ridgeon as its new CEO following approval by the IAAF Council on the first day of its 215th Council Meeting in Monaco. Ridgeon will take up the new role in March 2019.

SSCE 8981426494/8296260082 Downloaded from - www.onlinessce.com Page 17

10)Who assumes charge as Technical Member in the Appellate Tribunal for Electricity?

Ans:-Ravindra Kumar Verma

Explanation:-Ravindra Kumar Verma took charge as Technical Member in the Appellate Tribunal for Electricity (APTEL), Ministry of Power, for a period of 3 years. Justice Manjula Chellur, Chairperson of APTEL, administered the Oath of Office and Secrecy to Ravindra Kumar Verma. Ravindra Kumar Verma occupied various posts in the Central Electricity Authority (CEA) for over

Ans:-37 years.

11)Which country plans to leave OPEC from January 2019?

Ans:-Qatar

Explanation:-Qatar has announced its decision to leave the Organization of the Petroleum Exporting Countries (OPEC) from January 2019 and focus more on the production of natural gas. The announcement was made by Qatar's new energy minister, Saad al-Kaabi. Qatar is the world's largest exporter of LPG. Qatar also wants to raise its oil production from 4.8 million barrels of oil equivalent a day to 6.5 million barrels.

12)Who has been appointed as the Secretary-General of NATHEALTH, recently?

Ans:-Siddhartha Bhattacharya

Explanation:-NATHEALTH has announced the appointment of Siddhartha Bhattacharya as Secretary-General. In his new role, Siddhartha will lead the organization and work with key partners in realizing NATHEALTH's vision/mission to spearhead the goal of high quality, affordable health for all.

13)This company plans to merge with GlaxoSmithKline Consumer Healthcare.

Ans:-Hindustan Unilever Limited

Explanation:-FMCG major Hindustan Unilever Limited (HUL) announced that its board has approved the merger with GlaxoSmithKline Consumer Healthcare (GSKCH India) through an-all equity deal, valuing the total business of the latter at Rs31,700 crore. The transaction is an all-equity merger with 4.39 shares of HUL being allotted for every share in GSKCH India, which sells consumer healthcare products, including popular drink brand Horlicks. Post the acquisition, the turnover of the company's Foods and Refreshment (F&R) business will exceed Rs10,000 crore.

14)Which state is the second state in the country to launch the emergency mobile number?

SSCE 8981426494/8296260082 Downloaded from - www.onlinessce.com Page 18

Ans:-Nagaland

Explanation:-Nagaland became the first state in the Northeast and second in the country, after Himachal Pradesh, to launch a pan-India single-number (112) emergency mobile application which will have a special women safety feature for immediate assistance from police and volunteers. Union Home Minister Rajnath Singh stated that a 'SHOUT' feature has been made available exclusively for women in the '112 India' mobile app which is connected to the Emergency Response Support System (ERSS). The launch coincided with the state's formation day and the inaugural day of the 2018 edition of the Hornbill Festival.

15)Which country has topped the Global Passport Power Rank 2018?

Ans:-UAE

Explanation:-As per the ratings released by Passport Index, the United Arab Emirates has topped the Global Passport Power Rank 2018, emerging as the country with the most powerful passport in world. UAE passport holders can now fly to 113 countries without a visa, and 54 others with a visa-on-arrival. Singapore and Germany ranks second with 166 countries. India ranked 66th in Global Passport Power Rank 2018.

16)The first Indian who won Bronze medal in the international ultra running event is;

Ans:-Ullas Narayan

Explanation:-Ullas Narayan became the first ever Indian individual medal winner in an international ultra running event by clinching a bronze in the 2018 International Association of Ultra Running (IAU) 24 Hour Asia & Oceania Championships in Taipei. Narayan bagged the third position, completing 250 km, behind two Japanese runners Yoshihiko Ishikawa (253 km) and Nobuyuki Takahashi (252 km). Japan won the team event gold with a distance of 756 km and Australia (684 km) got the silver.

17)Who won Ballon d'Or 2018 in Paris?

Ans:-Luka Modric

Explanation:-Real Madrid and Croatia midfielder Luka Modric won Ballon d'Or 2018 in Paris. In the process, he broke the decade-long duopoly over the award held by Cristiano Ronaldo and Lionel Messi. The last player to win the award outside of Ronaldo and Messi was Brazilian great Kaka in 2007.

18) Who bagged the Hero to Animals by PETA?

Ans:-Imran Hussain

Explanation:- Delhi Minister of Food Supplies, Environment and Forest and Elections Imran Hussain was awarded by the People for the Ethical Treatment of Animals (PETA)-

SSCE 8981426494/8296260082 Downloaded from - www.onlinessce.com Page 19

India for banning the Chinese Manjha, or the glass-coated kite strings, across Delhi. The award, named 'Hero to Animals' was also given to three other individuals and companies that have pushed animal rights forward.

19)TIPS an Instant Payment System launched by;

Ans:-European Central Bank

Explanation:-The European Central Bank launched a new system aimed at letting banks settle payments instantly across Europe, helping them to compete with PayPal and other global tech giants. Developed in just over a year, the ECB's TARGET Instant Payment Settlement (TIPS) system will let people and companies in Europe transfer euros to each other within seconds and regardless of the opening hours of their local bank. TIPS is open only to providers that have an account at a central bank connected to the euro zone's TARGET 2 network, meaning it is effectively restricted to European Union banks.

20)Which is the first manned mission to the International Space Station (ISS)?

Ans:-Soyuz mission

Explanation:-After the October launch failure, a Soyuz rocket carrying Russian, American and Canadian astronauts took off from Kazakhstan and reached orbit on 3rd December in the first manned mission. Russian cosmonaut Oleg Kononenko, Anne McClain of NASA and David Saint-Jacques of the Canadian Space Agency were launched off for a six-and-a-half month mission on the International Space Station (ISS). After the rocket lifted off from the Baikonur Cosmodrome, Russian space agency Roscomos announced that the capsule was successfully launched into orbit.

Daily Current Affairs 5thDec,2018

1)Which ministry released a book title "A Handbook for Students on Cyber Safety to aware teenagers about online Crimes and Threats"?

Ans:-Ministry of Home Affairs

Explanation:-The Ministry of Home Affairs released a booklet in the English language titled "A Handbook for Students on Cyber Safety" to aware the Students and Adolescents about the cyber safety. The book explains the various kinds of Cyber-crimes like identity theft, job fraud, email spoofing and how children can overcome them. The booklet also address the problems associated with increased use of smartphones, Social Media, Online gaming.

2)Name the Indian who won gold in the Senior Women's Sabre event of the Commonwealth Senior Fencing Championships held in Canberra, Australia.

Ans:-Bhavani Devi

SSCE 8981426494/8296260082 Downloaded from - www.onlinessce.com Page 20

Explanation:-Bhavani Devi won gold medal in the Senior Women's Sabre event. She has become the first Indian to win a gold medal in the Commonwealth Championship. India won a total of 3 medals: 2 gold and 1 bronze, in the championship. The medal tally was topped by England with 6 gold, 6 silver and 4 bronze medals. Karan Singh won bronze medal in the Senior Men's Sabre event. Karan Singh teamed with Sagar Sahi, Manpreet Singh and Vishal Thapar and won gold medal in the Senior Men's Sabre team event.

3)The "rakhi with khaki" initiative of which police station holds the Guinness World Records?

Ans:-Bilaspur Police

Explanation:-The Bilaspur Police in Chhattisgarh now holds the Guinness World Records for its 'Rakhi with Khakhi' as part of which girls and women tied rakhis to 50,033 police officials rakhis over a span of ten hours and then uploaded selfies with hashtags #rakhiwithkhaki and #HappyRakshaBandhan. The initiative was the brainchild of Bilaspur Superintendent of Police Sheikh Arif Husen & was held on 25th August 2018, a day before Raksha Bandhan. The Guinness World Records certificate was awarded to Bilaspur police at a function held at the Lakhiram Memorial Auditorium on 1st December 2018. The aim of the initiative is to reach out to women and girls and to assure them of security.

4)SHINYUU Maitri-18 is a bilateral air exercise between India and this country.

Ans:-Japan

Explanation:- A bilateral air exercise 'SHINYUU Maitri-18', between the Japanese Air Self Defence Force (JASDF) and Indian Air Force (IAF) began on December 3, 2018 at Air Force Station Agra. The exercise will end on December 7, 2018. The theme of the exercise is joint Mobility/Humanitarian Assistance & Disaster Relief (HADR) on Transport aircraft. IAF is participating with An-32 and C-17 aircraft with aircrew & observers.

5)Who was named as the brand ambassador of SBI's YONO?

Ans:-Swapna Barman

Explanation:-State Bank of India roped in Asian Games gold medalist Swapna Barman as brand ambassador for its digital app YONO(You only need one). YONO is an integrated digital banking platform offered by State Bank of India (SBI) to enable users to access a variety of financial and other services such as taxi bookings, online shopping, or medical bill payments.

6)World Soil Day is celebrated annually on the 5th of December. The theme of this year is;

Ans:-StopSoilPollution

SSCE 8981426494/8296260082 Downloaded from - www.onlinessce.com Page 21

Explanation:-World Soil Day is celebrated annually on the 5th of December at the Food and Agriculture Organization of the United Nations (FAO) headquarters in Rome, the regional offices and through national and local events. #StopSoilPollution is the objective of this World Soil Day on 5 December and throughout the year.

7)Which government launched the Common Mobility Card recently to travel aboard both metro trains and State-run buses?

Ans:-Delhi

Explanation:-Delhi Transport Minister Kailash Gahlot launched the Common Mobility Card which can be used by commuters to travel aboard both Delhi metro trains and State-run buses. The ONE card, which is essentially the rebranded Delhi Metro smart card with the tagline 'One Delhi. One Ride' with the logos of all three transport operators in the city, the Delhi Metro, the Delhi Transport Corporation and the Delhi government. National Common Mobility Card (NCMC). The Urban Development Ministry brought in the National Payments Corporation of India (NPCI) with the task of management, clearing, and settlement of payments, simulating cards and terminals and maintenance of the network.

8)Who flagged off the first IndiGo Airlines flight to Singapore from the Vijayawada?

Ans:-Venkaiah Naidu

Explanation:-Vice-President M.Venkaiah Naidu flagged off the first IndiGo Airlines flight to Singapore from the Vijayawada international airport, Vijayawada on 4th December. Mr.Venkaiah Naidu also laid the foundation stone for the integrated passenger terminal being developed at an estimated cost of Rs.611 crore at the airport. He appreciated the efforts of the State and the Central governments in facilitating Andhra Pradesh with air connectivity under UDAN (Ude Desh ka Aam Nagrik) scheme. He mentioned the scheme would enable greater regional connectivity. Vijayawada and Tirupati airports were already declared as international airports.

9)Which is the first Artificial Intelligence (AI) powered robot to fly in Space, which made an interaction with the spacefaring crew of International Space Station?

Ans:-CIMON

Explanation:-CIMON (Crew Interactive Mobile Companion), the first Artificial Intelligence (AI) powered robot to fly in Space, was launched to the International Space Station (ISS) in June 2018. German astronaut Alexander Gerst talked with the artificially intelligent crew- assistant CIMON during a 90-minute experiment on November 15, 2018, aboard the International Space Station (ISS).

10)Recently, Delhi Daredevils was renamed as ______.

Ans:- Delhi Capitals

SSCE 8981426494/8296260082 Downloaded from - www.onlinessce.com Page 22

Explanation:-Delhi Daredevils renamed as Delhi Capitals ahead of 2019 IPL season. The change in name comes close on the heels of JSW Sports picking up a 50 percent stake in the franchise. Ahead of the 2019 IPL auctions, the Delhi Daredevils released former captain Gautam Gambhir and a host of international which include Jason Roy, Glenn Maxwell and Liam Plunkett, along with Indian paceman Mohammed Shami.

11)The Indian Olympic Association (IOA) has submitted a formal expression of interest to host ______Olympics.

Ans:-2032

Explanation:-The Indian Olympic Association (IOA) has submitted a formal expression of interest to bid for 2032 Olympics. Mumbai and Delhi are the top two options under consideration by Indian Olympic Association (IOA), which has approached the International Olympic Committee. India is holding on to its dream of hosting the Olympic games for the first time. The bidding process is expected to start in 2022 and a host city will be decided by 2025.

12)How many small satellites are launched by SpaceX recently?

Ans:-64

Explanation:-SpaceX successfully launched 64 small satellites, including India's ExseedSAT-1 in the company's largest-ever "rideshare" mission on a twice-reused Falcon 9 rocket that lifted off from a California launchpad. This is a new US record. The Falcon 9 rocket carried 15 micro-satellites and 49 CubeSats belonging to 34 different clients including public, private and university sources from 17 different countries across the world including South Korea, France and Kazakhstan.

13)Who is considered as the Father of the Indian Navy?

Ans:-Chhatrapati Shivaji Bhosle

Explanation:-The Maratha Emperor, "Chhatrapati Shivaji Bhosle" is considered as 'Father of the Indian Navy'. India celebrated its 47th Navy Day. The day is celebrated in honour of the maritime force's role during the war with Pakistan in 1971 when Indian warships attacked Karachi port. The Navy showcased an impressive show of skill amid a sea of colours at the Gateway of India in Mumbai.

14)With which ministry Madame Tussauds Wax Museum signs an MoU recently?

Ans:-Ministry Of Railways

Explanation:-Ministry of Railways' National Rail Museum (NRM) and Madame Tussauds Wax Museum join hands together to provide an amazing combo offer to Delhi NCR tourists. As a part of the collaboration, a special discount of 35% on ticket prices will be offered to

SSCE 8981426494/8296260082 Downloaded from - www.onlinessce.com Page 23

NRM visitors when they visit the Madame Tussauds Museum. Similarly, Madame Tussauds Wax Museum visitors will be getting an attractive discount of 30% on combo packages of NRM. National Rail Museum (NRM), managed by Indian Railway, is one of the top edutainment tourist destinations of Delhi NCR area having an annual footfall of more than 5 lakhs. Madame Tussauds wax museum is a leading chain of Wax Museums in the world.

15)Which of the following theatre level maritime war exercise will be conducted by Indian Navy?

Ans:-TROPEX

Explanation:-The Indian Navy will conduct its flagship Theatre Level Operational Readiness Exercise (TROPEX), from January end till early March 2019. This will provide an opportunity to test the combat capability of the Indian Coast Guards (ICG), and the Indian Air Force (IAF), Indian Navy, and strengthened inter-operability and joint operations in complex conflict situation. As part of TROPEX, the Indian Navy would also conduct a large-scale coastal defence exercise 'Exercise Sea Vigil' which will involve all stakeholders across mainland and island territories. The aim of such an exercise is to test the robustness of our entire coastal security apparatus through his massive exercise. The yearly exercise takes place on the Western Seaboard which will have participation of over 45 ships from both the Western and Eastern Naval Commands of the Indian Navy, including the aircraft carrier INS Vikramaditya, submarines including the nuclear powered Chakra, naval aircraft MiG-29K, helicopters as well as ships from the ICG.

16)The first India-ASEAN InnoTech Summit was held in ______.

Ans:-New Delhi

Explanation:-The first India-ASEAN InnoTech Summit was held in New Delhi. It was organized by Federation of Indian Chambers of Commerce & Industry (FICCI) in association with the Department of Science & Technology (DST) and Ministry of External Affairs. Participating ASEAN member countries in this summit were Brunei, Cambodia, Indonesia, Lao PDR, Malaysia, Myanmar, Philippines, Singapore, Thailand, Vietnam.

17) Who won the first ever women's Ballon d'Or award?

Ans:-Ada Hegerberg

Explanation:-Olympique Lyonnais striker Ada Hegerberg became the first woman to be awarded a Ballon d'Or at a grand ceremony in Paris. The 23-year-old was announced the first ever winner of the Women's Ballon d'Or by former France international player David Ginola before she gave an inspiring speech encouraging more girls to play football. She has won four league and three Champions League titles with Lyon.

18)The 24th Conference of the Parties (COP 24) to the UNFCCC will be held in ______, Poland.

SSCE 8981426494/8296260082 Downloaded from - www.onlinessce.com Page 24

Ans:-Katowice

Explanation:-The 24th Conference of the Parties (COP 24) to the UNFCCC (United Nations Framework Convention on Climate Change) has started in Katowice, Poland from 3- 14 December 2018. The conference is expected to finalize the rules for implementation of the Paris Agreement on climate change under the Paris Agreement work programme (PAWP). It will also include a number of high-level events, mandated events, action events and roundtables. About 20 thousand people from 190 countries will take part in the event, including politicians, representatives of non-governmental organizations, scientific community and business sector.

19)The first leader who lose the Freedom of Paris award is;

Ans:-Aung San Suu Kyi

Explanation:-Aung San Suu Kyi, the Nobel peace laureate & Myanmar leader, will be stripped of Freedom of Paris award over her failure to speak out against a crackdown on Myanmar's Rohingya minority. The move, which follows similar decisions by Glasgow, Edinburgh and Oxford, would make Myanmar's de facto leader the first person to lose the freedom of the French capital, a purely symbolic award. The move will be finalised by the city council at a meeting in mid-December 2018. Apart from this, Suu Kyi has already been stripped of her honorary Canadian citizenship and her Amnesty International's "Ambassador of Conscience Award". In 2017, nearly 5,00,000 people belonging to Myanmar's Rohingya minority have been displaced after violence allegedly instigated by the country's military, causing a major humanitarian crisis.

20)The 2nd Russia-India-China 'RIC' Trilateral Summit has recently held in which of the cities?

Ans:-Buenos Aires

Explanation:-In Argentina, the 2nd Russia-India-China (RIC) Trilateral Summit was held in Buenos Aires after a gap of 12 years, on the sidelines of the G-20 summit to discuss cooperation in various areas. The three leaders - Prime Minister Narendra Modi, Chinese President Xi Jinping and Russian President Vladimir Putin - agreed to have regular consultations to jointly promote international and regional peace and stability, to strengthen cooperation through BRICS (Brazil, Russia, India, China and South Africa), the SCO (Shanghai Cooperation Organisation) and the East Asia Summit (EAS) mechanisms, to address global challenges such as terrorism and climate change, and to encourage peaceful resolution of all differences.

Daily Current Affairs 6thDec,2018

1)Where was the 2nd Meeting of NITI Forum held?

Ans:- Guwahati

SSCE 8981426494/8296260082 Downloaded from - www.onlinessce.com Page 25

Explanation:-The second meeting of the NITI Forum for the North East is being held in Guwahati, Assam. Minister of State for Development of North Eastern Region, Dr Jitendra Singh and Vice Chairman of NITI Aayog Rajiv Kumar attended the meeting. NITI Forum for North East was constituted this year to periodically review the development status in the region. The members of the forum include Secretaries of various central ministries, Chief Secretaries of all eight North-eastern states, senior government officials and experts from various fields.

2)Which of the following signed MoU with the Maharashtra Government to promote Fintech?

Ans:-Economic Development Board

Explanation:-The Economic Development Board (EDB), the investment promotion arm of the Kingdom of Bahrain signed Memorandum of Understanding (MoU) with Maharashtra Government to promote FinTech in their respective markets. The MoU was signed by Dr. Simon Galpin, Managing Director of EDB, and Mr. S.V.R. Srinivas, IAS Principal Secretary, Directorate of Information Technology.

3)Who of the following is the highest-earning celebrity in India, according to Forbes?

Ans:-Salman Khan

Explanation:-Salman Khan has been named the highest-earning celebrity in India for the third time in a row by Forbes, with earnings of ₹253.25 crore. The second spot was captured by Indian cricket team skipper Virat Kohli, with Rs 228.09 crore to his name, to become the first sportsperson to reach the position. Akshay Kumar came third on the Forbes India Celebrity 100 2018 with Rs 185 crore, followed by Deepika. The fifth spot went to former Indian cricket team captain Mahendra Singh Dhoni who earned Rs 101.77 crore during 2018.

4)The first Indian to scale Sahayadri mountain range.

Ans:-Tuhin Satarkar

Explanation:-Tuhin Satarkar, became the first Indian to climb three routes in the treacherous Sahayadri mountain range in 12 days, from November 16 to 28. The pune based bouldering(rock climbing) sensation climbed- Dhodap, Jivdhan and Naneghat, three famous peaks which are known to have been ascended by Chatrapati Shivaji Maharaj and his Marathi Malvas. Both the parents ofthe Red Bull athlete Tuhin were also climbers. Tuhin took to the vertical sport at the age of eight and has been pushing boundaries ever since.

5)World's 1st 3D-Printed e-Bike was manufactured by ______.

Ans:-Germany

SSCE 8981426494/8296260082 Downloaded from - www.onlinessce.com Page 26

Explanation:-German firm BigRep claims it has produced the world's first fully functioning e-motorbike NERA using a 3D printer. According to the company, NERA has groundbreaking features like airless tires, embedded electronics, and forkless steering. The bike was conceptualized and developed in 12 weeks and includes 15 parts, which are all 3D printed except its motor and battery. The name is derived from the slogan "New Era".

6)How many projects are sanctioned under National Mission for Clean Ganga over 24 thousand crore?

Ans:-254

Explanation:-The government sanctioned 254 projects worth over 24 thousand crore rupees under National Mission for Clean Ganga. As per Water Resources, River Development and Ganga Rejuvenation Minister Nitin Gadkari, around five thousand crore rupees have been spent in the last four years under it. The Announcement was made at the India Water Impact Summit 2018 in New Delhi.

7)Recently, India and this country signed 3,500 crore currency swap agreement.

Ans:-UAE

Explanation:-India and UAE signed 3,500 crore currency swap agreement. This agreement will boost investment and enable direct trade without using dollars or other international currencies. The agreement was signed after the 12th India-UAE joint commission meeting co-chaired by External Affairs Minister Sushma Swaraj and the UAE's Foreign Minister Sheikh Abdulla bin Zayed al Nahyan in Abu Dhabi. In 2015, China's central bank extended a currency swap agreement with the UAE Central bank worth $5.54 billion.

8)What will be the GDP rate of India for March 2019?

Ans:-7.2%

Explanation:-Moody's expects the real gross domestic product (GDP) in India to grow 7.2 per cent in the year ending March 2019 and 7.4 per cent in the following year. Earlier Moody's Investors service growth prediction for India was 7.5% in 2018-19, 7.5% in 2019- 20 released on 23rd August,2018. This growth is driven by investment growth and strong consumption. The stable outlook is based on six parameters - operating environment, asset quality, capital, funding and liquidity, profitability and efficiency, and government support.

9)Which state government launched the PEETHA scheme which is a part of the 3T initiative of Technology, Transparency and Team Work model.

Ans:-Odisha

Explanation:-IN a move to reinforce brand Naveen and give it a national perspective after the success of the Hockey Men's World Cup, Odisha Government launched yet another

SSCE 8981426494/8296260082 Downloaded from - www.onlinessce.com Page 27

initiative, Peoples Empowerment - Enabling Transparency and Accountability of Odisha Initiatives (PEETHA). The Scheme is aimed at creating awareness about various schemes and improving transparency in the distribution of individual and social benefits. This is part of the 3T initiative of Technology, Transparency and Team Work model. The PEETHA is a sub-scheme of the Odisha government's flagship Ama Gaon Ama Bikas Yojana.

10)Who launched 'Chance of Approval' the industry-first feature that will help loan applicants choose their most-suited lender?

Ans:-Paisabazaar

Explanation:-Paisabazaar.com, India's largest online marketplace for financial products has launched an industry-first feature that will help loan applicants choose their most-suited lender on its platform. Named 'Chance of Approval', this advanced predictive algorithm has been created by Paisabazaar.com by using the last four years of lending data. While this is the first such initiative in India, globally a few leading players, like Credit Karma in the US, have developed similar features to improve the acceptance rate on their platforms.

11)Name the Indian cricketer who was honoured by Bradman Museum, recentlt?

Ans:-Virat Kohli

Explanation:-The Indian cricket team captain Virat Kohli has received a special honour as a signed jersey that he donated after scoring his hundred in the fourth Test at the Sydney Cricket Ground (SCG) during India's tour of Australia in 2014-15 has been placed next to Sachin Tendulkar's portrait at the Bradman Museum. The Indian Captain Virat Kohli signed and presented his shirt to Cricket Australia Liaison Officer Pearce Gibbons in appreciation of his assistance to the Indian side throughout the summer at the conclusion of the fourth test and thereafter Pearce Gibbons donated the shirt to the museum for public display. Indian cricketers like the legendary Kapil Dev and Tendulkar have a memento celebrating their achievements placed at the iconic museum. The Museum is also known as 'The Home of Cricket Memories'.

12)Which country signed an agreement with Peru for co-operation and mutual assistance in Customs Matters?

Ans:-India

Explanation:-India signed an agreement with Peru for co-operation and mutual assistance in Customs Matters. It provides a legal framework for the sharing of information and intelligence between the Customs authorities of the two countries. The agreement will also help in the proper application of Customs laws, prevention and investigation of Customs offenses. According to the Finance Ministry, the agreement signed in New Delhi is also expected to facilitate trade and ensure the efficient clearance of goods traded between the two sides.

SSCE 8981426494/8296260082 Downloaded from - www.onlinessce.com Page 28

13)Which country has become the first Latin American country to join China's Belt and Road Initiative (BRI)?

Ans:-Panama

Explanation:-Panama has become the first Latin American country to join China's Belt and Road Initiative (BRI). China established diplomatic relations with Panama last year. Chinese President Xi Jinping and his Panamanian counterpart, Juan Carlos Varela signed 19 deals, including: An extradition treaty An arrangement where China will provide a non-specified amount of non-reimbursable aid to carry out various projects. Memoranda of understanding on commercial, tourist, educational matters. Other deals on trade, infrastructure and banking.

14)Name the Indian origin woman who was appointed as South Africa's 1st female chief prosecutor.

Ans-Shamila Batohi

Explanation:-South African President Cyril Ramaphosa has appointed Indian-origin Shamila Batohi as the country's first female chief prosecutor. Batohi will start her new role as the National Director of Public Prosecutions (NDPP) in February 2019. Batohi is a former state prosecutor from KwaZulu-Natal and has served as a senior adviser to the prosecutor in the International Criminal Court since 2009.

15)International Day of Persons with Disabilities was observed on which date?

Ans:-3rd December

Explanation:-December 3 is observed as United Nations International Day of Persons with Disabilities or World Disability Day.The theme for this year is "Empowering persons with disabilities and ensuring inclusiveness and equality". This theme focuses on the empowering persons with disabilities for the inclusive, equitable and sustainable development envisaged in the 2030 Agenda for Sustainable Development.Estimated one billion people living with disabilities worldwide. Since 1992 December 03 is celebrated as United Nations international Day of Persons with Disabilities.

16)Air passengers may be allowed to make mobile calls and surf the internet during flights in the Indian airspace from;

Ans:-January 2019

Explanation:-Phone calls, internet surfing may be allowed in flights from January 2019. The Telecom Commission - the highest policy-making body of the Department of Telecom - had cleared a proposal for allowing wider in-flight connectivity that is now available in most of the developed markets. A separate category of licensees - in-flight service providers - will be created for offering such services, and licence fee for such niche providers will be pegged at Rs 1.

SSCE 8981426494/8296260082 Downloaded from - www.onlinessce.com Page 29

17)Prime Minister Narendra Modi will inaugurate India's longest rail-cum-road bridge "Bogibeel Bridg" located in the state of;

Ans:-Assam

Explanation:-Prime Minister Narendra Modi will inaugurate the Bogibeel Bridge, India's longest rail-cum-road bridge, falling in the eastern part of Assam and Arunachal Pradesh on December 25. This project was first cleared by former Prime Minister HD Deve Gowda in 1997. The bridge spans over the Brahmaputra river in the north eastern state of Assam between Dhemaji district and Dibrugarh district.

18)Who will be the head of the committee to look at selling 149 fields of ONGC, OIL to private companies?

Ans:-Rajiv Kumar

Explanation:-A six-member committee has been constituted by the government which will look at selling as many as 149 small and marginal oil and gas fields of state-owned ONGC and Oil India to private and foreign companies to boost domestic output. The committee will be headed by NITI Aayog Vice Chairman Rajiv Kumar and includes Cabinet Secretary P K Sinha, Economic Affairs Secretary Subhash Chandra Garg, Oil Secretary M M Kutty, NITI Aayog CEO Amitabh Kant and ONGC Chairman and Managing Director Shashi Shanker. Earlier on 12thOctober, Prime Minister Narendra Modi called a meeting to review domestic production profile of oil and gas and the roadmap for cutting import dependence by 10 per cent by 2022 from the over 77 per cent dependence in 2014-15. But the dependence has only increased and is now over 83 per cent.

19)The 80th Session of the Policy Commission of the World Customs Organization was held in ______.

Ans:-Mumbai

Explanation:-The 80th Session of the Policy Commission of the World Customs Organization (WCO) was held in Mumbai, Maharashtra . The Session is being organized by the WCO and hosted by the Central Board of Indirect Taxes and Customs (CBIC), India. The Indian delegation was headed by CBIC Chairman, S Ramesh. The custom heads of more than 30 countries participated in the 3-day long discussion on various matters related to customs. Some of the important topics discussed during the meeting include trade facilitation, controlling illicit financial flow, performance measurement, challenges of small island economies, etc. The WCO is the only global organization which defines global standards and procedures for customs clearances at the border and their implementation. It is headquartered at Brussels, Belgium.

Daily Current Affairs 7thDec,2018

SSCE 8981426494/8296260082 Downloaded from - www.onlinessce.com Page 30

1)Who inagurated the Dr. Bhim Rao Ramji Ambedkar School Of Media Empowerment (ASME), to train Dalit and tribal youth in journalism?

Ans:-Thaawarchand Gehlot

Explanation:-On the 62nd death anniversary of B R Ambedkar, Social Justice and Empowerment minister Thaawarchand Gehlot launched the website of a first-of-its-kind media school, Dr. Bhim Rao Ramji Ambedkar School Of Media Empowerment (ASME), to train Dalit and tribal youth in journalism. The media school, ASME supported by the Indian Institute of Mass Communication (IIMC), will have its branches in Pune, Assam, Uttarakhand, and other regions. The Pune branch will begin shortly.

2)Ministry of Finance approves ______to upgrade Doordarshan and .

Ans:-Rs 850 crore

Explanation:-Finance ministry has approved Rs 850 crore investment to upgrade Doordarshan (DD) and All India Radio (AIR). The sanctioned amount will be invested over FY19 and FY20. Decision came after Centre has started a manpower audit of Prasar Bharati, acting on the recommendation of an expert committee in 2014 led by Sam Pitroda which recommended that Prasar Bharati 'undertake a comprehensive manpower audit and human resources planning exercise to map workforce requirements for the future, in line with the public broadcaster's mandate'.

3)India's heaviest communication satellite that is launched successfully from French Guiana.

Ans:-GSAT-11

Explanation:-ISRO's heaviest and most advanced high throughput communication satellite 'GSAT-11' was successfully launched by an European Ariane-5 vehicle (Flight VA246) rocket from Guiana Space Centre at Kourou in French Guiana. Initially, the satellite is placed in the Geosynchronous Transfer Orbit (GTO) and will be raised to the Geostationary Orbit (36,000 km above the equator) through phase-wise orbit-raising manoeuvres in the days ahead, using its on-board propulsion systems. GSAT-11 will be positioned at 74-degree east longitude in the geostationary orbit.The mission of the 5,854-kg giant 'bird' is to provide high data rate connectivity to users of Indian mainland and islands through 32 user beams in Ku- band and 8 hub beams in Ka-band. It will boost the broadband connectivity to rural and inaccessible Gram Panchayats under BharatNet Project which is part of the Digital India initiative. The satellite is capable of providing high bandwidth connectivity with speeds of up to 16 gigabytes per second. GSAT-11' s co-passenger is South Korea's GEO-KOMPSAT- 2A, a meteorology satellite.

4)Who has been selected to host the 2019 Academy Awards?

SSCE 8981426494/8296260082 Downloaded from - www.onlinessce.com Page 31

Ans:-Kevin Hart

Explanation:-Kevin Hart (39-year-old) has been selected to host the 2019 Academy Awards, also known as the Oscars, for the first time. The 91st Academy Awards ceremony is scheduled to take place on February 24, 2019. Hart takes over hosting duties from Jimmy Kimmel who presided over the last two ceremonies. The Oscar nominations will be announced on January 22, 2019.

5)AI 4 All Global Hackathon was organised by ______.

Ans:-NITI Aayog

Explanation:-NITI Aayog organised 'AI 4 All Global Hackathon' to source sustainable, innovative and technologically-enabled solutions to address various challenges in the development space, with the vision of 'Artificial Intelligence, AI for All'. Taking the initiative forward, NITI Aayog is now partnering with Perlin - a Singapore-based AI start up - to launch the 'AI 4 All Global Hackathon', and is inviting developers, students, start-ups and companies to develop AI applications to make significant positive social and economic impact for India.

6)Name the IT firm which will buy 7 IBM Products over Rs 12,700 crore.

Ans:-HCL Technologies

Explanation:-Noida-based IT major HCL Technologies will acquire seven IBM products by mid-2019 for $1.8 billion (over Rs 12,700 crore) in its biggest ever acquisition so far. The all-cash deal entails software in areas including security, marketing, collaboration solutions, and represents a total addressable market of over $50 billion. The deal includes the transfer of some IBM employees as well.

7)Noted agriculturalist Nel Jayaraman passed away recently. He belongs to which state?

Ans:-Tamil Nadu

Explanation:-Noted agriculturalist Nel Jayaraman (54), who pioneered the protection and propagation of indigenous paddy varieties in Tamil Nadu, passed away in Chennai on December 6, 2018. Hailing from Thiruthuraipoondi, in the heart of Tamil Nadu's delta rice bowl, Jayaraman has been credited with documenting and protecting more than 170 paddy varieties.His diligent work in the field of protecting the gene prototypes of indigenous paddy varieties won him the National Award in 2015 and a state award in 2011.

8)Which company opened its new technology and innovation hub in Hartford?

Ans:-Infosys

SSCE 8981426494/8296260082 Downloaded from - www.onlinessce.com Page 32

Explanation:-Infosys opened its new technology and innovation hub in Hartford, Connecticut. It has hired more than 7,000 American workers in the last 18 months. The Hartford hub will help Infosys work more closely with its clients in the region and serve as the global hub for Infosys' InsurTech and HealthTech efforts. It is expected that labs at the Hartford hub will bring together expertise in business and technologies such as blockchain, extended reality and cognitive capabilities with techniques like design thinking, agile and DevOps.

9)eATM : Instant fund credit feature to get cash within 30 minutes launched by ______.

Ans:-ICICI Securities

Explanation:-ICICI Securities (ISec), a brokerage firm, launched the 'eATM' instant fund credit feature to enable real-time payments in bank accounts upon selling stocks in Bombay Stock Exchange (BSE). This feature is available to retail investors who can get payment against sell of stocks in BSE instantly compared to earlier waiting time of T+2 days. The payment is made directly to the bank accounts to within 30 minutes and the limit is Rs. 50,000 per client per day. This is being offered to all ICICIdirect customers at no additional cost and will be available to stocks sold on the BSE. It is open to all trades in cash segment on nearly 600 stocks traded in BSE.

10)Who of the following has been elected to Asia Pacific seat on the UN's Committee on Economic, Social and Cultural Rights?

Ans:-Preeti Saran

Explanation:-A former senior Indian diplomat, Preeti Saran, has been elected unopposed to an Asia Pacific seat on the UN's Committee on Economic, Social and Cultural Rights (CESCR). Saran was elected to the CESCR for a four-year term beginning on the 1st of January 2019. All States are obliged to submit regular reports to the committee on how the rights are being implemented. The committee examines each report and addresses its concerns and recommendations to the State party in the form of concluding observations. The panel meets in Geneva and holds two sessions per year.

11)Where was the Conference on EU-India Partnership for Cultural Heritage Conservation held?

Ans:-New Delhi

Explanation:-A two-day "Conference on EU-India Partnership for Cultural Heritage Conservation" commenced at the National Museum Auditorium, Janpath, New Delhi. It concluded on December 5, 2018. To enhance EU-India collaboration in the field of cultural heritage conservation. The conference was inaugurated by: Minster of State (I/C) Shri. Hardeep Singh Puri. As part of this, the Delegation of the European Union together with the National Museum Institute, Government of India, will convene 20 Indian and 18 European

SSCE 8981426494/8296260082 Downloaded from - www.onlinessce.com Page 33

experts. t also marked the pinnacle of the European Year of Cultural Heritage (EYCH), which will conclude on December 6-7, 2018.

12)With which country, India signed an agreement on Joint Activities under the Human Spaceflight Programme recently?

Ans:-Russia

Explanation:-The Union Cabinet chaired by Prime Minister Narendra Modi has signed an MoU between India and Russia on Joint Activities under the Human Spaceflight Programme on Dec 6, 2018. It includes an MoU between India and Tajikistan on Cooperation on Peaceful Uses of Space Technology for Development. Agreement between India and Algeria on Cooperation in the field of Space Sciences, Technologies and Applications. MoC between India and Japan in the field of Healthcare and Wellness.

13)Who released a book 'Blue Waters Ahoy!' on 04 December 2018?

Ans:-Admiral Sunil Lanba

Explanation:-Admiral Sunil Lanba, chief of the Naval Staff, released a book titled 'Blue Waters Ahoy!'. It tells Indian Navy's History from 2001-10. It was unveiled on the occasion of the Indian Navy Day, celebrated every year on 04 December 2018. The book is authored by Vice Admiral Anup Singh, who retired as the Flag Officer Commanding-in-Chief of Eastern Naval Command in 2011.

14)Which team won the Admiral's Cup 2018?

Ans:-Italy

Explanation:-Team Italy won the Admiral's Cup 2018. The ninth edition of 'Admirals Cup' Sailing Regatta (2018) concluded with a glittering closing ceremony at Ettikulam Beach at Indian Naval Academy (INA), Ezhimala. Team Singapore was the Runners-Up and Team USA came third. The host country team (Indian Naval Academy) finished fourth in the overall position of Admiral's Cup 2018.

15)Which city will host the 2023 World Athletics Championship?

Ans:-Budapest

Explanation:-The International Association of Athletics Federations (IAAF) announced that Budapest will host the 'World Athletics Championship' in 2023. Budapest is the capital of Hungary. Budapest has twice hosted the IAAF world indoor championships and the European athletics championships. The 2019 'World Athletics Championship' will be held in Doha and the 2021 edition in Eugene, Oregon.

16)World's first country to make all public transport free from 2020.

SSCE 8981426494/8296260082 Downloaded from - www.onlinessce.com Page 34

Ans:-Luxembourg

Explanation:-Luxembourg is set to become the first country in the world to make all public transport free, implementing the move in 2020. The move is aimed at addressing environmental issues and the problem of traffic congestion. Earlier, Luxembourg introduced free transport for all people under the age of 20.

17)Department of Industrial Policy and Promotion, Government of Goa and this ministry hosted the annual Startup India Venture Capital Summit in Goa.

Ans:-Ministry of Commerce and Industry

Explanation:-The Department of Industrial Policy and Promotion (DIPP) in the Ministry of Commerce and Industry and the Government of Goa, hosted the annual Startup India Venture Capital Summit in Goa. The theme of the Summit was 'Mobilizing Global Capital for Innovation in India.' The event showcased the Indian startup opportunity for funds from around the world. The key objectives of the Summit were to showcase the India opportunity, increase capital flow for Indian startups and further promote ease of doing business.

18)According to EY FinTech Adoption Index, what is the position of India?

Ans:-2nd

Explanation:-According to EY FinTech Adoption Index, India has the second highest FinTech adoption rate among digitally active consumers at 52percent, only second to China at 69 percent. Usage is significantly high in large cities with a 66 percent adoption rate followed by 51 percent usage and adoption in small and medium cities. In India, fintech industry is being encouraged by the various government initiatives such as Jan Dhan Yojana, Aadhaar and the emergence of UPI which provide a good foundation to boost financial inclusion in India. Rural India lags at 33 percent. Money transfers and payments services leading the charge in India with adoption standing at 72 percent. Insurance is the next popular category with an adoption rate of 47 per cent, much higher than the global average of 24 per cent.

19)International Volunteer Day (IVD) is observed every year on 5 December. The theme of 2018 IVD is;

Ans:-Volunteers build Resilient Communities

Explanation:-International Volunteer Day (IVD) is observed every year on 5 December. It is an international observance mandated by the UN General Assembly in 1985. The theme for IVD 2018 is 'Volunteers build Resilient Communities'. IVD is a chance for individual volunteers, communities and organisations to promote their contributions to development at the local, national and international levels.

Daily Current Affairs 8thDec,2018

SSCE 8981426494/8296260082 Downloaded from - www.onlinessce.com Page 35

1)With which country, India signed Two MoUs in the Renewable Energy sector to enhance the technology and bring in power efficiency?

Ans:-Sweden

Explanation:-Sweden and India signed Two Memorandum of Understandings (MoUs) in the Renewable Energy sector to enhance the technology and bring in power efficiency. Among the two MoUs signed, the first was signed between Swedish firm Spowdi and EMVEE for setting up local manufacturing and assembly unit in Bangalore and the second MoU was inked between Swedish Neutral and Tata Power DDL for setting up pilot for earth fault protection in Delhi. The MoUs were signed on the sidelines of inauguration of the 'Sustainability by Sweden - Showroom India' themed as "Sustainability is Everybody's Business"by Swedish Energy, Agency Business Sweden and Embassy of Sweden under the Sweden India Nobel Memorial Programme. The India-Sweden Innovations' Accelerator (ISIA) programme is a part of intergovernmental cooperation between India and Sweden in the area of new and renewable energy technology.

2)With which bank, HDFC ERGO General Insurance Company ties up for offerings non-life insurance to the customers of the Bank?

Ans:-Federal Bank

Explanation:-HDFC ERGO General Insurance Company, India's leading private sector general insurance company, and Federal Bank, announced tie-up for the distribution of HDFC ERGO's range of non-life insurance offerings to the customers of the Bank through its network branches. This Corporate Agency Tie-up offers comprehensive Insurance solutions to customers both digitally and physically, which benefits the customers in many ways. Federal Bank's deep-rooted presence in Southern markets allows the Company to strengthen its foothold within the region.

3)Who inagurated the 2018 All India Police Shooting Competition?

Ans:-Shri Kiren Rijiju

Explanation:-Minister of State for Home Kiren Rijiju inaugurated the 19th All India Police (AIPDM) Shooting Competition-2018 at Manesar, Gurugram in . The competition will continue till 11th of this month. More than 750 participants of 31 teams of State Police Forces and Central Armed Police Forces are participating in the event.

4)Which state launched a SMART initiative for agribusiness,rural transformation?

Ans:-Maharashtra

Explanation:-Maharashtra Chief Minister Devendra Fadnavis launched a unique initiative called- State of Maharashtra's Agribusiness and Rural Transformation or "SMART". This will be implemented in 10,000 villages with an objective to achieve sustainable farming

SSCE 8981426494/8296260082 Downloaded from - www.onlinessce.com Page 36

within the next 3 years. It's main aim is to propel rural transformation and double farmers income by 2022. This is a World Bank (WB) assisted project which has a net worth of 300 million US dollars. Of this WB has provided financial assistance of $210 million and remaining will be Contributed by State government and Village Social Transformation Foundation (VSTF).

5)Name the professor who won the 2018 Innovative Young Biotechnologist Award.

Ans:-Dr. Rajanish Giri

Explanation::-Dr. Rajanish Giri,an assistant professor from the Indian Institute of Technology-Mandi (IIT-Mandi),has been awarded the Innovative Young Biotechnologist Award (IYBA) 2018. Dr.Giri was selected for his proposed innovative idea on Zika virus Capsid Folding and inhibitor discovery. The Department of Biotechnology (DBT), Ministry of Science and Technology selected Dr.Giri for the award. Along with the award he received a research grant, for three years, to delve more insight into the biophysical research on Zika Capsid protein system.

6)Which ministry Conferred with the Skoch Award?

Ans:-Ministry of New and Renewable Energy

Explanation:-Ministry of New and Renewable Energy, Government of India has been conferred the Skoch Award for National Significance at an event held recently in New Delhi. The award was received by Secretary, Ministry of New Renewable Energy Shri Anand Kumar. The award has been conferred on the Ministry considering its purpose and critical role played in installing about 73 GW renewable energy capacity in the country.

7)Who became the fastest cricketer to take 200 Test wickets breaking an 82-year-old record?

Ans:-Yasir Shah

Explanation:-Pakistan's leg-spinner Yasir Shah became the fastest cricketer to take 200 Test wickets breaking an 82-year-old record, beating Australian leg-spinner Clarrie Grimmett's record of 36 Tests set against South Africa in Johannesburg in 1936. Yasir achieved this by taking New Zealand's batsman Will Somerville wicket in Abu Dhabi.

8)The Indian Coast Guard has conducted the Regional Level Marine Oil Pollution Response Exercise titled 'Clean Sea - 2018' ______.

Ans:-Port Blair

Explanation:-The Indian Coast Guard has conducted the Regional Level Marine Oil Pollution Response Exercise titled 'Clean Sea - 2018' at sea off Port Blair. Coast Guard ships Viswasth, Vijith, Rajveer, Rajshri, 4 interceptor boats, and its air assets Dornier and Chetak

SSCE 8981426494/8296260082 Downloaded from - www.onlinessce.com Page 37

helicopters participated in the exercise. The preparedness of all agencies during the exercise was conducted under observation of Inspector General Maneesh V Pathak, Commander Coast Guard Region, Andaman and Nicobar Islands.

9)Who has been appointed as the Chairman and Managing Director (CMD) of New India Assurance?

Ans:-Atul Sahai

Explanation:-Atul Sahai has been appointed as the Chairman and Managing Director (CMD) of the country's largest general insurance company New India Assurance for a five year period. The post had been lying vacant since July 2018 after former chairman and managing director G Srinivasan retired.

10)This country is planning to increase its green cover by building 20,000 national forest villages by 2020 in the country.

Ans:-China

Explanation:-China is planning to increase its green cover by building 20,000 national forest villages by 2020 in the country. As the country has been facing the problem of severe pollution, it has spent over $100 billion on trees and its total forest cover area is almost 22%. Liu Dongsheng, Deputy Director of China's National Forestry and Grassland Administration said that the country will continue to enhance the ecological environment in villages across the country. It also plans to bring the green coverage rate in rural areas to 30%.

11)Who gets SEBIs approval to launch 2 new schemes?

Ans:-Yes Asset Management

Explanation:-Yes Bank, a Private sector lender, announced that its wholly-owned subsidiary Yes Asset Management received SEBI's approval to launch two mutual fund schemes. The asset management company is to launch two schemes, Yes Liquid Fund and Yes Ultra Short Term Fund. Yes Asset Management, India is the latest Indian company to enter the mutual fund segment.

12)Who was appointed as the US Ambassador to the United Nations?

Ans:-Heather Nauert

Explanation:-US President Donald Trump nominated State Department spokeswoman Heather Nauert as the US Ambassador to the United Nations. The former Fox News presenter's appointment to the UN role will have to be approved by the US Senate. Nauert will replace Nikki Haley, who will be leaving the post by the end of 2018. She became the State Department's spokeswoman in April 2017 and was named as the acting undersecretary for public diplomacy and public affairs in early 2018.

SSCE 8981426494/8296260082 Downloaded from - www.onlinessce.com Page 38

13)Who has approved India's proposal to observe an International Year of Millets in 2023?

Ans:-FAO

Explanation:-Food and Agriculture organisation, FAO Council has approved India's proposal to observe an International Year of Millets in 2023. Agriculture Minister Radha Mohan Singh stated that the 160th session of the FAO Council, in Rome approved the proposal. The FAO Council has also approved India's membership to the Executive Board of the United Nations World Food Program for 2020 and 2021.

14)Which IIT institute launched the Centre for Technology and Policing?

Ans:-IIIT Delhi

Explanation:-The Indraprastha Institute of Information Technology-Delhi (IIIT-D), New Delhi launched the Centre for Technology and Policing. It aimed at strengthening the Delhi Police by providing the latest technology in the field of crime prevention and control. Lieutenant-Governor of Delhi Anil Baijal inaugurated the Centre. The research activities at the Centre will help the police control criminal activities. It will assist the police in criminal identification, law and order management, cyber policing, traffic management, and combating terrorist activities using Artificial Intelligence, social media analysis, biometrics, image processing, big data, and network forensics. It also plans to train police personnel to acquire knowledge in using the latest tools and techniques to aid other aspects of policing such as traffic management, disaster management, urban crime, and terrorism.

15)Which country launched the 1st Mission to Land on the far side of the Moon?

Ans:-China

Explanation:-China launched the first-ever spacecraft, Chang'e 4, that will attempt landing on the far side of the Moon, which always shows the same face to Earth as it's close enough to be locked by the planet's gravitational field. The Chang'e-4 mission will see a static lander and rover touch down in Von K?rm?n crater, located on the side of the Moon which never faces Earth. The payload blasted off atop a Long March 3B rocket from Xichang Satellite Launch Center.

16)'Ideate for India' is the joint collaboration between National e-governance Division and which technological giant?

Ans:-Intel

Explanation:-Union Minister Prasad has launched National Challenge for Youths 'Ideate for India - creative solutions using technology' at New Delhi on December 6. The aim of this National Challenge is to give school students across the country a platform and opportunity to become solution creators for the problems they see around them and their

SSCE 8981426494/8296260082 Downloaded from - www.onlinessce.com Page 39

communities. It is open to students of classes 6 - 12 all across the country - all 29 States and 7 Union Territories and aims to reach out to at least 1 million youth over the next 3 months. "Ideate for India" will help inculcate innovation skills in youth and equip all students to utilise technology for the betterment of their communities and the society at large. Simultaneously, this will prepare them to contribute towards the realization of Digital India goals. The challenge has been designed and launched by the National e-governance Division in collaboration with Intel India, with support from the Department of School Education and Literacy (DoSE&L) of Ministry of Human Resources Development (HRD).

17)Who has been appointed the new Chief Economic Advisor (CEA)?

Ans:-Krishnamurthy Subramanian

Explanation:-Dr Krishnamurthy Subramanian has been appointed the new Chief Economic Advisor (CEA) for a period of three years. The post of CEA had been lying vacant since Arvind Subramanaian left the finance ministry after a four-year stint. Dr Krishnamurthy is currently working as an Associate Professor and the executive director at the Indian School of Business (IBS), Hyderabad. The CEA is the ex-officio cadre controlling authority of the Indian Economic Service (IES) & is under the direct charge of the Minister of Finance.

18)Which airlines becomes first Indian airline to have 200 aircraft?

Ans:-IndiGo

Explanation:-Four new aircraft have joined the fleet, including two Airbus A320 ceo (VT- IKA and VT-IKB) and two A320 neo (VT IZK AND VT-IZI), taking the fleet count to 200 and number of deliveries to 226. IndiGo currently offers over 1,200 flights to 63 domestic destinations, which include 49 domestic and 14 international ones. The airline, founded by billionaires Rahul Bhatia and Rakesh Gangwal, started operations in August 2006 with a Delhi-Imphal flight.

19)Who won 2018 'IAAF Female Athletes of the Year' award?

Ans:-Caterine Ibarguen

Explanation:-World marathon record holder Eliud Kipchoge of Kenya and 2018 triple and long jump Diamond League champion Caterine Ibarguen of Colombia are the 2018 IAAF Male and Female Athletes of the Year. Both are first-time winners of the award. Kipchoge is just the second Kenyan ever to win the award after David Rudisha in 2010.

20)Who of the following are nominated for Golden Globe awards?

Ans:-Lady Gaga and Bradley Cooper

Explanation:- Lady Gaga and Bradley Cooper have earned nominations at the 2019 Golden Globe awards for their performances in the film 'A Star is Born'. Gaga was nominated

SSCE 8981426494/8296260082 Downloaded from - www.onlinessce.com Page 40

alongside Nicole Kidman and Melissa McCarthy in the 'Best Actress - Drama' category. Cooper was nominated in the 'Best Actor - Drama' category along with 'Bohemian Rhapsody' star Rami Malek, among others.

Daily Current Affairs 9thDec,2018

1)Who won the 2018 Turner Award, Britain's prestigious contemporary art award in London?

Ans:-Charlotte Prodger

Explanation:-Glasgow based artist and film maker Charlotte Prodger won the 2018 Turner Award, Britain's prestigious contemporary art award, at Tate Britain in London, UK. Prodger, 44, won this award for her 33-minute film called 'BRIDGIT', that explores queer identity, which was shot in an iPhone. She would receive $31,785 as a prize. The Award was instituted in 1984 by a group named Patrons of Art.

2)Who laid the Foundation stone of Pusa Kisan Haat in ICAR, New Delhi?

Ans:-Radha Mohan Singh

Explanation:-Shri Radha Mohan Singh, Union Agriculture and Farmers Welfare Minister laid the Foundation stone of Pusa Kisan Haat in Indian Council of Agricultural Research's (ICAR) Agricultural Technology Information Centre (ATIC) in New Delhi. Pusa Kisan Haat will be built across 2.5 acres which includes 60 stalls measuring 3m x 3m where farmers can sell their agricultural products.

3)The theme for 2018 International Civil Aviation Day is;

Ans:-Working Together to Ensure No Country is Left Behind

Explanation:-Every year December 7 is observed as International Civil Aviation Day.The purpose of the annual celebration is to establish and reinforce worldwide awareness of the importance of international civil aviation in the social and economic development of States. The theme for International Civil Aviation Day this year is: "Working Together to Ensure No Country is Left Behind." This theme was established on a recurring basis for the 2016- 2018 ICAO triennium, in order to highlight the enormous transformational power of air connectivity to improve people's lives. The International Civil Aviation Day was established in 1994 as part of ICAO's 50th anniversary activities. In 1944, participants from 54 nations gathered in Chicago, US had signed Convention on International Civil Aviation, also known more popularly as the 'Chicago Convention'. Since then, the defining international agreement has permitted global civil aviation system to develop peacefully, in manner benefitting all nations across the world.

4)NASA's 'OSIRIS-REx' spacecraft's mission involves a close-up survey of the asteroid ______.

SSCE 8981426494/8296260082 Downloaded from - www.onlinessce.com Page 41

Ans:-Bennu

Explnation:-NASA's 'Origins, Spectral Interpretation, Resource Identification, Security- Regolith Explorer' (OSIRIS-REx) spacecraft arrived at the asteroid named 'Bennu' on 3rd December 2018. The spacecraft's mission involves a close-up survey of the asteroid Bennu. It is the part of NASA's New Frontiers Programme. 'OSIRIS-REx' was launched in September 2016.

5)The Cooperative department of Punjab decided to merge ______District Central Cooperative Banks (DCCBs) with the Punjab State Cooperative Bank (PSCB).

Ans:-20

Explanation:-The Cooperative department of Punjab has decided to merge 20 District Central Cooperative Banks (DCCBs) with the Punjab State Cooperative Bank (PSCB). The aim of the merger is to strengthen the state's rural credit system and facilitate farmers taking credit from cooperative banks. The 20 DCCBs were having 804 branches across the state and were facing a loss of Rs 700 crore. With the merger, the DCCBs would now be able to use the branches of the PSCB and function like other commercial banks. The decision was taken in view of RBI guidelines, which mandate that all DCCBs should have a Capital to Risk Weighted Assets Ratio (CRAR) of 9 per cent. At present the DCCBs are adhering to the required CRAR of 9 per cent, but there was hardly any space for majority of these DCCBs to enhance their business and to increase their profitability.

6)According to a study by Global Carbon Project, India ranks ______highest emitter of carbon dioxide in the world.

Ans:-4th

Explanation:-India is the fourth highest emitter of carbon dioxide in the world, accounting for 7 percent of global emissions in 2017, according to a study by Global Carbon Project on 06th December 2018. The top 10 emitters were China, the US, the EU, India, Russia, Japan, Germany, Iran, Saudi Arabia and South Korea. India's emissions look set to continue their strong growth by an average of 6.3 per cent in 2018, with growth across all fuels including coal (7.1 percent), oil (2.9 percent) and gas (6 percent). The Indian emissions were projected to grow 2 percent in 2017, compared to 6 per cent per year averaged over the previous decade, due to significant government interventions in the economy.

7)Arjuna Award holder Arun Kumar shaw passed away in Kolkata. He belongs to which sports?

Ans:-Swimming

Explanation:-80 years old seven time National Champion and first swimmer to win Arjuna Award from Bengal Arun Kumar shaw passed away following a prolonged illness in

SSCE 8981426494/8296260082 Downloaded from - www.onlinessce.com Page 42

Kolkata. He won National championship in 1959, 1962, 1964, 1965-67 and worked as National selector for several years.

8)This bank extended a credit facility of USD 500 million to Tanzania for water supply projects in the country

Ans:-Exim Bank

Explanation:-Export-Import Bank of India (Exim Bank) extended a credit facility of USD 500 million to Tanzania for water supply projects in the country. Exim Bank signed an agreement in May 2018, with the Tanzania government for making available a Government of India-supported line of credit of USD 500 million for the purpose of financing water supply schemes.

9)A Currency Swap Agreement was signed between Central Bank of the UAE and this Indian bank.

Ans:-RBI

Explanation:-Currency Swap Agreement was signed between the Reserve Bank of India and the Central Bank of the UAE. The MoU was signed by Ambassador of India to UAE, Shri Navdeep Suri and Dy. Governor of Central Bank of UAE.

10) Which of the following District Court becomes first in India to accept e-payments?

Ans:-Pune District Court

Explanation:-After the approval of the Supreme Court and the Bombay High Court, a pilot project for e-payments of court fee, judicial deposits, fines and penalties is slated to start from the Pune District Court complex in Shivajinagar on December 15. This will be the first time such a facility is being started anywhere in the country. As per the court records, every month about Rs 32 lakh is collected for fees less than Rs 2,000 and transactions above Rs 2,000 amount to Rs 11 crore.

11)Central Government has approved Anti-Smuggling National Coordination Centre (SCord) under which directorate to tackle Smuggling.

Ans:-Directorate of Revenue Intelligence (DRI)

Explanation:-The Central Government has approved Anti-Smuggling National Coordination Centre (SCord) under the Directorate of Revenue Intelligence (DRI) to tackle Smuggling. Scord will help India to formulate a national anti-smuggling policy to tackle Smuggling on real time basis. Border Agencies BSF, Assam Rifles, ITBP, SSB and Coast Guards would be member of SCord and all organization be coordinate with each other on real time basis.

SSCE 8981426494/8296260082 Downloaded from - www.onlinessce.com Page 43

12)Which Indian state will host AdventureNext 18?

Ans:- Madhya Pradesh

Explanation:-The main focus on adventure tourism by combining marketplace (chowk Bazar), meetings, networking opportunities, inspirational speakers and educational sessions together for national attendees as well as delegations from aboard. The successful convention was witnessed by nearly 200 attendees including global buyers, international/national media representatives, sellers and office bearers of the Adventure Tour Operators Association and hotels, hospitality and tour operators who came to expand their knowledge spectrum and itinerary offerings. The theme of the event is 'Pulse of Tomorrow' and Madhya Pradesh is the first in Asia to host 'Adventure Next'.

13)Who was appointed as the new government nominee on private sector ICICI Bank?

Ans:-Lalit Kumar

Explanation:-The government appointed Lalit Kumar, a 1995-batch Indian Economic Service (IES) officer as the new government nominee on private sector ICICI Bank. He is currently an Economic Advisor in the department of financial services of the finance ministry. He would replace Lok Ranjan, who has been laterally transferred to Department of Personnel and Training (DOPT). Lalit Kumar had also worked with the insurance regular (Insurance Regulatory Development Authority of India) as an executive director and financial adviser.

14)Atul Sahai will be appointed as Chairman and Managing Director of which of the following company?

Ans:-New India Assurance

Explanation:-The government announced that Atul Sahai will be appointed as Chairman and Managing Director (CMD) of New India Assurance (NIA) for a five year period replacing G Srinivasan. Atul Sahai is currently the General-Manager at Oriental Insurance Company Pvt.ltd. New India Assurance country's largest general insurer posted a 56 percent year-on-year (YoY) drop in its net profit at Rs 3.28 billion for the September 2019 quarter. Sahai had joined NIA in 1984 and would retire in 2022. He had moved to OIC in 2012 after getting promoted as a DGM.

15)Name of the CM who approved 50-crore rupees to Science & Technology Innovation (STI) Fund for assistance in research and development.

Ans:-Vijay Rupani

Explanation:-Gujarat Chief Minister Vijay Rupani approved 50-crore rupees to Science & Technology Innovation (STI) Fund for assistance in research and development. Thus became the first state in the country to create such a fund for high-end technology and innovation.

SSCE 8981426494/8296260082 Downloaded from - www.onlinessce.com Page 44

The assistance will be for a maximum period of three years. Assistance up to Rs. 50-lakh will be provided to UGC and AICTE recognized central and state universities and colleges.

Daily Current Affairs 10thDec,2018

1)International Anti-Corruption Day celebrated on which date?

Ans:-December 9th

Explanation:-Sixteenth International Anti-Corruption Day celebrated on December 9, 2018. The day has been celebrated since the passage of the United Nations Convention Against Corruption on 31 October 2003 to raise public awareness for anti-corruption. UN along with the United Nations Development Programme (UNDP) and United Nations Office on Drugs and Crime (UNODC) organized a global campaign called- "Corruption: An impediment to Sustainable Development Goals"- for spreading the awareness on corruption.

2)Union Cabinet approved a Memorandum of Understanding (MoU) between India and the United States of America (USA) focused on Scientific and Technical Cooperation in which of the following field?

Ans:-Earth Sciences

Explanation:-The Union Cabinet chaired by Prime Minister Shri Narendra Modi approved Memorandum of Understanding (MoU) between India and the United States of America (USA). The MOU focuses on Scientific and Technical Cooperation in the Earth Sciences. The MoU was signed on 1st November 2018.

3)Human Rights Day is observed on ______.

Ans:-10th December

Explanation:-Human Rights Day is observed every year on 10 December. On this day, the United Nations General Assembly adopted, in 1948, the Universal Declaration of Human Rights. This year, Human Rights Day marks the 70th anniversary of the Universal Declaration of Human Rights. #StandUp4HumanRights is the global campaign launched on for year 2018.

4)Who released the Compilation of Selected speeches of the President, Ram Nath Kovind named The Republican Ethic in New Delhi?

Ans:-Shri Venkaiah Naidu

Explanation:-The Vice President of India Shri M.Venkaiah Naidu released Compilation of Selected speeches of the President, Ram Nath Kovind titled, "The Republican Ethic" in English and "Loktantra ke Swar" in Hindi at a function in Vigyan Bhawan in New Delhi. Both the books- The Republican Ethic & Loktantra ke Swar are a collection of President of

SSCE 8981426494/8296260082 Downloaded from - www.onlinessce.com Page 45

India Shri Ram Nath Kovind's selected speeches made by him in the first year of assuming the office. The event has been organized by the Ministry of Information & Broadcasting.

5)Global Investors Meet (GIM) 2019 wil be hosted by which city?

Ans:-Chennai, Tamil Nadu

Explanation:-The Global Investors Meet (GIM) 2019 is to be held in Chennai, India on January 23 and 24, 2019. The officials of the Airports Authority of India (AAI) have planned to spruce up the Chennai airport to give a facelift to a host of facilities. AAI has set up separate counters for e-visas, foreign passports and passengers having Overseas Citizen of India (OCI) passports. A special parking area will be created for the those coming to attend the GIM 2019. It has also decided to deploy additional manpower and put up some facilitation counters to help those arriving at the airport. Even when the first ever Global Investors Meet happened in 2015, the airport was revamped.

6)Who signed USD 100 mn agreement with UNFCC approved Green Climate Fund to boost USD 250 mn worth solar power project?

Ans:-NABARD

Explanation:-The National Bank for Agriculture and Rural Development (NABARD) signed a loan agreement worth USD 100 mn with Green Climate Fund (GCF) to boost rooftop solar power capacity projects through the private sector participation. The Total corpus of the project is USD 250 mn, which will be executed by Tata Cleantech Capital Ltd. The Agreement was signed, on the sidelines of ongoing COP24 in Katowice, Poland, by Shri Shankar A Pande, Chief General Manager, NABARD in the presence of Shri A K Mehta, Additional Secretary, Ministry of Environment, Forests and Climate Change (MoEFCC).

7)Name the state which is the only state in the country with four international airports.

Ans:-Kerala

Explanation:-The Kannur international Airport-KIAL in Kerala began its operations with Union minister for civil aviation Suresh Prabhu and Kerala Chief minister Pinarayi Vijayan jointly lighting the lamp inside the terminal building, marking the inauguration of the airport. With this, Kerala became the only state in the country with four international airports.

8)Who was appointed as the Economic Advisor, recently?

Ans:-Dr. Krishnamurthy Subramanian

Explanation:-The government appointed Dr. Krishnamurthy Subramanian as the new Chief Economic Advisor. Dr. Subramanian will have a tenure of three years. The Appointments Committee of the Cabinet has cleared his name. He is currently working as an Associate Professor at the Indian School of Business, Hyderabad. A Ph.D. from the University of

SSCE 8981426494/8296260082 Downloaded from - www.onlinessce.com Page 46

Chicago Booth School of Business and a top-ranking IIT-IIM alumnus, Subramanian is one of the world's leading experts in banking, corporate governance and economic policy.

9)Which Indian city tops in the list fastest growing cities in the world from 2019 to 2035?

Ans:-Surat, Gujarat

Explanation:- According to Oxford Economics report, the top 10 fastest growing cities in the world from 2019 to 2035 are in India. Gujarat's Surat tops the list followed by Uttar Pradesh's Agra. Surat, a diamond processing and trading center in the western state of Gujarat, will see the fastest expansion through 2035, averaging more than 9 percent. Agra and Bangalore gets 2nd and 3rd spot respectively. By 2035, the GDP of Asian countries will be 17 percent higher than all North American and European urban centers combined. Mumbai was termed as the world's 12th richest city with a total wealth of $950 billion and India is the 6th largest wealth market in the world (in terms of total wealth held) after USA, China, Japan, Germany and the UK.

10)For which state, Shalindra Kumar was appointed as the Chief Electoral Officer?

Ans:-Jammu and Kashmir

Explanation:-Shalindra Kumar will take over as the Chief Electoral Officer of Jammu and Kashmir on 12th December 2018. He will be replacing the incumbent Shaleen Kabra. His office will take effect a day after the completion of the ongoing nine-phased panchayat polls in the State. The panchayat elections in J&K is being held after a gap of seven years. The elections in the State began on November 17 and will culminate on December 11.

11)Where was the Global Venture Capital Summit held?

Ans:-Goa

Explanation:-The Department of Industrial Policy and Promotion (DIPP) in the Ministry of Commerce and Industry and the Government of Goa hosted the annual #startupindia Global Venture Capital Summit in Goa. The theme of the Summit was: 'Mobilizing Global Capital for Innovation in India.' The International Finance Corporation (IFC), a sister organisation of the World Bank Group, Invest India and the Indian Private Equity and Venture Capital Association (IVCA) were the partners for this initiative. Participation came from around 100 funds from countries like United States of America, China, Japan, Hong Kong and Singapore. Government of Goa through its participation aims to make the state, one of the most preferred start-up destinations in India, and one of the top25 start-up destinations in Asia by 2025.

12)Which of the country is the world's top recipient of remittances in 2018?

Ans:-India

SSCE 8981426494/8296260082 Downloaded from - www.onlinessce.com Page 47

Explanation:-India will retain its position as the world's top recipient of remittances in 2018 with its diaspora sending a whopping USD 80 billion back home, according to the World Bank report. India is followed by China (USD 67 billion), Mexico and the Philippines (USD 34 billion each) and Egypt (USD 26 billion), according to the global lender. The Bank estimates that officially-recorded remittances to developing countries will increase by 10.8% to reach USD 528 billion in 2018.

13)Which country's government has recently enacted controversial law to accept foreign blue-collar workers?

Ans:-Japan

Explanation:-The Japanese government enacted a new law to bring more blue-collar foreign workers into the country, in a controversial move to address chronic labour shortages. Under the new law, Japan plans to bring in 345,000 foreign workers in construction, food services, nursing and other sectors for 5 years. The law will take effect in April 2019 and create two new visa categories. The first status allows foreign nationals with skills in sectors facing severe shortages to obtain five-year visas, which will not allow them to bring their families. The second status allows foreign workers in those fields who hold stronger qualifications and pass a more difficult Japanese language test will be able to obtain a visa that can be extended indefinitely, eventually leading to residency, and will be able to bring over family.

14)The 10th Edition of the bilateral exercise between the Indian Navy and Russian Federation Navy begins in ______.

Ans:-Visakhapatnam

Explanation:-The 10th Edition of the bilateral exercise between the Indian Navy and Russian Federation Navy (RuFN) started at Visakhapatnam. The Maritime exercise will conclude on December 16. The Primary aim of the exercise is to increase interoperability amongst the two navies, develop common understanding and procedures for maritime security operations. This Year's edition of exercise would be held in two phases, The Harbour phase would be held at Visakhapatnam from 9-12 December and the Sea phase would be held from 13-16 December in the bay of Bengal. Anti-Submarine Warfare (ASW), Air Defence Drills, Surface firings, Visit Board Search and Seizure (VBSS) operations and tactical procedures will be the thrust of exercises at sea.

15)Which of is the India's Second Inland Water Transport origin destination pair for containerized cargo movement on National Waterway-1?

Ans:-Kolkata-Patna

Explanation:-After the Success of container cargo being shipped form Kolkata to Varanasi, Kolkata-Patna has become India's Second Inland Water Transport origin destination pair for containerized cargo movement on National Waterway-1. Bihar Capital Patna will witness to a new landmark in India's Inland Water Transport (IWT) sector with 16 TEUs of Container

SSCE 8981426494/8296260082 Downloaded from - www.onlinessce.com Page 48

Cargo from food giants PepsiCo India and Emami Agrotech Ltd reaching Gaighat IWT terminal from Kolkata on river Ganga.

16)Name the Union Minister who resigned recently?

Ans:-Upendra Kushwaha

Explanation:-Rashtriya Lok Samta Party (RLSP) Chief Upendra Kushwaha resigned as Union Minister of State in the Ministry of Human Resource Development. This comes a day ahead of the start of the Parliament's Winter Session, triggering a realignment of political equations in Bihar.

17)Vanessa Ponce de Leon won the Miss World 2018 crown. She belongs to the country of;

Ans:-Mexico

Explanation:-Mexico's Vanessa Ponce de Leon won the coveted Miss World 2018 crown at a grand event held in the Chinese city, Sanya. Thailand's Nicolene Pichapa Limsnukan was declared the runner-up. Former Miss World Manushi Chillar (India) presented the crown to the new winner during the pageant in which 118 contestants participated. India's Anukreethy Vas failed to make a mark at the pageant as she could not even make it to the top 30.

18)India's first underwater museum will be come up in ______.

Ans:-Puducherry

Explanation:-Indian Navy agreed to gift INS Cuddalore, a decommissioned Pondicherry- class vessel, to the Union territory of Puducherry for the creation of country's first underwater museum. The 60 m long and 12 m wide vessel was decommissioned in March 2018 after sailing 13000 nautical miles. It would be sent to the seabed at a depth of 26 m and 7 km off the Pondicherry coast to act as an underwater marine museum for scuba diving and snorkeling activities. This would enhance tourism in the UT.

Daily Current Affairs 11thDec,2018

1)This state government plans to introduce a new software to curb cheating in board exams.

Ans:-Uttar Pradesh

Explanation:-The Uttar Pradesh State government has designed a new software and a four- step process initiated to check the menace of copying in the Uttar Pradesh Board exams. The aim is to curtail the use of unfair means by students while making efforts to improve the quality of education. This process will begin from 7th February 2019. Exam centres in the State, which reported maximum cases of cheating in the past, will be strictly monitored.

SSCE 8981426494/8296260082 Downloaded from - www.onlinessce.com Page 49

Owing to the efforts of the U.P. government, the use of unfair means has stopped, and the number of students studying in government schools has increased by 18-20%.

2)2018 Military Literature Festival was held in ______.

Ans:-Chandigarh

Explanation:-The 3-day Military Literature Festival (MLF) 2018, commenced at Lake Club in Chandigarh. The festival is a joint initiative of Punjab Chief Minister Captain Amarinder Sing, Punjab Governor VP Singh Badnore and The Western Command, Indian Army. First session on 'Role of Cross Border Operations & Surgical Strikes', Second session on the 'Contribution of India towards the First World War', Third session was on sessions 'Two Battles For Survival - Ferozshah 1848 and Chilianwala 1849', and 'Indian Cavalry Charges of the First World War and the Gallipoli Campaign'.

3)Who of the following stepped down from the post of Reserve Bank of India?

Ans:-Urjit Patel

Explanation:-Based on the results of polling in the five states, Reserve Bank of India governor Urjit Patel stepped down from his post. Patel, who cited "personal reasons" behind his decision, was in a conflict with the Union finance ministry over matters related to the monetary policy.

4)For which state, Asian Infrastructure Investment Bank (AIIB) had approved a $400 million loan for a Water Sanitation Project?

Ans:-Andhra Pradesh

Explanation:-The Beijing Headquartered Asian Infrastructure Investment Bank (AIIB) had approved a $400 million loan for a Water Sanitation Project in Andhra Pradesh. The Project will be very helpful for women and girls of affected community in Andhra Pradesh and it will reduce the health risks and health expenditure for the people of Andhra Pradesh. India is the second largest shareholder of Asian Infrastructure Development Bank after China. After this Loan the overall loan amount of India from AIIB reached to $2 Billion spread over nine projects. This loan will strengthen the AIIB's capacity and loan demand from other states is also expected to increase.

5) The first International Conference on 'Sustainable Water Management' was held at;

Ans:-Mohali

Explanation:-The first International Conference on 'Sustainable Water Management' was held at Indian School of Business (ISB) in Mohali, Punjab. The conference was organized by Bhakra Beas Management Board (BBMB) under the aegis of the National Hydrology Project of the Union Ministry of Water Resources, River Development and Ganga Rejuvenation.

SSCE 8981426494/8296260082 Downloaded from - www.onlinessce.com Page 50

The theme of the international conference is 'Sustainable Water Management'. The inaugural session of the conference was attended by Himachal Pradesh Governor Acharya Devvrat, who was the chief guest of the event and UP Singh, Secretary at the Union Ministry of Water Resources, who was the guest of honour.

6)Which IIT institute won DSCI Excellence Award 2018 for its contributions inCyber Security education?

Ans:-IIT Kharagpur

Explanation:-IIT Kharagpur (KGP) won DSCI Excellence Award 2018, given by Data Security Council of India (DSCI), for its contributions in Cyber Security education. It was awarded for its focused curricula and research work in cryptography, hardware security, cyber security, network security which have helped spread awareness among the student. Some of the notable ventures of the department of computer science and engineering at IIT Kharagpur are The Defence Research and Development Organisation (DRDO)-funded projects on developing lightweight protocols for authenticating IoT nodes, IC reverse engineering and trojan detection and the DST-funded 'Swarnajayanti' project on secure authentication protocol for smart grids.

7)The first International Conference on 'Sustainable Water Management' was held at;

Ans:-Mohali

Explanation:-The first International Conference on 'Sustainable Water Management' was held at Indian School of Business (ISB) in Mohali, Punjab. The conference was organized by Bhakra Beas Management Board (BBMB) under the aegis of the National Hydrology Project of the Union Ministry of Water Resources, River Development and Ganga Rejuvenation. The theme of the international conference is 'Sustainable Water Management'. The inaugural session of the conference was attended by Himachal Pradesh Governor Acharya Devvrat, who was the chief guest of the event and UP Singh, Secretary at the Union Ministry of Water Resources, who was the guest of honour.

8)Who is appointed as the Chief Business Officer of Walmart India?

Ans:-Sameer Aggarwal

Explanation:-Walmart India, the U.S. retail major, appointed Sameer Aggarwal as its Chief Business Officer. Mr.Sameer has been elevated from his position as EVP, chief strategy and administrative officer to the Chief Business Officer with immediate effect. Aggarwal will now lead strategy, merchandising, cost analytics, e-commerce, technology, marketing, and replenishment. Walmart India also appointed Anuj Singh as Head-Category Merchandising. The announcement followed after the resignation of Executive Vice-President (EVP) and Chief Operating Officer (COO) Devendra Chawla. Mr.Chawla had joined Walmart India in July last year from Future Consumer, a part of Kishore Biyani-led Future Group.

SSCE 8981426494/8296260082 Downloaded from - www.onlinessce.com Page 51

9)India Successfully Test-Fires Inter-Continental Ballistic Missile Agni-5 from which of the space station?

Ans:-Abdul Kalam Island

Explanation:-The indigenously built Inter-Continental Ballistic Missile Agni-5 has been test-fired successfully from Abdul Kalam island under Chandipur Interim Test Range. It can carry both nuclear and traditional weapons. The first test was conducted in 2012 from ITR Chandipur in Balasore district. This is the fourth successful test of this missile. Previously the test of Agni 4 was unsuccessful. Having 17 meters length and 2 meters diameter the missile can hit the target from a distance of 5 thousand kilometers.

10)Smartworks a leading workspaces player, introduced a female AI robot in productivity mascot. It was named as;

Ans:-Smart Mitri

Explanation:-Smartworks, a leading workspaces player, introduced 'Smart Mitri', a female AI robot, which will be Smartworks' in-house productivity mascot. It will play an influential role in handling the overall office management for the facility, ranging from the visitor system to being a counsellor eventually. Smartworks recently launched its flagship facility in Bengaluru. It is spread over 3 lakh sq ft with 6,000 seats. This facility is the firm's largest centre. With 20 locations across nine cities, the firm caters to more than 500 organizations across large enterprises, SMEs and startups. Some of the clients are Tata Communications, Microsoft, ArcelorMittal, Amazon, Carrier, Otis, Daikin, Lenovo, Bacardi and OLX.

11). Who has become the youngest Indian golfer to win Asian Tour Order Of Merit?

Ans-Shubhankar Sharma

Explanation:-Shubhankar Sharma has become the fifth and the youngest Indian golfer to clinch the Asian Tour Order of Merit after a groundbreaking year of achievements. With this achievement, He joined the exclusive club of Indians who have achieved the feat in the past - Jyoti Randhawa (2002), Arjun Atwal (2003), Jeev Milkha Singh (2006 and 2008) and Anirban Lahiri (2015).

12)Who conducted an exercise "Cross Bow 18"?

Ans:-Indian Air Force

Explanation:-The Indian Air Force (IAF) conducted combined guided weapons firing of Surface-to-Air Missiles from the Air Force Station located at Suryalanka, Andhra Pradesh. The first-of-its-kind exercise for the Air Force was code-named 'CROSS BOW-18'. CROSS BOW-18 saw the successful firing of four different classes of missiles like Akash, Spyder, OSA-AK-M and IGLA.

SSCE 8981426494/8296260082 Downloaded from - www.onlinessce.com Page 52

13)Who has been honoured with this year's Sanctuary Wildlife Service Award?

Ans:-Imran Siddiqui

Explanation:-Hyderabad-based conservationist Imran Siddiqui has been honoured with this year's Sanctuary Wildlife Service Award, which is India's most prestigious award. Siddiqui, who is an Assistant Director for Conservation Science at WCS-India monitors tiger in 10,000 sq km, including Nagarjunasagar Srisailam, Amrabad and Kawal Tiger Reserves, and corridors connecting to Tadoba and Tirupati Forests.

14)International Mountain Day is observed on 11 December, The theme for 2018 IMD is;

Ans:-Mountains Matter

Explanation:-International Mountain Day is held every year on 11 December, which was established by the UN General Assembly in 2003 to encourage sustainable development in mountains. The theme for 2018 IMD is "Mountains Matter".

15)Jagdish Thakkar who passed away in New Delhi was a famous ______.

Ans:-Journalist

Explanation:-Public Relations Officer (PRO) in the prime minister's office and senior journalist Jagdish Thakkar passed away in New Delhi. Thakkar, 72, who was unwell for some time, passed away at the All India Institute of Medical Sciences.

16)Ananth Narayanan, who resigned his CEO post from which of the following firm?

Ans:-Myntra

Explanation:-Flipkart-owned Myntra's CEO Ananth Narayanan has resigned from his role with his position set to be abolished. Flipkart's Amar Nagaram has been given the responsibility to head Myntra. Key Myntra leaders such as Chief Revenue Officer Mithun Sundar and human resources head Manpreet Ratia have also resigned.

17)2019 Khelo India Youth Games will be held in ______.

Ans:-Pune

Explanation:-Khelo India Youth Games-2019 will be held in Pune of Maharashtra in January. Sports Minister Col Rajyavardhan Rathore announced that 9000 youth will participate in this edition of the game. Under Khelo India programme, the government is providing scholarships to 1500 students from across the country. Last year around 3500 school children had participated in the event. In this edition of Khelo India Youth Games university players will also take part.

SSCE 8981426494/8296260082 Downloaded from - www.onlinessce.com Page 53

18)Name of the first Indian who wins Mister Supranational title in Poland.

Ans:-Prathamesh Maulingkar

Explanation:-Goa's Prathamesh Maulingkar has created history by becoming the first Asian/Indian to win Mister Supranational title in a competition held at Krynica-Zdroj, Poland. The 27-year-old defeated 37 contestants from all over the world in the final round to win the coveted title. At the same pageant, Mister Supranational Poland was adjudged first runner-up, Mister Supranational Brazil was adjudged second runner-up and Mister Supranational Thailand was announced as the third runner-up.

19)Who was elected as the 14th chairman of the Coca-Cola Company?

Ans:-James Quincey

Explanation:-James Quincey was elected as the 14th chairman of the Coca-Cola Company, succeeding Muhtar Kent, who plans to retire in April after a Coca-Cola system career that started in 1978. Quincey, currently serves as president and CEO of the Coca-Cola Company and will become chairman following the company's annual meeting in April 2019. Brian Smith will become president and chief operating officer of the company on Jan. 1, 2019. The company also announced that Sam Nunn, the Coca-Cola director, will also retire and Maria Elena "Mel" Lagomasino, will succeed Nunn as an independent director.

20)Which state government launches 24-hour helpline 181 for women, recently?

Ans:-Tamil Nadu

Explanation:-Tamil Nadu Government launched a 24-hour toll-free helpline number 181 for women facing domestic violence and sexual harassment to get assistance ranging from police help, legal aid or medical services including an ambulance. The Service developed at a cost of 62.70 lakh will be available round-the-clock on all days of the week and women can also get information about the welfare schemes aimed at their benefit.

Daily Current Affairs 12thDec,2018

1)Where was the Conference on 'Insolvency and Bankruptcy Code - A New Paradigm for Stressed Assets held?

Ans:-New York, United States of America (USA)

Explanation:-A conference on the theme 'Insolvency and Bankruptcy Code of India - New Paradigm for Stressed Assets' was held at the Indian Consulate in New York. The conference was organised by the Insolvency and Bankruptcy Board of India (IBBI) jointly with the Consulate General of India, New York, USA. The Union Minister of Finance and Corporate Affairs, Mr. Arun Jaitley addressed the conference via Video Conferencing. Dr. M. S. Sahoo, Chairperson, IBBI also addressed at the function. The Conference was followed by a

SSCE 8981426494/8296260082 Downloaded from - www.onlinessce.com Page 54

roundtable with prospective stakeholders of the Indian insolvency regime which witnessed participation of many international fund houses and law firms.

2)ENSURE - National Livestock Mission was launched by the ministry of;

Ans:-Ministry of Agriculture

Explanation:-Union Minister of Agriculture and Farmers' Welfare Radha Mohan Singh launched a portal 'ENSURE'- National Livestock Mission-EDEG developed by NABARD and operated under the Department of Animal Husbandry, Dairying & Fisheries. The NABARD has developed an online portal "ENSURE" (ensure.nabard.org) so that the information related to beneficiary and processing of the application can be made readily available.

3)Who of the following has joined SoftBank as its first venture partner at its $100 billion Vision Fund?

Ans:-Kirthiga Reddy

Explanation:-Kirthiga Reddy, the former managing director of Facebook India, has joined SoftBank as its first venture partner at its $100 billion Vision Fund. She is the first women coming on board in a company where the rest are all men. Reddy was Facebook's first employee in India and served as the managing director for India and South Asia market until 2016, following which she relocated to the company's US headquarters, serving as Managing Global Client Partner and Emerging Markets Lead.

4)Name the Union Minister who launched a National Challenge for Youths, "Ideate for India - Creative Solutions using Technology", in New Delhi.

Ans:-Ravi Shankar Prasad

Explanation:-Union Minister for Electronics & IT, Law & Justice, Shri Ravi Shankar Prasad, launched a National Challenge for Youths,"Ideate for India - Creative Solutions using Technology", in New Delhi. The Challenge has been designed and launched by: The National e-Governance Division, Ministry of Electronics & IT in collaboration with Intel India, with support from the Department of School Education and Literacy (DoSE&L), Ministry of Human Resource Development. The DoSE&L will propagate the challenge to State Education Departments, NVS, KVS and CBSE authorities. The National Challenge will be open to at least 1 million students of classes 6 - 12 all across the country - all 29 States and 7 Union Territories over the next 3 months ending in March 2019.

5)Reserve Bank of India has imposed a fine of Rs 1 crore on this bank for violating cybersecurity norms.

Ans:-Indian Bank

SSCE 8981426494/8296260082 Downloaded from - www.onlinessce.com Page 55

Explanation:-The Reserve Bank of India has imposed a fine of Rs 1 crore on Indian Bank for violating cybersecurity norms. The contravention is with regard to RBI's directions on Frauds - Classification and Reporting by Commercial Banks.

6)Who presented the lifetime achievement award at the Trinity Arts Festival in Chennai?

Ans:-T T Srinivasaraghavan

Explanation:-Sundaram Finance Managing Director T T Srinivasaraghavan was presented the lifetime achievement award at the Trinity Arts Festival in Chennai. The award was presented by publisher and former Editor-in-Chief of The Hindu, and the Press Trust of India, Chairman, N Ravi. He was awarded for his leadership qualities that made Sundaram Finance as one of India's most trusted non-banking finance companies. Over the last 13 years, he has been organising mikeless kutcheries for children aged below 15 years which got him an entry into the Limca Book of Records for the longest open air kutcheri in the country.

7)Who became the youngest and the fifth Indian to win the Asian Tour Order of Merit in golf?

Ans:-Shubhankar Sharma

Explanation:-Shubhankar Sharma became the youngest and the fifth Indian to win Asian Tour Order of Merit in golf. The Indian won it even before the finish of the last two events of the year. Before Sharma, Jyoti Randhawa (2002), Arjun Atwal (2003), Jeev Milkha Singh (2006 and 2008) and Anirban Lahiri (2015) had achieved the feat. With the win, Shubhankar Sharma also captured the Asian Tour Habitat for Humanity Standings title after a ground breaking year of achievements. The 22 years old Sharma's Order of Merit win was confirmed after he managed to secure total prize money of $755,994 on the Asian Tour for 2018. He will be officially crowned as Asian Tour's No. 1 during the BNI Indonesia Masters competition scheduled next week in Jakarta, Indonesia. Shubhankar Sharma was also honoured with the Arjuna Award 2018, which was presented to him by the President of India.

8)Which country become a permanent member of Financial Action Task Force?

Ans:-Israel

Explanation:-Israel became a full member of the Financial Action Task Force (FATF), an international body set up to combat money laundering, terrorist financing and other threats to the international financial system. The Jewish state has now taken its place alongside 37 other members including most of the G20, the world's 20 leading industrialized and emerging economies just 16 years after being blacklisted by the organization.

SSCE 8981426494/8296260082 Downloaded from - www.onlinessce.com Page 56

9)According to the Yahoo Year in Review list, Who has retained the position of top newsmaker in India in the year 2018?

Ans:-Narendra Modi

Explanation:-Prime Minister Narendra Modi was once again the top newsmaker in India in the year 2018, according to the Yahoo Year in Review list. Modi has featured at the top of the list for at least a couple of years now and is followed this year by Congress president Rahul Gandhi. Former Chief Justice of India Dipak Misra wrote his way into the list with third position after landmark verdicts on triple talaq and Article 377.

10)Who was the first-ever recipient of the Women's Tennis Association (WTA) Coach of the Year Award?

Ans:-Sascha Bajin

Explanation:-Sascha Bajin, coach of the world number five Naomi Osaka, has become the first-ever recipient of the Women's Tennis Association (WTA) Coach of the Year Award. WTA Coach of the Year Award is given to the coach with not only success on-court, but also who serves as an ambassador of the sport and brings coaching to the forefront of the game.

11)Who was appointed as additional director on the board of Axis Bank?

Ans:-Amitabh Chaudhry

Explanation:-Axis Bank inducted Amitabh Chaudhry as additional director on its board. He will replace incumbent Shikha Sharma whose fourth term has been curtailed by the RBI by nearly two-and-half years. The board also gave its nod for the reappointment of Samir Barua, Som Mittal and Rohit Bhagat as independent directors. Amitabh Chaudhry, the former MD and CEO of HDFC Standard Life Insurance Company, was in September named the MD and CEO of Axis Bank for a period of three years, with effect from January 1, 2019.

12)Who was the new governor of RBI?

Ans:-Shaktikanta Das

Explanation:-The government has appointed Shaktikanta Das as the new RBI governor, a day after Urjit Patel resigned as RBI Governor. He is the 25th Governor of the Reserve Bank of India. A former economic affairs secretary from 2015 to 2017, Das worked closely with the central bank. He is currently a member of the Finance Commission of India, and the government's representative at the Group of 20 summits.

13)India's first Jean Monnet Centre of Excellence was opened in ______.

Ans:-New Delhi

SSCE 8981426494/8296260082 Downloaded from - www.onlinessce.com Page 57

Explanation:-India's first Jean Monnet Centre of Excellence established by the European Union was inaugurated at the Department of European Studies (DES) at Manipal Academy of Higher Education in New Delhi by Ambassador of the European Union (EU) to India and Bhutan, Tomasz Kozlowski. The award comes with a grant of 1,00,000 Euros to pursue India-EU interdisciplinary studies in culture, literature, education and society. This is the fifth time that MAHE has been awarded grants under the Erasmus+ Jean Monnet initiative.

14)Whom has been honoured with the title "Meethoileima" by the Manipur Government?

Ans:-

Explanation:-Manipur Government honored AIBA World Women Boxing Gold medalist MC Mary Kom with the title "Meethoileima" (par excellent queen) at a function in Imphal, Manipur. On the occasion, State Chief Minister N. Biren Singh handed over a cheque of ten lakh rupees to her. The Chief Minister also declared that the Imphal West DC Road towards Game Village will be named as MC Mary Kom Road.

15)Asian Development Bank signed an Loan Agreement for $31 million to build-up the State Tourism Industry of this state recently.

Ans:-Tamil Nadu

Explanation:-The Government of India and the Asian Development Bank (ADB) signed in New Delhi a Loan Agreement for $31 million to build-up the State Tourism Industry and boost visitor arrivals in Tamil Nadu. The signatories to the tranche 4 loan for Infrastructure Development Investment Program for Tourism (IDIPT) were Mr. Sameer Kumar Khare, Additional Secretary (Fund Bank and ADB), Department of Economic Affairs, Ministry of Finance, who signed for the Government of India, and Mr. Kenichi Yokoyama, Country Director for ADB in India, who signed for ADB.

16)Which country became the world's leading exporter of Liquified Natural Gas (LNG) for the first time?

Ans:-Australia

Explanation:-Australia surpassed Qatar to become the world's leading exporter of Liquified Natural Gas (LNG) for the first time. This was announced according to the Australian Bureau of Statistics which stated that Australia had exported 6.55 million tonnes of LNG in November 2018 compared to Qatar's 6.27 million tonnes. LNG is Australia's third-largest export commodity behind coal and ore.

17)Scientists from which country developed a "rewritable" paper?

Ans:-China

SSCE 8981426494/8296260082 Downloaded from - www.onlinessce.com Page 58

Explanation:-Scientists from the Fujian Normal University in China developed a "rewritable" paper that can be drawn or printed on over and over again by changing the temperature. The new material consisted of three layers in a sandwich-like structure where the text could be changed or wiped clean over 100 times by changing the temperature. According to a study published in the journal ACS Applied Material & Interfaces, the messages can last more than half a year, compared to other re-writable papers. It can be used in practical applications such as long-lasting information recording and reading, rewritable label, reprintable display.

Daily Current Affairs 13thDec,2018

1) Intergovernmental Conference on the Global Compact for Migration for year 2018 was held in;

Ans:-Marrakech

Explanation:-The Intergovernmental Conference to Adopt the Global Compact for Safe, Orderly and Regular Migration was held in Marrakech, Morocco on 10th and 11th of December, 2018. It is the first-ever UN global agreement on a common approach to international migration in all its dimensions. The global compact is non-legally binding. It is grounded in values of state sovereignty, responsibility-sharing, non-discrimination, and human rights, and recognizes that a cooperative approach is needed to optimize the overall benefits of migration, while addressing its risks and challenges for individuals and communities in countries of origin, transit and destination.

2)Name of the Actress who conferred with the tittle named India's Hottest Vegetarians by PETA.

Ans:-Anushka Sharma

Explanation:-Actors Anushka Sharma and Kartik Aaryan have been named India's Hottest Vegetarians, according to votes on the official website of People for Ethical Treatment of Animals (PETA) India. Anushka Sharma is the only woman to bag this title twice. She previously won in 2015. Rajkummar Rao, Alia Bhatt were named India's Hottest Vegetarians by PETA last year.

3)Which Twitter handle has been launched by the Union Government to spread awareness about cyber crimes?

Ans:-@CyberDost

Explanation:-The Union Ministry of Home Affairs has launched a Twitter account to spread awareness about cyber crimes and normal precautions to be taken. The Twitter handles - @CyberDost - is aimed at enhancing people's basic knowledge about cyber crimes and precautions to be taken for prevention.

SSCE 8981426494/8296260082 Downloaded from - www.onlinessce.com Page 59

4)Who was awarded the International Table Tennis Federation's (ITTF) Breakthrough Star Award 2018?

Ans:-Manika Batra

Explanation:-Manika Batra, India's star paddler, was awarded the International Table Tennis Federation's (ITTF) Breakthrough Star Award 2018 during a ceremony at the Grand Hyatt Hotel. In the Commonwealth Games 2018 at Gold Coast, Manika won four medals including a historic women's singles gold, team gold, women's doubles silver (with Mouma Das) and mixed doubles bronze (with G.Sathiyan). She had also bagged the bronze in mixed doubles (with A. Sharath Kamal) at the 2018 Asian Games.

5)Which country signed the 30-member Trans Regional Maritime Network pact in Rome, Italy?

Ans:-India

Explanation:-India signed an ascension pact to the 30-member Trans-Regional Maritime Network (T-RMN) at Italian Naval Headquarters in Rome, Italy. The T-RMN consists of 30- member countries and is led by Italy.

6)The Indian Navy launched its first non-tethered Deep Submergence Rescue Vehicle system in ______.

Ans:-Mumbai

Explanation:- The Indian Navy launched its first non-tethered Deep Submergence Rescue Vehicle (DSRV) system at the Naval Dockyard in Mumbai. The DSRV is used to rescue crew members from submarines stranded underwater in the high seas. The cost of one system is Rs.1,000 crore. In March 2016, the Indian Navy had signed a Rs.2,000 crore contract with the U.K.-based James Fisher Defence (JFD) for two submarine rescue systems as well as maintenance for 25 years. Admiral Lanba said the second system would be inducted in April and would be based at the Eastern Naval Command in Vishakapatnam. Deep Submergence Rescue Vehicle (DSRV) system. It can rescue 14 people at a time. It is also equipped with a decompression chamber that can accommodate submariners and a Remotely Operated Vehicle (ROV), which can be used to beam images and provide immediate assistance.

7)Indo-China joint military exercise 'Hand-in-Hand' has started in ______.

Ans:-Chengdu

Explanation:-The armies of India and China has started the 7th edition of joint military exercise 'Hand-in-Hand 2018' from Dec 11-23 at Chengdu in China. The purpose of the joint military drill is to improve the capabilities of the two armies in fighting terrorism and promote mutual understanding. Both sides have fielded 100 personnel to take part in the

SSCE 8981426494/8296260082 Downloaded from - www.onlinessce.com Page 60

exercise that focusses on tactical level operations in an International Counter Insurgency/Counter Terrorist environment as per UN mandate. The drills are being held after a gap of one year as they could not be held in 2017 because both armies were locked in a 73- day standoff at Doklam in the Sikkim sector.

8)Name the Indian personality who has been appointed as Vice-Chair of UN Panel of Auditors for 2019.

Ans:-Rajiv Mehrishi

Explanation:-Rajiv Mehrishi, the Comptroller and Auditor General (CAG) of India, has been appointed as Vice-Chair of UN Panel of Auditors for the year 2019. The United Nations Panel of Auditors consists of External Auditors of the United Nations and its agencies. Presently, the panel consists of 11 countries -- India, Germany, Chile, Canada, France, Italy, Philippines, Ghana, Indonesia, and United Kingdom. Currently, the panel is chaired by CAG of the UK. The panel held its annual meeting in New York from 3 to 4 December 2018 and discussed various issues concerning audit of United Nations and the agencies under the United Nations System.The panel also elected the CAG of the UK as Chair of the Panel for another term (2019). The next meeting of the panel will be held in in Bonn, Germany in November-December 2019.

9)Who resigned from Prime Minister Narendra Modi's Economic Advisory Council (PMEAC)?

Ans:-Surjit Bhalla

Explanation:-Indian economist Surjit Bhalla announced his resigned from Prime Minister Narendra Modi's Economic Advisory Council (PMEAC). His resignation is effective from December 1, 2018. After the resignation, he joined the CNN IBN group. Also he released a book called Citizen Raj: Indian Elections 1952-2019, focusing on Indian elections since 1952.

10)Russia sends 2 nuclear-capable bombers to which of the following country?

Ans:-Venezuela

Explanation:-Russia has sent two Tu-160, strategic bombers capable of carrying nuclear weapons, to Venezuela in an apparent show of support to President Nicolas Maduro's regime. Russia is a major political ally of Venezuela, which has become increasingly isolated in the world under growing sanctions led by the United States and the European Union, which accuse Maduro of undermining democratic institutions to hold onto power while overseeing an economic and political crisis that is worse than the Great Depression.

11)Who of the following has been appointed as the Chairman of SWIFT India?

Ans:-Arundhati Bhattacharya

SSCE 8981426494/8296260082 Downloaded from - www.onlinessce.com Page 61

Explanation:-Arundhati Bhattacharya, former chairman of State Bank of India (SBI) has been appointed as SWIFT India Chairman. She will succeed former banker M V Nair. SWIFT India is a joint venture of top Indian public and private sector banks and SWIFT (Society for Worldwide Interbank Financial Telecommunication).

12)This company collaborate with Oracle Marketing Cloud to empowers organizations to take a smarter approach to customer experience management.

Ans:-Route Mobile Limited

Explanation:-Route Mobile Limited (Route Mobile), a cloud-communication platform service provider and a member of Oracle Partner Network (OPN) announced the collaboration with Oracle Marketing Cloud, part of Oracle Customer Experience (CX) Cloud Suite, which empowers organizations to take a smarter approach to customer experience management and business transformation initiatives.

13)Who developed a Light Utility Helicopter which achieves important milestone of flying at 6 km altitude?

Ans:-HAL

Explanation:-Hindustan Aeronautics Limited (HAL) indigenously developed Light Utility Helicopter achieved important milestone of flying at 6 Km altitude in Bengaluru. It is 3-ton class new generation helicopter designed and developed by indigenously Rotary Wing Research and Design Center (RWRDC) of HAL. It will replace ageing fleet of Cheetah and Chetak helicopters used by Indian Armed Forces.

14)Which Indian power station created a world record for the longest uninterrupted operation for 941 days?

Ans:-Kaiga Power Station-1, Karnataka

Explanation:-Karnataka's Kaiga Power Station-1 has created a world record for the longest uninterrupted operation for 941 days, thereby breaking the earlier record of 940 days by the United Kingdom. Heysham-2 Unit-8 of the U.K. had held the earlier record of the longest uninterrupted operation (940 days) among all nuclear power reactors (of all technologies) in the world. Kaiga Generating Station (KGS-1) equalled the world record on Sunday and broke it on Monday, thus creating history. While KGS-1 is a Pressurised Heavy Water Reactor (PHWR), Heysham-2 Unit-8 is an Advanced Gas Cooled Reactor (AGR).

15)Kamal Nath was sworn as the Cheif Minister of this state recently.

Ans:-Madhya Pradesh

Explanation:-Congress party has chosen MP statue unit president Kamal Nath as the chief minister of Madhya Pradesh. He will replace outgoing Chief Minister Shivraj Singh

SSCE 8981426494/8296260082 Downloaded from - www.onlinessce.com Page 62

Chouhan. A one-line resolution has been passed at the Congress Legislature Party (CLP) meeting in Bhopal to choose the Madhya Pradesh chief minister. Congress party high command will now take the final decision.

16)Commonwealth Games gold medallist Krishna Poonia becomes an MLA of this state, recently.

Ans:-Rajasthan

Explanation:-Discus thrower Krishna Poonia, the 2010 CWG gold medallist and Congress candidate, won the Sadulpur assembly seat in Rajasthan, defeating sitting BSP MLA Manoj Nyangali. It was Poonia's second attempt to get entry in the state legislature, having suffered defeat in the previous elections.

Daily Current Affairs 14thDec,2018

1)Government of India and ADB signed ______Loan Agreement in New Delhi for Assam.

Ans:-60 Million

Explanation:-The Asian Development Bank (ADB) and the Government of India signed a $60 Million Loan Agreement in New Delhi to continue financing riverbank protection works, renovation of flood embankments, and community-based flood risk management activities in critically flood-prone areas along the Brahmaputra River in Assam. The Tranche 2 loan is part of the $120 million multi-tranche financing facility (MFF) for the Assam Integrated Flood and Riverbank Erosion Risk Management Investment Program approved by the ADB Board in October 2010.

2)Ashok Gehlot was sworn in as the Chief Minister of this state, recently.

Ans:-Rajasthan

Explanation:-The Congress party has picked old guard Ashok Gehlot over Sachin Pilot as Chief Minister of Rajasthan. Pilot will be the deputy CM of Rajasthan and will also continue as the state Congress chief. India's Grand Old Party won 99 seats of the 199 in the Rajasthan Assembly, just one short of the halfway mark to form the government.

3)Which team won the 115th Aga Khan Gold Cup tournament in New Delhi?

Ans:-SAIL Hockey Academy

Explanation:-Steel Authority of India's Hockey Academy wins 115th Aga Khan Gold Cup tournament by beating Army XI Danapur by 2-1 at Major Dhyan Chand hockey stadium in New Delhi. Jay Prakash Patel scored the match-winning goal for the SAIL Hockey Academy (SHA) in the final match of Aga Khan Gold Cup which is one of the oldest hockey

SSCE 8981426494/8296260082 Downloaded from - www.onlinessce.com Page 63

tournament in the country. The SHA team defeated Madhya Pradesh Hockey Academy by 4- 2 in the semifinals. SAIL Hockey Academy is run by SAIL's Rourkela Steel Plant.

4)Brahm Dutt was appointed as an Interim Chairman of this bank.

Ans:-Yes Bank

Explanation:-Yes Bank Ltd has named one of its independent directors Brahm Dutt as the bank's part-time non-executive independent chairman after former Finance Secretary Ashok Chawla resigned from the position. Brahm Dutt is a former bureaucrat and is one of the oldest members of Yes Bank's board.

5). With which country, India signed the bilateral annual Haj 2019 agreement?

Ans:-Saudi Arabia

Explanation:-India and Saudi Arabia signed the bilateral annual Haj 2019 agreement at Jeddah. The agreement was signed by the Union Minister for Minority Affairs, Mukhtar Abbas Naqvi and Haj and Umrah Minister of Saudi Arabia, Dr. Mohammad Saleh bin Taher Benten. The agreement is a part of strengthened bilateral relations of the two countries. Both India and Saudi Arabia share strong cultural, historical, economic and political relations.

6)What is the theme of 2018 International Universal Health Coverage Day?

Ans:-Unite for Universal Health Coverage: Now is the Time for Collective Action

Explanation:-International Universal Health Coverage Day (UHC Day) was observed throughout the world. It aims to mobilize diverse stakeholders to call for stronger, more equitable health systems to achieve universal health coverage, leaving no one behind. The theme for the 2018 UHC Day is: "Unite for Universal Health Coverage: Now is the Time for Collective Action". Universal health coverage (UHC) ensures all people, everywhere, can access the quality health services they need without suffering financial hardship. The United Nations proclaimed 12 December as International Universal Health Coverage Day (UHC Day) by resolution 72/138 in 2017.

7)Who was appointed as the new president of the Film and Television Institute of India ?

Ans:-Brijendra Pal Singh

Explanation:-Producer-director Brijendra Pal Singh was appointed as the new president of the Film and Television Institute of India (FTII) and chairman of the institute's governing council by the Ministry of Information and Broadcasting. Mr Singh, best known as the producer-director of popular TV series 'CID', is currently the vice-chairman of the FTII governing council. He succeeds veteran actor , who had stepped down as the chairman citing "international commitments".

SSCE 8981426494/8296260082 Downloaded from - www.onlinessce.com Page 64

8)What is the theme of 2018 International Universal Health Coverage Day?

Ans:-Unite for Universal Health Coverage: Now is the Time for Collective Action

Explanation:-International Universal Health Coverage Day (UHC Day) was observed throughout the world. It aims to mobilize diverse stakeholders to call for stronger, more equitable health systems to achieve universal health coverage, leaving no one behind. The theme for the 2018 UHC Day is: "Unite for Universal Health Coverage: Now is the Time for Collective Action". Universal health coverage (UHC) ensures all people, everywhere, can access the quality health services they need without suffering financial hardship. The United Nations proclaimed 12 December as International Universal Health Coverage Day (UHC Day) by resolution 72/138 in 2017.

9) The 39th Gulf Cooperation Council Summit was held in ______.

Ans:-Riyadh

Explanation:-The 39th Gulf Cooperation Council Summit was inaugurated by the King Salman of Saudi Arabia in Riyadh. It was announced at the conclusion of the summit that the 40th Gulf Cooperation Council (GCC) summit will be held in the United Arab Emirates. The presidency of GCC shifts among the six members based on the Arabic alphabet. Although the 39th summit was held in Riyadh upon a request from Oman, Oman will be the president for the next 12 months.

10)On which date, International Day of Neutrality observed?

Ans:- December 12

Explanation:-December 12 is seen as International Day of Neutrality after its adoption by United Nations General Assembly in the 69th plenary meeting. It is celebrated by United Nations to ensure neutrality among nations and abstention of a state from all participation in a war. It is aimed at promoting the use of preventive diplomacy, and enhancing public awareness of the value of neutrality in international relations.

11)World's first "floating" nuclear power plant was opened in;

Ans:-Russia

Explanation:-Akademik Lomonosov, the world's first "floating" nuclear power plant (FNPP) for installation in remote areas, has been started and brought to 10% of its capacity, Russian state-run atomic energy corporation Rosatom announced. An FNPP is basically a mobile, low-capacity reactor unit operable in remote areas isolated from the main power distribution system, or in places hard to access by land. They are designed to maintain both uninterruptible power and plentiful desalinated water supply in remote areas.

12)Which country tops in the Climate Change Performance Index?

SSCE 8981426494/8296260082 Downloaded from - www.onlinessce.com Page 65

Ans:-Sweden

Explanation:-The Climate Change Performance Index was released on the sidelines of COP24 Un Summit in Katowice, Poland. he index was produced by the New Climate Institute, Climate Action Network and Germanwatch. No countries were in the top three ranks because none did well in all categories assessed. But among the league of nations, Sweden topped as the 4th country (for the second time as on 2017) followed by Morocco in the 5th position. India ranked 11th, But Germany fell from 22nd to 27th because of its reliance on lignite coal, a big source of greenhouse gas emissions.

13)As per the study of Royal Society Open Science, which of the following is the fastest animal on earth?

Ans:-Dracula ant

Explanation:-In a study, published in the journal Royal Society Open Science, scientists found that the Dracula ant possesses the fastest animal movement on record. With speeds of up to 90 metres per second or 200 miles per hour, the ants can snap-jaws and power up their mandibles by pressing the tips together.

14)Who will be the head of the group constituted by the government to vet the recommendations of a high-level panel on stressed power projects?

Ans:-Arun Jaitley

Explanation:-The government has constituted a Group of Ministers (GoM) headed by Finance Minister Arun Jaitley to vet the recommendations of a high-level panel on stressed power projects. The panel, headed by Cabinet Secretary P K Sinha, had submitted its report in November 2018. The other members of the GOM are Road Transport Minister Nitin Gadkari, Commerce Minister Suresh Prabhu, Oil Minister Pradhan, Railways Minister Piyush Goyal and Power Minister R K Singh.

15)Which state will establish Umbrella University named after former PM late Atal Bihari Vajpayee?

Ans:- Uttar Pradesh

Explanation:-The Uttar Pradesh Government announced creation of an umbrella university named after former Prime Minister late Atal Bihari Vajpayee. The decision was taken in a cabinet meeting headed by Chief Minister Yogi Adityanath at Lucknow. The University would be the controlling authority of all the medical and dental colleges and nursing institutions in the state. This would help in bringing transparency in entrance and examination system and would enhance quality of medical education.

SSCE 8981426494/8296260082 Downloaded from - www.onlinessce.com Page 66

Daily Current Affairs 15thDec,2018

1)ECO Niwas Samhita 2018 was launched by the ministry of;

Ans:-Ministry of Power

Explanation:-Ministry of Power has launched the ECO Niwas Samhita 2018, an Energy Conservation Building Code for Residential Buildings (ECBC-R). The Code was launched on the occasion of National Energy ConservationDay 2018 (14th December) in the presence of Chief Guest Smt. Sumitra Mahajan, Hon'ble Speaker, Lok Sabha. It aims to benefit the occupants and the environment by promoting energy efficiency in design and construction of homes, apartments and townships.

2)UN honoured Indian startup 'HelpUsGreen' recently. This startup belongs to which state?

Ans:-Uttar Pradesh

Eplanation:-Uttar Pradesh-based 'HelpUsGreen' named startup has been honoured by the UN. The startup works to clean up the mighty river Ganga by recycling thousands of tonnes of floral waste from temples. 'HelpUsGreen' has come up with the world's first profitable solution to the monumental temple waste problem: 'flowercycling'.

3)For which state, Union Ministry of Agriculture conferred an award named Krishi Karman?

Ans:-Jharkhand

Explanation:-Jharkhand has been selected for "Krishi Karman" award in the rice category from the Union Ministry of Agriculture. The award to be given away in February 2019 carries a cash prize of Rs 2 crore and a citation.

4)India and this country agreed to simplify export clearances to take forward joint manufacturing in the defence.

Ans-Russia

Explanation:-India and Russia have agreed to simplify export clearances to take forward joint manufacturing in the defence. This decision was made during the 18th meeting of the India-Russia Inter-Governmental Commission on Military Technical Cooperation (IRIGC- MTC) chaired by Defence Minister Nirmala Sitharaman and her Russian counterpart General Sergei Shoigu on 13th December 2018. India is planning for joint production under Make in India. This includes allowing Indian companies, both Defence Public Sector Undertakings (DPSU) and private sector, to manufacture spares for Russian systems in India. So far, four military-industry conferences have been held between Russian manufactures and Indian companies to take the process forward, in this regard.

SSCE 8981426494/8296260082 Downloaded from - www.onlinessce.com Page 67

5)With which Association did the Indian Tea Association (ITA) sign MoU to promote green and black tea consumption in major tea markets?

Ans:-CTMA

Explanation:-Indian Tea Association (ITA) signed a memorandum of understanding (MoU) with China Tea Marketing Association (CTMA). It aims at promoting green and black tea consumption in major tea markets in Europe, the U.S., Russia, and West Asia. The MoU covers the areas of trade promotion, intellectual property protection, and technology exchange. China imported 30 million kg of black tea annually amid its rising popularity in the country where green tea had earlier held sway. Indian exports stood at about 8.7 million kg in 2017 with the market being dominated by Sri Lanka and Kenya. An export of 15 million kg was being targeted next year. China was the world's third largest exporter, mainly of green tea.

6)Who has entered the Limca Book of Records for training 2,08,125 women and girls in 989 self-defence programme in 2017?

Ans:-Delhi Police

Explanation:-Delhi Police has entered the Limca Book of Records for training 2,08,125 women and girls in 989 self-defence programme in 2017. The self-defence programme of Delhi Police was started in 2002 and is carried out by the Special Police Unit For Women and Children (SPUWAC) which has trained 9,80,456 women under 5,140 programmes till November 2017. The Delhi Police will receive the award in February next year. The Delhi Police have set a target to train around 3.5 lakh women in 2019.

7)Where was the 19th All India Police (AIPDM) Shooting Competition 2018 held?

Ans:-Manesar, Haryana

Eplanation:-Minister of State for Home Kiren Rijiju inaugurated the 19th All India Police (AIPDM) Shooting Competition-2018 at Manesar, Gurugram in Haryana, on 6th December 2018. The competition was held till 11th December 2018. More than 750 participants of 31 teams of State Police Forces and Central Armed Police Forces took part in the event. Total of 13 matches and 41 medals and 13 Trophies were awarded to the winners.

8)Which country has recently picked the Chinese character "disaster" to define 2018?

Ans:-Japan

Explanation:-Japan picked the Chinese character "disaster" to define the year 2018, a year that saw deadly floods, strong earthquakes and heavy storms. The selection came after Japan was buffeted this year by torrential rains in the country's west and an large earthquake in Hokkaido, heightening public awareness of the importance of disaster prevention measures. Japan was also rocked by man-made calamities such as stolen cryptocurrencies and the

SSCE 8981426494/8296260082 Downloaded from - www.onlinessce.com Page 68

uncovering of harassment by coaches in the athletic world. The competition is rum by every year by the Kyoto-based Kanji Aptitude Testing Foundation since 1995. The Chinese characters (or Kanji) are used widely in Japanese as well as other alphabets. Earlier, "North" won in 2017 after North Korea's missile launches, and "gold" in 2016 after the Rio Olympics.

9)Who has been appointed as a Secretary of NCST?

Ans:-Ashok Kumar Singh

Explanation:-Ashok Kumar Singh, Lok Sabha Secretariat Service, Additional Secretary, Lok Sabha Secretariat has assumed charge as Secretary, National Commission for Scheduled Tribes (NCST).

10)Who has been appointed as the presiding officer in the Securities Appellate Tribunal, Mumbai?

Ans:-Justice Tarun Agarwala

Explanation:-Justice Tarun Agarwala retired Chief Justice of the Meghalaya High Court has been appointed as the presiding officer in the Securities Appellate Tribunal, Mumbai. Justice Agarwala will remain presiding officer for a period of five years or till the age of 70 years, whichever is earlier. The Securities Appellate Tribunal is a statutory body established under the provisions of Section 15K of the Securities and Exchange Board of India(SEBI) Act, 1992, to hear and dispose of appeals against orders passed by the SEBI or by an adjudicating officer under the Act. Justice Agarwala had graduated from the Allahabad University with a law degree. He started practicing as an advocate in the Allahabad High Court in 1981 in the matters of Civil Law, Constitutional, Taxation, Labour, Testamentary and Corporate Affairs matters.

11)RBI grants 40 branch licenses to which of the following bank?

Ans:-Bandhan Bank

Explanation:-Reserve Bank of India (RBI) has allowed Bandhan Bank to open as many as 40 new branches, by the end of December, releasing the restriction that RBI has put on the Bandhan bank's branch opening spree three months back. The RBI had barred the bank from opening new branches in September after the private-sector lender failed to bring down main shareholder's stake to below 40 % as mandated in the licensing norms for universal banks. Bandhan Financial Services (BFSL) the promoter of Bandhan Bank, currently holds 82.28 % stake in the bank. The bank is planning either to float an open offer for sale or to acquire a non-bank lender which would in turn bring down the promoters' stake.

12)Who launched a mobile app for farmers in Myanmar?

Ans:-Ram Nath Kovind

SSCE 8981426494/8296260082 Downloaded from - www.onlinessce.com Page 69

Explanation:-President Ram Nath Kovind launched a mobile app developed for farmers in Myanmar and dedicated a centre and a bio park to promote agriculture research and education to the people. App provides crop and livestock knowledge and information apart from cropping guidelines based on location. The app links farmers with more than 2,000 experts.

13)How many states have achieved complete household electrification under the PM Sahaj Bijli Har Ghar Yojana (SAUBHAGYA) scheme, as per Ministry of Power's announcement?

Ans:-9

Explanation:-Ministry of Power announced that 9 states have achieved complete household electrification under the PM Sahaj Bijli Har Ghar Yojana (SAUBHAGYA) scheme. Madhya Pradesh, Tripura, Bihar, Jammu and Kashmir, Uttarakhand, Mizoram, Sikkim, Telangana and West Bengal. With this, 16 States in the country now have 100 per cent household electrification. Furthermore, the country is expected to achieve 100 per cent household electrification by December 31, 2018. Additionally, 31.68 crore LED bulbs have been distributed under the Unnat Jyoti by Affordable LED for All (UJALA) scheme resulting in estimated cost saving of Rs.16,457 crore per year. The Power Ministry also aims to replace 1.34 crore conventional streetlights with smart and energy efficient LED lights by March, 2019.

14)Who of the following has been selected for the National Bravery Award?

Ans:-Situ Malik

Explanation:-Situ Malik (15), a minor boy from Kendrapara district of Odisha has been selected for the National Bravery Award for saving his paternal uncle from a crocodile attack. He will be awarded from Prime Minister Narendra Modi on January 23, 2019. Situ had saved his uncle Binod Malik from attack of a crocodile which had sneaked into the village pond on February 20 2018.

15)The fourth Partners' Forum to improve the health and well-being of women, children and adolescents, held in ______.

Ans:-New Delhi

Explanation:-The Prime Minister, Shri Narendra Modi, inaugurated the 2-day fourth Partners' Forum in New Delhi. It concluded on December 13, 2018. India is hosting the Partners' Forum for the second time. The previous chapters were held in Johannesburg, South Africa in 2014, New Delhi, India in 2010 and Dar es Salaam, Tanzania in 2007. Its objective was to improve the health and well-being of women, children and adolescents.

16)Who has been honoured with Jnanpith Award for 2018?

SSCE 8981426494/8296260082 Downloaded from - www.onlinessce.com Page 70

Ans:-Amitav Ghosh

Explanation:-Noted English author Amitav Ghosh has been honoured with Jnanpith Award for 2018, an award for outstanding contribution to literature. The decision was taken in a meeting of Jnanpith Selection Board chaired by eminent novelist, scholar and Jnanpith laureate Pratibha Ray. Ghosh, one of the most prominent contemporary Indian writers, is known for a series of novels such as "Shadow Lines", "The Glass Palace", "The Hungry Tide", and Ibis Trilogy - "Sea of Poppies", and "River of Smoke". He has become the first English author to be honoured with the award.

17)India's first private sector unit for manufacturing unmanned aerial vehicles was opened in ______.

Ans:-Hyderabad

Explanation:-India's first private sector unit for manufacturing unmanned aerial vehicles (UAVs) opened at Adani Aerospace Park in Hyderabad. Built by the Adani Group and Israel-based Elbit Systems, the 50,000 square feet facility will develop Hermes 900 medium altitude long-endurance UAVs for the Indian and global markets. The unit will begin with the manufacture of complete carbon composite aerostructures for Hermes 900, followed by Hermes 480, catering to global markets.

18)Name of the IAS Officer who took Oath of Office and Secrecy as Member of Union Public Service Commission.

Ans:-Bharat Bhushan Vyas

Explanation:-Shri Bharat Bhushan Vyas, IAS officer from Jammu and Kashmir cadre of 1986 batch was administered the Oath of Office and Secrecy as Member, Union Public Service Commission (UPSC) by Shri Arvind Saxena, Chairman, UPSC. Born in 1957, Shri Bharat Bhushan Vyas has served in State & Centre in various capacities, retired as Chief Secretary, Jammu & Kashmir and was serving as Adviser to the Governor, Jammu & Kashmir before assuming the charge as Member, UPSC.

19)Name of the Atomic Power Station (APS) which has created a world record for the longest uninterrupted operation for 941 days.

Ans:-Kaiga

Explanation:- The Kaiga Atomic Power Station (KAPS) has created a world record for the longest uninterrupted operation for 941 days. The Unit-1 of Kaiga Generating Station (KGS) at Kaiga, located the river Kali in Uttar Kannada district of Karnataka, has been generating electricity continuously since May 13, 2016. It is an indigenously-built PHWR run by domestic fuel (uranium). It began commercial operations on November 16, 2000, and has produced 500 cr. units of power so far. It evidences the excellence in design, construction, safety, quality and operation & maintenance practices of Nuclear Power Corporation of India

SSCE 8981426494/8296260082 Downloaded from - www.onlinessce.com Page 71

(NPCIL). Earlier, the record of 940 days was held by Unit 2 of Heysham of the United Kingdom (UK).

20)This company has set up its Global Artificial Intelligence Accelerator (GAIA) in India.

Ans:-Ericsson

Explanation:Ericsson, Swedish telecom company, said it has set up its Global Artificial Intelligence Accelerator (GAIA) in India. The facility has been planned to be set up in Bengaluru. It will focus on research and development in Artificial Intelligence and automation and will be key in accelerating the execution of Ericsson's strategy. This facility will help create 150 new jobs for data scientists, engineers, AI architects and software developers in 2019.

Daily Current Affairs 16th Dec,2018

1) Who has been appointed as the next Ambassador of India to the Republic of Guinea?

Ans:-T.C. Barupal

Explanation:-T.C. Barupal was appointed the next Ambassador of India to the Republic of Guinea. He is presently Consul General of India, Zanzibar.

2)Who will be the head of the 7-member panel constituted by the government to select managing directors of public sector insurance companies?

Ans:-B P Sharma

Explanation:-The government has constituted a seven-member panel to select managing directors of public sector insurance companies. The panel would be headed by Banks Board Bureau (BBB) chairman B P Sharma. The other members of the panel are Financial Services Secretary, Department of Public Enterprises Secretary and Chairman of Insurance Regulatory and Development Authority of India. Three insurance sector experts G N Bajpai, Mathew Varghese and T Bhargava have also been appointed on the selection panel. In 2014 the Modi government had constituted BBB to select managing directors and chairman for public sectors banks after the post of CMDs was bifurcated into two separate positions as non-executive chairman and managing director.

3)This company won the Prime Minister's Trophy for the 'Best Performing Integrated Steel Plant' in India for the year 2016-17.

Ans:-Tata Steel

Explanation:-Tata Steel Limited had been declared as the winner of the Prime Minister's Trophy for the 'Best Performing Integrated Steel Plant' in the country for the performance

SSCE 8981426494/8296260082 Downloaded from - www.onlinessce.com Page 72

year 2016-17 making it the fourth consecutive year for Tata Steel being honoured with this coveted award. Along with the trophy a cash award of Rs. 2 crore was also given to the company. Having won the PM's Trophy 11 times and receiving the Certificate for Excellence twice, Tata Steel has been recognized as the best integrated steel plant 13 times, since the inception of this award in 1992-93. The performance of the plants is judged by a Panel of Judges headed by Secretary (Steel) and with members from eminent technologists, management experts, trade union leaders, economists, consumers of iron & steel according to a scheme drawn up by an expert committee.

4)The new human protein discovered by researchers to fight the Ebola virus.

Ans:-RBBP6

Explanation:-Researchers have discovered a human protein that helps fight the Ebola virus, and could one day lead to an effective therapy against the deadly disease. The newly discovered ability of the human protein RBBP6 to interfere with Ebola virus replication suggests new ways to fight the infection, according to the study published in the journal Cell.

5)The 21st Usha National Athletics Sports Championship for the Blind held in ______.

Ans:-New Delhi

Explanation:-The 21st Usha National Athletics Sports Championship for the Blind held in New Delhi in the presence of champion boxer Vijender Singh. The championship witnessed six new national records in javelin, long jump, discuss, 100m race and shotput and three new meet records in 100 meet race, javelin and shotput. Radhika Bharat Ram is chairperson of India Blind Sports Association.

6)Which state has emerged as the largest beneficiary of Ayushman Bharat scheme, till November 2018?

Ans:-West Bengal

Eplanation:-The minister of state for health and family welfare Anupriya Patel announced that West Bengal, Bihar and Chhattisgarh emerged the largest beneficiaries of Ayushman Bharat. Till November 2018, the government had spent Rs 798.34 crore on the scheme, of which West Bengal was allotted a total of Rs 193.34 crore, Bihar got an allocation of Rs 188.27 crore, and Chhattisgarh Rs 114.43 crore.

7)How many Indian companies placed in the ranking list of Stockholm International Peace Research Institute's?

Ans:-4

SSCE 8981426494/8296260082 Downloaded from - www.onlinessce.com Page 73

Explanation:-As per the Stockholm International Peace Research Institute's (SIPRI) latest rankings, four Indian companies figure among the world's top 100 arms producers in 2017. The four companies are Indian Ordnance Factories (Rank 37), Hindustan Aeronautics (Rank 38), Bharat Electronics (Rank 64), and Bharat Dynamics (Rank 94). Lockheed Martin of the United States got the 1st rank.

8)Which two countries signed a bilateral annual Haj agreement 2019?

Ans:-India and Saudi Arabia

Explanation:- A bilateral annual Haj agreement 2019 was signed between India and Saudi Arabia by Union Minister for Minority Affairs, Shri Mukhtar Abbas Naqvi, and Umrah Minister of Kingdom of Saudi Arabia, Dr Mohammad Saleh bin Taher Benten at Jeddah, Saudi Arabia. This year more than 2,100 women have already applied to go to Haj without 'Mehram' (male companion) after the Modi Government had lifted ban on women going to Haj without Mehram last year and they had also been exempted from the lottery system. The Haj Committee of India has received more than 2.47 lakh applications next year's pilgrimage which includes about 47 per cent women.

9) Who was re-designated from Advisor to Principal Advisor in NITI Aayog?

Ans:-Anil Srivastava

Explanation:-Appointments Committee of the Cabinet (ACC) approved the re-designation of Shri Anil Srivastava from Advisor to Principal Advisor in NITI Aayog. He is a 1985 IAS from the Madhya Pradesh care who is currently an Advisor, NITI Aayog.

10)Who was named the first woman CEO of the French unit of Air France-KLM Group?

Ans:-Anne Rigail

Explanation:-The French unit of Air France-KLM Group named Anne Rigail as its new CEO, making her the first woman to lead the airline. The 49-year-old, who is currently Executive Vice President (Customer) at Air France, will replace acting CEO Benjamin Smith.

11)The government of this country bans the Indian currency notes above Rs 100.

Ans:-Nepal

Explanation:-The Nepal government banned Indian currency above Rs 100. New Indian note of Rs 2,000, Rs 500 and Rs 200 denominations were banned as the Nepal government has not yet legalised them in the market. The announcement was made by Nepal's Minister for Information and Communications Gokul Prasad Baskota. The move can affect the Indian tourist and Nepali people working in India, who carry the currency in Nepal for transaction.

SSCE 8981426494/8296260082 Downloaded from - www.onlinessce.com Page 74

12)Who won the Lifetime Achievement Award at the 23rd International Film Festival of Kerala?

Ans:-Majid Majidi

Explanation:-The Chief Minister of Kerala Pinarayi Vijayan inaugurated the closing ceremony of the 23rd International Film Festival of Kerala (IFFK) at Nisagandhi Auditorium in Thiruvananthapuram, Kerala. Around 160 films from around the world were screened at the week-long festival that commenced on December 7, 2018. The winners of the IFFK 2018 awards are Lifetime Achievement Award: Filmmaker Majid Majidi (Iran). Suvarna Chakoram for Best Film: The Dark Room. Rajata Chakoram for Best Director: Lijo Jose Pellissery for Ee.Maa.Yove.

13)Which company signed an MoU with FISME to empower MSME sector?

Ans:-Amazon India

Explanation:-Amazon India partnered with the Federation of Indian Micro and Small & Medium Enterprises (FISME) to encourage MSMEs to sell products online through e- commerce. Amazon.in will collaborate with FISME to conduct various events and workshops across the country to educate MSMEs about Online selling domestically, B2C exports to enable them to sell across multiple international locations. Areas such as logistics, cataloguing, imaging, taxation, Brand building, documentation, listing methodology and services such as Fulfilment by Amazon & Sponsored Ads.

14)Who launched an Operation Oliva as part of its annual mission to ensure the safe mid-sea sojourn of breeding Olive Ridley sea turtles in Gahirmatha marine sanctuary area in Odisha?

Ans:-Indian Coast Guard

Explanation:-The Indian Coast Guard has launched 'Operation Oliva' as part of its annual mission to ensure the safe mid-sea sojourn of breeding Olive Ridley sea turtles in Gahirmatha marine sanctuary area, Devi river mouth, and Rushikulya beach of Odisha. The turtle conservation programme is executed in a joint coordination with the forest department of Odisha to keep watch and vigil on illegal fishing along the turtle concentration zone. As a part of the operation, a fleet of ships and a service aircraft has been pressed into service besides the coast guard has also organised interactive sessions with fishermen communities to sensitise them on the legal embargo on fishing during ridleys' nesting season.

15)Which coutry will organise the 2019 Gita festival?

Ans:-Mauritius

Explanation:-Mauritius Acting President Paramasivum Pillay Vyapoory announced that Mauritius would organise a Gita festival in February 2019. A delegation from India has also

SSCE 8981426494/8296260082 Downloaded from - www.onlinessce.com Page 75

been invited for the event. He made this announcement at the inauguration of a three-day international seminar organised as part of the international Gita festival in Kurukshetra, Haryana.

Daily Current Affairs 17th Dec,2018

1)European Parliament approved the world's largest free trade agreement between which country and the EU?

Ans:-Japan

Explanation:-The European Parliament approved a free trade agreement between Japan and the EU, covering 635 million people and almost one-third of the world's economy. The world's largest free trade agreement, the EU-Japan Economic Partnership Agreement will remove duties on almost all agricultural and industrial products as well as open up the service sector and procurement. It also moves to eliminate non-tariff barriers to trade. The EU-Japan Economic Partnership Agreement which has been under discussion since 2013 will come into effect from 1stFebruary 2019. This agreement is projected to increase exports between the two economies by 34 percent for the EU and 29 percent for Japan, liberalising up to 99 percent of bilateral trade. The elimination of tariffs will save consumers and importers $1 billion per year in the EU and will support a substantial increase of jobs, maximizing benefits for both companies and citizens.

2)Who was appointed as the new Director of National Crime Record Bureau (NCRB), recently?

Ans:-Ramphal Pawar

Explanation:- The Government appointed Ramphal Pawar as the new Director of National Crime Record Bureau (NCRB). An IPS officer from the West Bengal cadre, Mr Pawar was Joint Secretary in NATGRID. He will be joining NCRB against existing vacancy from the date of joining the post and up to tenure extendable as per IPS Tenure Policy or till further orders, whichever is earlier.

3)Union Ministers Shri Shripad Yesso Naik and Dr. Jitendra Singh laid the foundation stone of IHAMP in which state?

Ans:-Jammu and Kashmir

Explanation:-Union Minister of State (Independent Charge) for AYUSH Shri Shripad Yesso Naik and Union Minister of State in Prime Minister's Office (PMO) Dr. Jitendra Singh laid the foundation stone of Institute of High Altitude Medicinal Plants (IHAMP) with an estimated cost of Rs. 100 crore at Bhaderwah in District Doda in Jammu & Kashmir. The institute will be a dedicated facility for research in medicinal plants found at high altitude and shall also become a major source of income for the farmers in cultivation of medicinal plants.

SSCE 8981426494/8296260082 Downloaded from - www.onlinessce.com Page 76

4)Which team won the first ever World Cup title in Odisha?

Ans:-Belgium

Explanation:-In Hockey, Belgium has claimed their first ever World Cup title. They defeated the Netherlands in a dramatic shoot-out in the final of the Men's World Cup in Bhubaneswar, Odisha. Both the teams, goalless at full time, were at level 2-2 after five penalties. This is Belgium's first ever World Cup title. Earlier, the last edition champions Australia secured a consolation bronze medal.

5)Name the Indian American who has been crowned Miss India Worldwide 2018 at Fords City of New Jersey.

Ans:-Shree Saini

Explanation:- Indian-American Shree Saini has been crowned Miss India Worldwide 2018 at a pageant held in the Fords City of New Jersey. Sakshi Sinha from Australia and Anusha Sareen from the United Kingdom were respectively declared first and second runner-ups in the 27th annual global pageant for people of Indian origin. Indian-origin girls from as many as 17 countries participated in the annual beauty pageant. India's Mandeep Kaur Sandhu, who hails from Haryana, was crowned Mrs. India Worldwide 2018.

6)Which country Sets World Record in Nuclear Plant Operation?

Ans:-India

Explanation:-The indigenously developed Kaiga Atomic Power Station in Karnataka has set a new world record, with one of its units operating uninterrupted for more than 940 days. This is a record for all kinds of nuclear power-generating units, including advanced gas- based reactors in the world. The record was earlier held by Unit 2 of Heysham of the United Kingdom, which operated non-stop for 940 days. The same unit had set a world record in October 2018, by operating uninterrupted for 894 days among Pressurized Heavy Water Reactors.

7)The Nigerian military has recently lifted ban on this UN agency operations in ravaged northeast.

Ans:-UNICEF

Explanation:-The Nigerian military has lifted a ban on UNICEF operations in the ravaged northeast of the country. Earlier, the army had suspended the aid agency & accused it of training spies supporting Boko Haram jihadists. Boko Haram's Islamist insurgency has killed more than 27,000 people since it began in 2009 and has triggered a humanitarian crisis in the wider Lake Chad region, where the jihadists have stepped up attacks in recent months.

8) Who becomes the first Indian to win BWF World Tour Finals tournament?

SSCE 8981426494/8296260082 Downloaded from - www.onlinessce.com Page 77

Ans:-PV Sindhu

Explanation:-PV Sindhu made history, becoming the first Indian to win the season-ending BWF World Tour Finals tournament. Sindhu came up against a familiar in Nozomi Okuhara in the title clash but unlike her previous meetings, the Indian superstar showed much more patience in her gameplay against the Japanese. This was critical to the Indian shuttler's cause as Okuhara was made to toil and given no breathing space as PV Sindhu claimed win in the final for her first title of the year.

9)Which country agreed to fight terrorism jointly with India in the Indo-Pacific, civil nuclear, defence and security, space, trade and economic sectors?

Ans:-France

Eplanation:-India and France also agreed to enhance their strategic partnership through cooperation in the Indo-Pacific, civil nuclear, defence and security, space, trade and economic sectors. India's External Affairs Minister (EAM) Sushma Swaraj stated that Paris and New Delhi are working together on multiple international forums including Financial Action Task Force (FATF). The two nations are jointly working on Jaitapur nuclear power project.

10)Ranil Wickremsinghe was sworn in as Prime minister of ______.

Ans:-Sri Lanka

Explanation:-Mr. Ranil Wickremsinghe was sworn in as Prime minister of Sri Lanka by President Maithripala Sirisena thus bringing an end to the weeks-long political crisis in the country. Mr. Wickremsinghe was removed as Prime minister in October and was replaced by Mahinda Rajapaksa who resigned after failing to garner a majority in Parliament. This is the fifth time Mr. Wickremsinghe has been sworn in as Prime minister which is a record in Sri Lanka.

11)The 3rd Edition of the Women Transforming India Awards 2018 was organised by;

Ans:-NITI Aayog

Explanation:-The NITI Aayog organised the 3rd Edition of the Women Transforming India Awards 2018 and launched the upgraded portal of the Women Entrepreneurship Platform (WEP). The Vice President of India, M. Venkaiah Naidu presided over as the chief guest of the event where he launched the WEP Portal 2.0 and conferred WTI Awards 2018. The Women Transforming India Awards were instituted to recognise and celebrate stories of exemplary women from across India. This year's theme is "Women and Entrepreneurship" and fifteen winners have been chosen through an extensive six-month-long process to identify fascinating and inspiring stories of women entrepreneurs.

SSCE 8981426494/8296260082 Downloaded from - www.onlinessce.com Page 78

12)Catriona Gray has been crowned Miss Universe 2018. The native country of Catriona Gray is;

Ans:-Philippines

Explanation:-Philippine's Catriona Gray has been crowned Miss Universe 2018, becoming the fourth woman from the country to win the title. Tamaryn Green from South Africa was named the first runner-up while Miss Venezuela Sthefany Gutierrez stood third at the pageant which was held in Bangkok, Thailand. The 24-year-old Gray, who was born in Australia and studied at the Berklee College of Music in Boston, Massachusetts, was one of the odds-on favorites going into the competition. India was represented by 22-year-old Nehal Chudasama.

13)Which sports player makes record to score 25 Test Centuries after Sir Don Bradman?

Ans:-Virat Kohli

Explanation:-Indian Cricket Team captain Virat Kohli became the second fastest player to score 25 Test Centuries after scoring 123 runs on the third day of Second Test against Australia at Perth. Virat Kohli is only behind of former Australian captain and cricketing legend Sir Don Bradman. Virat Kohli also equalled Sachin Tendulkar's record of 6 Test Centuries in Australia.

14)Which country will host the 2020 Asia cricket cup, as approved by Asian Cricket Council?

Ans:-Pakistan

Explanation:-The Asian Cricket Council (ACC) awarded the hosting rights of the 2020 Asia cup to the Pakistan Cricket Board (PCB) in a meeting held in Bangladesh Capital Dhaka. Pakistan will host the 2020 Asia cricket cup. The Decision was taken by ACC President Nazmul Hassan Papon but the final venue of the event is still unclear over whether it will be played in UAE or Pakistan.

15)On which date Vijay Diwas was celebrated?

Ans:-December 16

Explanation:-The nation celebrated Vijay Diwas to commemorate India's victory over Pakistan in 1971 war - the war for the Independence of Bangladesh from Pakistan. The day marks the end of Bangladesh Liberation War, when the chief of the Pakistani forces, General Amir Abdullah Khan Niazi, along with 93 thousand troops, had surrendered unconditionally to the allied forces consisting of Indian Army and Mukti Bahini, led by Lieutenant General Jagjit Singh Aurora in Dhaka. A 72-member delegation of war veterans and Mukti Jodhas,

SSCE 8981426494/8296260082 Downloaded from - www.onlinessce.com Page 79

led by parliamentarian Ms. Quazi Rosy from Bangladesh are taking part in Vijay Diwas celebrations.

16)How many Indian war veterans are honoured on the Bangladesh's 47th anniversary of the Victory Day?

Ans:-12

Explanation:-Bangladesh in its 47th anniversary of the 'Victory Day', 16th December, honoured 12 Indian war veterans, marking its emergence as an independent nation following the 1971 Liberation War. President Abdul Hamid and Prime Minister Sheikh Hasina urged the nation to work together for the prosperity of the country. It honoured the 12 Indian armed forces personnel for sacrificing their lives during the 1971 war. In India, Vijay Diwas (Victory Day) is celebrated every year on this day to mark India's decisive win over Pakistan and the birth of Bangladesh.

17)A National Medical Devices Promotion Council will be set up under the Ministry of Commerce and Industry. Who will be the head of the Council?

Ans:-Ramesh Abhishek

Explanation:-At the 4th WHO Forum, held in Visakhapatnam, Andhra Pradesh, from 13th to 15th December 2018, Union Minister of Commerce and Industry and Civil Aviation, Suresh Prabhu, announced setting up of a National Medical Devices Promotion Council under the Department of Industrial Policy and Promotion (DIPP) in the Ministry of Commerce & Industry. The Council will be headed by Secretary, DIPP, Shri. Ramesh Abhishek. It will also have representatives of healthcare industry and quality control institutions. Furthermore, Andhra Pradesh MedTech Zone, Visakhapatnam, will provide technical support to the Council. The establishment of the council will boost the domestic manufacturing sector and would aid in exports as well.

18)Where was the 4th WHO Global Forum on Medical Devices "Increasing access to medical devices" held?

Ans:-Visakhapatnam, Andhra Pradesh

Explanation:-The three day 4th WHO Global Forum on Medical Devices "Increasing access to medical devices" concluded Andhra Pradesh Medtech Zone in Visakhapatnam, Andhra Pradesh. It was the first time the WHO Global Forum on Medical devices was being held in India. This Forum was meant to face the challenges present in the selection, regulation and use of medical devices.

19)Who has been sworn-in as the new Chief Minister of Mizoram?

Ans:- Pu Zoramthanga

SSCE 8981426494/8296260082 Downloaded from - www.onlinessce.com Page 80

Explanation:-Pu Zoramthanga, the President of Mizo National Front (MNF), has been sworn-in as the new Chief Minister of Mizoram. He was administered the oath of office and secrecy in Mizo Language by Mizoram Governor K Rajasekharan during a ceremony at Raj Bhavan in Aizawl. This is his third stint as chief minister of the northeastern state, having led the MNF government in Mizoram in 1998 and 2003. Mr Tawnluia will be the deputy chief minister. The MNF came to power in Mizoram capturing 26 of the 40 seats in the state assembly in the recently-held polls.

20) Who was honoured with the French Knight of the National Order of Merit?

Ans:-Ashok Amritraj

Explanation:-Producer Ashok Amritraj was honoured with the French Knight of the National Order of Merit. The former CEO of National Geographic Films and United Nations Goodwill Ambassador was lauded for his contribution to the world of cinema and Indo- French film industry. Mr. Amritraj has been the producer of Hollywood films such as Ghost Rider: Spirit of Vengeance with Nicholas Cage and 99 Homes starring Andrew Garfield.

Daily Current Affairs 18th Dec,2018

1)India's first private Unmanned Aerial Vehicles (UAV) factory has opened in ______.

Ans:-Hyderabad

Explanation:-India's first private sector unit for manufacturing Unmanned Aerial Vehicles (UAVs) has recently opened at Adani Aerospace Park near Shamshabad in Hyderabad. The 50,000-sq ft state-of-the-art facility facility is built by the Adani Group and Israel-based Elbit Systems. It will start operations with the manufacturing of complete carbon composite aero- structures for the Hermes 900 UAV, followed by the Hermes 450 UAV, catering to the global markets, and will further be ramped-up for the assembly and integration of complete UAVs here. As of now, after manufacturing, the integration of the UAV units will be done in Israel.

2)India and Afghanistan signed an MoU in the field of:

Ans:-Human resource development

Explanation:-India and Afghanistan signed an MoU in the field of human resource development on 18th December 2018. The Union Cabinet chaired by Prime Minister Narendra Modi also approved an MoU between India and France in the field of new and renewable energy. Establishment of two new AIIMS at Tamil Nadu & Telangana under Pradhan Mantri Swasthya Suraksha Yojana. Apprised of Joint Issue of Postage Stamps by India and South Africa. MoU between India and Australia for Cooperation in the Disability Sector.

SSCE 8981426494/8296260082 Downloaded from - www.onlinessce.com Page 81

3)The President of this country declares 2019 as Year of Tolerance.

Ans:-UAE

Explanation:-United Arab Emirates (UAE) President H.H. Sheikh Khalifa bin Zayed Al Nahyan has declared 2019 to be the "Year of Tolerance." to highlight UAE as the Global Capital of Tolerance, instilling the values of co-existence and peace in local, regional and international communities The Year of Tolerance will focus on "Five Pillars". First one aims to deepen the values of tolerance and co-existence among cultures and peoples by concentrating on teaching the youth on the values of tolerance. The second pillar will focus on solidifying the UAE as the global capital for tolerance, via a series of projects with dialogue between various cultures and civilizations The third will enact multiple cultural programmes and contributions to build tolerant communities.

4)"One City-One Operator scheme" is related to which of the given department?

Ans:-Sewage

Explanation:-In Uttar Pradesh, Prime Minister Narendra Modi has recently inaugurated a 175 km long of sewerage network at Prayagraj, which will carry 7.8 Cr litres sewage water per day to existing STPs (Sewage Treatment Plants) at Salori, Kodra & Ponghat for treatment which earlier ended up in Ganga. He also laid the foundation stone for two projects in the trans-Ganga/Yamuna area under One City-One Operator scheme with a sanctioned budget of Rs 767.59 crore.The project is based on Hybrid Annuity based Public Private Partnership (PPP) mode that includes operations and maintenance of all the sewerage infrastructure for 15 years. These projects include intersection and diversion (I&D) of sewage network, seven sewage pumping stations and three STPs having a total capacity of 72 million litre per day (MLD). With these projects, there will be comprehensive sewerage coverage of Prayagraj, including areas on the other side of the Ganga and Yamuna rivers.

5)Who designed satellite phones NaVIC to fishermen involved in deep sea fishing?

Ans:-Indian Space Research Organisation

Explanation:-Tamil Nadu Chief Minister Edappadi K. Palaniswami launched the distribution of satellite phones to fishermen involved in deep sea fishing. It will assist the fishermen in their navigation while in the high seas. The Indian Space Research Organisation (ISRO) has designed the NaVIC (Indian Regional Navigation Satellite System) receivers in a bid to the fishermen. So far, ISRO has delivered 200 NaVICs to the State government.

6)The British Telecommunication firm which plans to set up a research centre in Bengaluru?

Ans:-Global Telecom

SSCE 8981426494/8296260082 Downloaded from - www.onlinessce.com Page 82

Explanation:- Global telecom, British Telecommunications giant, said it plans to set up a research centre in Bengaluru in partnership with the Indian Institute of Science. The British Telecom Indian Research Centre will focus on the development of artificial intelligence, mobility and cyber-security technologies for use in the company's programmes, products, and services. The India centre's structure and organization will follow a model used for the British firm's global research locations, combining academic, industry and government partnerships. It will join BT's network of collaborative research facilities in China, Northern Ireland, the UAE, and the US.

7)Ashok Gehlot sworn-in as 12th Chief Minister of this state recently.

Ans:-Rajasthan

Explanation:-Ashok Gehlot took oath as the Rajasthan 12th Chief Minister of Rajasthan whereas Sachin Pilot took oath as the Deputy Chief Minister after the successful lead in 2018 Rajasthan Legislative Assembly election. Governor Kalyan Singh administered the oath at the historic Albert Hall in Jaipur,Rajasthan. Congress President Rahul Gandhi, former Prime Minister Manmohan Singh, outgoing chief minister Vasundhara Raje attended the event. Ashok Gehlot was a senior congress leader who has selected for Chief Minister third time. He was a Chief Minister for the first time in 1998 and again in 2008.

8) Information Fusion Centre (IFC) for the Indian Ocean Region (IOR) will be inagurated by;

Ans:-Indian Navy

Explanation:-The Navy will inaugurate the Information Fusion Centre (IFC) for the Indian Ocean Region (IOR). Through this Centre, information on 'white shipping', or commercial shipping, will be exchanged with countries in the region to improve maritime domain awareness in the Indian Ocean. The IFC has been established at the Navy's Information Management and Analysis Centre (IMAC) in Gurugram, Haryana.

9)Which aircraft was tested by Indian Air Force (IAF) and flew with blended bio-jet fuel?

Ans:-An-32 transport aircraft

Explanation:-An-32 transport aircraft of the Indian Air Force (IAF) flew with blended bio- jet fuel produced from Jatropha oil, unlocking the possibility of gradually expanding it to the entire service at some point. This has the dual benefit of reducing the carbon footprint as well as usage of fossil fuels. It was flown by highly qualified test pilots from the Aircraft and System Testing Establishment (ASTE). The IAF intended to fly the An-32 with 10% bio-jet fuel at the Republic Day flypast on January 26, 2019. This bio-jet fuel technology was developed by the Indian Institute of Petroleum (IIP) under the Council of Scientific and Industrial Research (CSIR) in 2009 and tested between 2011 and 2013.

SSCE 8981426494/8296260082 Downloaded from - www.onlinessce.com Page 83

10)On 18th December, The Union Cabinet approved an MoU between India and this country for Cooperation in the Disability Sector.

Ans:-Australia

Explanation:-The Union Cabinet chaired by Prime Minister Narendra Modi has approved an MoU between India and Australia for Cooperation in the Disability Sector. It also approves Establishment of two new AIIMS at Tamil Nadu & Telangana under Pradhan Mantri Swasthya Suraksha Yojana. Apprised of Joint Issue of Postage Stamps by India and South Africa.

11)Who launched an infrastructure and housing projects in Maharashtra?

Ans:-Shri Narendra Modi

Explanation:-Prime Minister Narendra Modi launched infrastructure and housing projects worth about Rs 41,000 crore in Maharashtra. During his day-long visit to the state, Prime Minister Modi laid the foundation stone for two important metro corridors at a public meeting in Kalyan (Thane-Bhiwandi-Kalyan Metro and the Dahisar-Mira-Bhayander Metro). Prime Minister also launched mass housing scheme worth 18,000 crore rupees which offers around 90,000 affordable homes for economically weaker sections under the Pradhan Mantri Awas Yojana (PMAY). Prime Minister Modi also visited Raj Bhavan in Mumbai to release a book titled "Timeless Laxman", based on the famous cartoonist, RK Laxman.

12)India's first railway university was opened in ______.

Ans:-Vadodara

Explanation:- The Union Railway Minister Piyush Goyal dedicated the National Rail and Transportation Institute (NRTI), India's first Railway University, to the nation in Vadodara on 15 December 2018. It is third such university in the world after Russia and China. The Railway University began its operations on 5th September 2018

13)Where was the 37th Senior National Rowing Championship begins?

Ans:-Pune

Explanation:-The 37th Senior National Rowing Championship began in the Army Rowing Node in Pune. Around 500 rowers and coaches from various rowing associations across the country are taking part in the Championship. The Rowers for the World Championships- 2019 will be selected from this event. World Championship -2019 is the first qualifying Championship for the 2020 Tokyo Olympics.

14)Which of the becomes 1st State To Bring Law Against 'Sextortion'?

Ans:-Jammu and Kashmir

SSCE 8981426494/8296260082 Downloaded from - www.onlinessce.com Page 84

Explanation:-Jammu and Kashmir has become the first state in the country to enact a law banning sexual exploitation of women "by those in positions of authority, having a fiduciary relationship or a public servant", Under Section 354E of the Ranbir Penal Code (RPC). Any person committing the offence of sextortion shall be punished with rigorous imprisonment for a term not be less than three years, but may extend to five years and with fine. The offence is "non-bailable" and "not compoundable".

15)Department of Telecommunication (DoT) expects to complete processes for 5G spectrum auction by;

Ans:-August 2019

Explanation:-Department of Telecommunication (DoT) said it expects to complete processes for 5G spectrum auction by August 2019 and the services will be rolled out by 2020. The Telecom Regulatory Authority of India (TRAI) recommended auction of about 8,644 MHz of telecom frequencies at an estimated total base price of Rs 4.9 lakh crore, which includes radio waves for 5G services. The roll out of 5G services in India will have an impact of USD 1 trillion on the economy.

16)Who is appointed as the Chief Executive Officer (CEO) of Thyssenkrupp Tata Steel B.V.?

Ans:-Andreas Goss

Explanation:-Andreas Goss is appointed as the Chief Executive Officer (CEO) of Thyssenkrupp Tata Steel B.V., the proposed joint venture (JV) between Tata Steel and Thyssenkrupp. Mr.Goss is currently the CEO of Thyssenkrupp's steel division. He would chair the management board of Thyssenkrupp Tata Steel B.V. Hans Fischer who is now the CEO of Tata Steel Europe would be the deputy CEO and chief technology officer of the JV Tata Steel and Thyssenkrupp had also decided on the members of the future management board for the JV to be headquartered in Amsterdam.

17)Star India's Chairman and CEO Uday Shankar has been elected Vice President for which of the following firm?

Ans:-FICCI

Explanation:-Star India's Chairman and CEO Uday Shankar has been elected Vice President of Federation of Indian Chambers of Commerce and Industry (FICCI) for 2018-19. He was recently appointed Star and Disney India's Chairman and President of The Walt Disney Company Asia Pacific. Shankar is the first-ever Indian media and entertainment executive to assume the leadership position in a national industry chamber.

18)Name the Indian film which is shortlisted for Oscar in the Documentary Short Subject category.

SSCE 8981426494/8296260082 Downloaded from - www.onlinessce.com Page 85

Ans:-Period. End Of Sentence

Explanation:-'Period. End of Sentence' has made it to the Oscar shortlist in the Documentary Short Subject category. This is a film about women in India fighting against the deeply rooted stigma of menstruation and delving upon the work of real-life 'Pad Man' Arunachalam Muruganantham. A total of 104 films had originally qualified in the category. India's entry for the Foreign Language Film Category, Village Rockstars is out of the Oscar race.

Daily Current Affairs 18th Dec,2018

1)Name the company which has partnered with Ola recently.

Ans:-Vogo

Explanation:-Vogo, a scooter sharing start-up, has partnered with Ola, the ride-hailing company. Ola will boost Vogo's supply by investing in 1,00,000 scooters on the Vogo platform, worth $100 million. Vogo's offerings will also be available for Ola's customer base of over 150 million, directly from the Ola app. Every Vogo scooter comes with an Internet of Things (IoT) device that allows the customer to access the key via a one-time password (OTP). This eliminates the need for human intervention and offers consumers a seamless experience.

2)Who was the author of the book named "God of Sin: The Cult, The Clout and Downfall of Asaram Bapu"?

Ans:-Ushinor Majumdar

Explanation:-Book on self-styled godman Asaram Bapu released after a Chandigarh court refused to stay its publication. Authored by Ushinor Majumdar named "God of Sin: The Cult, The Clout and Downfall of Asaram Bapu". The plea was filed seeking a permanent injunction against publishing of the book. Book was published Penguin Random. "God of Sin" is about Asaram's journey to spiritual godhood, his fall from grace and the long and arduous road to bring him to justice.

3)Who was appointed as the Additional Director of CBI?

Ans:-M.Nageswara Rao

Eplanation:-CBI's Joint Director M Nageswara Rao, who is currently the interim CBI Director, was promoted to the rank of Additional Director by the Appointments Committee of the Cabinet. The 1986-batch IPS officer from the Odisha cadre had joined the CBI in 2016. Before being appointed interim chief, Rao was No. 3 in the pecking order and was handling the Chandigarh branch along with certain branches in South India.

4)What is the name of the most-distant body ever observed in our solar system?

SSCE 8981426494/8296260082 Downloaded from - www.onlinessce.com Page 86

Ans:-Farout

Explanation:-Scientists have discovered the most-distant body ever observed in our solar system, located over 100 times farther than Earth is from the Sun. The new object has been nicknamed Farout. Earlier, 2018 VG18, the object is the first to be observed at a distance of more than 100 astronomical units from the sun. Farout is about 120 AU, or about 11 billion miles, from the sun. One AU is the distance from Earth to the sun or about 93 million miles

5)Name the tennis player who has been named as the 2018 ITF (International Tennis Federation) World Champion under Men's Singles category.

Ans:-Novak Djokovic

Explanation:-The International Tennis Federation announced Serbia's Novak Djokovic as the 2018 ITF World Champion. The ITF World Champions will receive their awards at the 2019 ITF World Champions Dinner on Tuesday 4 June, in Paris, during Roland Garros. In Men's Singles Category: Djokovic receives the honour for the sixth time. He won four titles including Wimbledon and the US Open, and climb from No. 22 in the rankings to secure the year-end world No. 1 position. In the Men's Doubles Category:USA's Mike Bryan and Jack Sock will receive the award. It will be the 12th time Bryan has received the award but the first time Sock has been honoured.

6)Who has been named the top Indian star of 2018, by the global film and TV website IMDb?

Ans:-Deepika Padukone

Explanation:-The '2018 Top 10 Stars of Indian Cinema' list, based on global film and TV website Internet Movie Database's (IMDb) 'StarMeter' stats and page views, was released. Actress Deepika Padukone has been named the top Indian star of 2018, by the global film and TV website IMDb. Shah Rukh Khan was second on the list, followed by and Aishwarya Rai Bachchan.

7)With whom Nasscom join hands for reskilling students and employees in the Information Technology sector?

Ans:-IIT Madras

Explanation:-Nasscom, IT industry body, announced its partnership with the Indian Institute of Technology (IIT), Madras for reskilling students and employees in the Information Technology sector and IT-enabled Services (ITeS). The partnership is aimed at building an able workforce to support the industry and other stakeholders. Nasscom aims to get India accelerated on the path to becoming the global hub for talent for the new emerging technologies like AI (Artificial Intelligence), IoT (Internet of Things), cyber security, big data, cloud and blockchain.

SSCE 8981426494/8296260082 Downloaded from - www.onlinessce.com Page 87

8) The 9th India-ROK Joint Commission Meeting held in ______.

Ans:-New Delhi

Explantion:-External Affairs Minister Sushma Swaraj and Foreign Affairs Minister of South Korea Kang Kyung-wha co-chaired the 9th India-ROK Joint Commission meeting. Ms. Kyung-wha arrived in New Delhi on a two-day visit to India. Mrs. Swaraj and her Korean counterpart reviewed the progress in multifaceted cooperation under the Special Strategic Partnership between the two countries. It includes follow-up action on decisions taken during the visit of South Korean President Moon Jae-in to India in July this year.

9)Union Cabinet recently approved the establishment of two new AIIMS at these two states.

Ans:-Tamil Nadu and Telangana

Explanation:- The Union Cabinet approved the establishment of two new All India Institute of Medical Sciences (AIIMS) at Tamil Nadu and Telangana. Tamil Nadu's AIIMS will be set at Madurai and Telangana's will be set at Bibinagar at the cost of Rs.1,264 crore and ₹1,028 crore respectively. Each will add 100 Undergraduate (MBBS) seats and 60 B.Sc (Nursing) seats. The two new institutes will be set up under the Pradhan Mantri Swasthya Suraksha Yojana (PMSSY). Construction of new AIIMS is fully funded by the central government

10) 18th December was observed as an International Migration Day to protect the rights of migrant workers and members of their families. The theme of 2018 IMD is;

Ans:-Migration with Dignity

Explanation:-The UN's (United Nation) International Migration Day (IMD) is observed every year on 18th December to protect the rights of migrant workers and members of their families. The theme for this year's IMD is 'Migration with Dignity'. In Dec 2000, the UN General Assembly proclaimed 18 Dec as International Migrants Day after considering the large and increasing number of migrants in the world.

11)ISRO plans to Launch GSAT- 7A Communication Satellite from this space station.

Ans:- Sriharikota

Explanation:-GSLV F 11 will carry the 2250 kilogram GSAT 7A from Sriharikota at 4.10 pm on 19th December. The countdown is progressing smoothly for the launch of GSLV F 11 carrying GSAT 7A at the Space research Centre Sriharikota. The process of propellant filling of liquid strap-ons had begun. The rocket will spew out the satellite within 20 minutes after lift off from the second launch pad of the Satish Dhawan Space Centre. The rocket, GSLV F 11 is the fourth generation launch vehicle with three stages.

SSCE 8981426494/8296260082 Downloaded from - www.onlinessce.com Page 88

12)Shaan and Arijit Singh was awarded with the 'Sangeet Mahasamman' by which of the government?

Ans:-West Bengal

Explanation:-Popular singers Shaan and Arijit Singh were given the "Sangeet Mahasamman" award by the West Bengal government at the inaugural session of the Bengal music and folk culture festival held in Kolkata. Legendary singer Sandhya Mukhopadhyay gave away the awards. Leading Bengali singers like , Raghab Chattopadhyay, Manomoy Bhattacharya, Saikat Mitra, Pratik Chowdhury and music director Debojyoti Mishra were also given the 'Sangeet Mahasamman' awards. Chief Minister Mamata Banerjee inaugurated the eight-day Bangla Sangeet Mela(Bengal Music Festival) and Biswa Bangla Lok Sanskriti Utsav( Folk Culture Festival). The government has given cards to two lakh folk artistes or 'Lok Prasar Shilpi', and each of them are given Rs 1,000 as monthly allowance.

13)Name the Indian Actress who was named PETA India's 2018 Person of the Year?

Ans:-Sonam Kapoor

Explanation:-Actress Sonam Kapoor, a vegan, has been named People for the Ethical Treatment of Animals (PETA) India's Person of the Year for 2018. Sonam has been feted as she sings in praise of vegan eating and because she keeps animal skins out of her handbag line for her fashion brand Rheson, apart from taking other action for the welfare of animals.

14)India ranked ______in the World Economic Forum (WEF) gender gap index 2018.

Ans:-108

Explanation:-India has been ranked 108th in World Economic Forum (WEF) gender gap index 2018, same as 2017 while recording improvement in wage equality for similar work and fully closing its tertiary education gender gap for the first time. As per the WEF's Global Gender Gap Report 2018, Iceland topped the list having closed more than 85.8% of its overall gender gap. The Report was published for total 149 countries. Norway and Sweden were on 2nd and 3rd position, respectively.

15)Road Transport & Highways Minister Nitin Gadkari will inaugurate and lay the foundation of various projects worth over 9,500 Crore Rupees in ______.

Ans::- Arunachal Pradesh

Explanation:-Road Transport & Highways Minister Nitin Gadkari will inaugurate and lay the foundation of various projects worth over 9,500 Crore Rupees in separate events in Arunachal Pradesh. Mr. Gadkari will inaugurate bridges across Dibang and Lohit River System, including approach road between Chowkham-Digaru. He will also inaugurate the

SSCE 8981426494/8296260082 Downloaded from - www.onlinessce.com Page 89

recently two-laned sections of NH 52 B between Mahadevapura to BuriDihing and Bordumsa to Namchik.

16)Name the person who won the Mrs.India Worldwide 2018 title at the 27th annual global pageant in US.

Ans:-Mandeep Kaur Sandhu

Explanation:-In the Mrs.India Worldwide 2018 Category,India's Mandeep Kaur Sandhu, who hails from Haryana, was declared as the winner and crowned at 27th annual global pageant.Jeya Priya Pandian from Malaysia and Kavita Malhotra Pattani from USA were declared the first and second runner ups of the Mrs India Worldwide 2018. Girls andWomen's of Indian Origin from 17 Countries Participated in this 27th annual global pageant.The next edition of the Miss India Worldwide will be held in Mumbai in 2019.

17)Senior politician Tulsi Giri has passed away on 18 December. He was the Former Prime minister of;

Ans:-Nepal

Explanation:-Former Prime Minister of Nepal Tulsi Giri has passed away in Kathmandu. He was 93. Giri was suffering from liver cancer for a long time. He twice served as the prime minister of Nepal first from 1962 to 1964 and again from 1975 to 1977.

18)Who has been named as the 2018 ITF World Champion under Women's Singles category?

Ans:-Simona Halep

Explanation:-The International Tennis Federation announced 's Simona Halep as the 2018 ITF World Champion. The ITF World Champions will receive their awards at the 2019 ITF World Champions Dinner on Tuesday 4 June, in Paris, during Roland Garros. In women's Singles Category: It is the first year Halep has been recognised.Halep also enjoyed Grand Slam success in 2018.The Romanian has spent 40 weeks of the year ranked as world No.1.she also won the 2018 WTA Shot of the Year. In the Women's Doubles Category: Barbora Krejcikova and Katerina Siniakova won the award.

19)Who sets a national record by completing 176.8 km in the women's 24-hour event at New Delhi?

Ans:-Apoorva Chaudhary

Explanation:-Apoorva Chaudhary, the 28-year old ultra runner from Bijnor in Uttar Pradesh, created a national record by completing 176.8 km in the women's 24-hour event at the NEB 24 Hour Stadium Run held at New Delhi under the aegis of Athletics Federation of India. The previous record of 175.4 km was set by Meenal Kotak. Apoorva became the

SSCE 8981426494/8296260082 Downloaded from - www.onlinessce.com Page 90

fourth Indian woman to run more than 100 miles (160 km) in 24 hours after Meenal, Hema Saini (172.3 km) and Aparna Choudhary (169.2 km). In the men's 24-hour run held at the Jawaharlal Nehru Stadium at New Delhi, Pranaya Mohanty from Bengaluru completed 206.8, becoming the third Indian to have crossed the 200 km distance and the first to have achieved it in a stadium race held in India.

20)A statue of former Indian captain MS Dhoni has been unveiled at which of the following museum?

Ans:-Jaipur Wax Museum, Jaipur

Explanation:-Jaipur Wax Museum, situated at Nahargarh Fort, marked its second anniversary by unveiling a wax statue of cricketer Mahendra Singh Dhoni. Weighing 56 kg, of which 19 kg is wax and the rest is fibre, the statue is five feet and nine inches tall, and is designed by sculptor Susanta Roy.

Daily Current Affairs 20th Dec,2018

1)Which Institute launched an e-library of ancient religious and historical works in Pune?

Ans:-BORI

Explanation:-The Bhandarkar Oriental Research Institute (BORI) launched an e-library of ancient religious and historical works. BORI houses one of South Asia's largest and most invaluable agglomeration of rare manuscripts. Nearly 1,000 rare books and manuscripts in Sanskrit and its related languages are presently available for readers worldwide to savor in this first phase of digitization. Three fully-automated Zeutschel high-resolution German scanners were specially procured by the institute at a cost of Rs.15 lakh each.

2)Karnataka Bank Partners with this firm for the Home Loans.

Ans:-Paisabazaar

Explanation:-Paisabazaar.com, India's largest online marketplace for financial products, and Karnataka Bank announced a strategic partnership to offer the bank's home loan product to customers on the Paisabazaar platform. Through this partnership, consumers will now be able to choose and apply for a home loan from Karnataka Bank, directly through the Paisabazaar platform.

3)Belgium Prime Minister, who has resigned amid pressure on his government over his support for the UN global pact on migration.

Ans:-Charles Michel

SSCE 8981426494/8296260082 Downloaded from - www.onlinessce.com Page 91

Explanation:-Belgium Prime Minister Charles Michel resigned amid pressure on his government because of his support for the UN global pact on migration. Due to the migration spat, Belgium Prime Minister lost the backing of coalition partner nationalist New Flemish Alliance and opposition parties refused to support his minority government during the debate in the parliament.

4)Assam Government approved to waive off what per cent of loan of farmers of maximum upto Rs.25000 under the state's Debt Relief Scheme?

Ans:-25%

Explanation:-Assam Government spokesperson and Parliamentary Affairs Minister Chandra Mohan Patowary announced that government has approved to waive off 25 per cent of loan of farmers of maximum upto Rs.25000 under the state's Debt Relief Scheme. The state government approved the cost of Rs.600 crore to help around eight lakh farmers loan of the state. The state government will provide up to Rs 10,000 subsidy on loans taken from Kisan Credit Card (KCC).The Government has also approved an interest relief scheme for farmers to take loans at zero interest from next financial years.

5)India's largest Cancer hospital 'National Cancer Institute' was opened in ______.

Ans:-Haryana

Explanation:-The National Cancer Institute (NCI) at Haryana's Jhajjar area, opened its OPD (out-patient department) services around five years after its foundation stone was laid by the then prime minister Manmohan Singh in 2014. The hospital is expected to be fully operational by December 2020. The 710-bed hospital is being built at a cost of Rs 2, 035 crore. NCI is being headed by Dr G K Rath who is also the chief of Institute Rotary Cancer hospital at AIIMS.

6) Which is launched to improve data relay and communication links with its remote sensing earth observation satellites?

Ans:-IDRSS

Explanation:-On 16th December 2018,India launched the Indian Data Relay Satellite System (IDRSS) to improve data relay and communication links with its remote sensing earth observation satellites. The two-satellite IDRSS will maintain continuous communication link with India's remote sensing earth observation satellites. The first one is expected to be launched in 2019. The Geosynchronous Satellite Launch Vehicle Mark III (GSLV Mk III) would carry three Indian astronauts to the space in 2022.

7)Who has been awarded the UN Human Rights Prize for 2018?

Ans:-Asma Jahangir

SSCE 8981426494/8296260082 Downloaded from - www.onlinessce.com Page 92

Explanation:-Asma Jahangir, the late Pakistan lawyer, and human rights activist, has been awarded the UN Human Rights Prize for 2018. Her daughter Munizae Jahangir received the prestigious award on behalf of her mother. Jahangir was announced as one of the four winners of the quinquennial prize in October. Other awardees included women's rights activist in Tanzania, Rebeca Gyumi; activist for the rights of indigenous Brazilian communities, Joenia Wapichana, and Ireland's human rights organization, Front Line Defenders.

8)Which Indian Atomic Power Station created a world record for continuous operation for 941 days?

Ans:-Kaiga

Explanation:-A 220-MW unit of Nuclear Power Corporation of India's (NPCIL) Kaiga Atomic Power Station has created a world record for continuous operation for 941 days. The previous world record of 940 days held by Heysham 2 Unit-8 of the United Kingdom. Kaiga Atomic Energy Station is the third greatest nuclear electricity plant in India. Tarapur (1400 MW) of Maharashtra is the biggest and Rawatbhata (1180 MW) of Rajatshan is the 2nd most significant energy plant in India. It provides electric power to Karnataka, Andhra Pradesh, Kerala, Tamil Nadu and Puducherry.

9)The Union Cabinet of India approved the MoU with this country in the field of education, recently.

Ans:-Afghanistan

Explanation:-The Union cabinet approved the MoU with Afghanistan in the field of education which will allow students from that country to use courses devised in India. It will facilitate students and faculty of educational institutions in Afghanistan to register and use the SWAYAM courses. SWAYAM is a government of India programme designed to achieve access, equity and quality in education.

10)Which state government has launched 'Shiksha Setu' app to ensure better connectivity with college students?

Ans:-Haryana

Explanation:-The Haryana Government has launched 'Shiksha Setu' app to ensure better connectivity with college students. It will provide students information on attendance, fee, online admission and scholarships of all government colleges of the state. The app will provide instant updates of important notices, circulars and other programmes to students and teachers, while allowing the former to pay their fee online. During admissions, the students would be able to check the status of seats or course in every college in the state and will also get access to information such as details of scholarships and eligibility conditions among others. Apart from this, the students will also get instant information on assignments and notifications.

SSCE 8981426494/8296260082 Downloaded from - www.onlinessce.com Page 93

11)The theme of World Arabic Language Day 2018 celebrated on 18th December 2018 is;

Ans:-Arabic Language and Youth

Explanation:-World Arabic Language Day is celebrated every year on 18 December since 2012. The date coincides with the day in 1973 that the General Assembly of the United Nations adopted Arabic as the sixth official language of the Organization. "Arabic Language and Youth" is the theme of World Arabic Language Day celebrations in 2018, which are held at UNESCO Headquarters in Paris, on 18 December.

12)Indian Armed Forces inaugurate the Tri Services Cycling and Trekking Expedition in which of the following state?

Ans:-Sikkim

Explanation:-The Indian Armed Forces inaugurated Tri Services Cycling and Trekking Expedition in Sikkim. Tri Services Adventure Expedition will be conducted from December 17, 2018 to January 05, 2019. The expedition was undertaken by a Tri Services team comprising of three Officers, three warrant rank officers and thirteen soldiers from the Army, Navy and Air Force.

13)'Sangeet Kala Sangam' was organised by the Ministry of;

Ans:-Human Resource Development

Explanation:-'Sangeet Kala Sangam' was organised by Ministry of Human Resource Development on 18 December 2018. Sangeet Kala Sangam provides a national platform for the art and music teachers of Kendriya Vidyalayas and Navodaya Vidyalayas. The Kendriya Vidyalaya Sangathan (KVS) and Navoday Vidyalaya Samiti (NVS) are two organisations of the HRD Ministry.

14)This country plans to introduce a tax system named 'GAFA tax' on internet firms from 1 January 2019.

Ans:-France

Explanation:-France will introduce its own tax on large internet and technology companies from 1 January 2019. The name of the tax is 'GAFA tax' which was named after Google, Apple, Facebook, and Amazon. The aim of imposing the tax is to ensure that the global giants pay a fair share of taxes on their massive business operations in Europe.

15)Which word has been chosen by Merriam-Webster as its 2018 word of the year?

Ans:-Justice

SSCE 8981426494/8296260082 Downloaded from - www.onlinessce.com Page 94

Explanation:-Merriam-Webster has chosen "justice" as its 2018 word of the year due to the concept of justice was at the center of many of our national debates in the past year: racial justice, social justice, criminal justice, economic justice. Already the word follows "toxic" picked by Oxford Dictionaries, and "misinformation" plucked by Dictonary.com as their 2018 word of the year.

16)Name of the player who become the youngest crorepati in IPL after being bought by RCB for Rs.1.5 crore in IPL auction 2019.

Ans:- Prayas Ray Barman

Explanation:-Royal Challengers Bangalore roped in the youngster Prayas Ray Barman for Rs 1.5 Cr from a base price Rs 20 lakh. Bengal's 16-year-old leg-spinner Prayas Barman made his professional cricket debut for Bengal in September, taking 11 wickets in nine Vijay Hazare Trophy matches at an economy rate of 4.45.

17)Name of the non-banking financial company which merged with IDFC Bank.

Ans:-Capital First Ltd

Explanation:-Private sector IDFC Bank and Non-Banking Financial Company (NBFC) Capital First have announced completion of their merger to create IDFC First Bank. Capital First Founder and Chairman V Vaidyanathan has been appointed Managing Director (MD) and Chief Executive Officer (CEO) of the merged entity. IDFC Bank Founder, MD and CEO Rajiv Lall will serve as Part-Time Chairman of IDFC First Bank. It will serve 7.2 million customers through its 203 bank branches, 129 ATMs, 454 rural business correspondent centres across the country's urban and rural geographies.

18)The Ministry of External Affairs appointed Harsh Vardhan Shringla as the new Ambassador of India to ______.

Ans:-United States of America

Explanation:- The Ministry of External Affairs (MEA) appointed Harsh Vardhan Shringla as the new Ambassador of India to the United States of America. Shringla is currently serving as the High Commissioner in Bangladesh. He would be succeeding Navtej Singh Sarna.

19)India, Nepal and this country plans to form a joint task force that would allow free movement of wildlife across political boundaries and check smuggling of wildlife across the Kanchenjunga Landscape.

Ans:-Bhutan

Explanation:-The governments of India, Nepal and Bhutan are planning to form a joint task force that would allow free movement of wildlife across political boundaries and check

SSCE 8981426494/8296260082 Downloaded from - www.onlinessce.com Page 95

smuggling of wildlife across the Kanchenjunga Landscape, a trans-boundary region spread across Nepal, India and Bhutan. Earlier this month, the forest officials and representatives of non-government organisation of the three countries mulled up the proposal after visiting parts of the landscape and later held a meeting at Siliguri in north Bengal.

20)Who has been appointed as the Chairman of Capgemini India on 18 December 2018?

Ans:-Srinivas Kandula

Eplanation:-Capgemini appointed Srinivas Kandula as the Chairman of Capgemini India on 18 December 2018. Ashwin Yardi was appointed as the Chief Executive Officer of Capgemini India. Ashwin Yardi is the former Chief Operating Officer of Capgemini India. Capgemini is a French multinational professional service and business consulting corporation headquartered in Paris, France.

Daily Current Affairs 21th Dec,2018

1)Which institute fines Iffco Tokio, Royal Sundaram Rs 5 lakh?

Ans:-IRDAI

Explanation:-The Insurance Regulatory and Development Authority of India (IRDAI) imposed fine of Rs 5 lakh each on Iffco Tokio General Insurance and Royal Sundaram General Insurance for violating motor tariff claims and file & use guidelines. IRDAI also informed the companies violated File & Use guidelines and circulars issued by it from time to time advising insurers that they shall continue to use the coverage, terms & conditions, wordings, warranties, clauses and endorsements of the erstwhile tariff of classes of insurance covers until further orders.

2)A book named "Changing India" was authored by;

Ans:-Manmohan Singh

Explanation:-Former Prime Minister Manmohan Singh released his book titled "Changing India" at an event in New Delhi. In the book, the Congress leader narrated his journey from an economist to a politician and shared anecdotes from his life.

3)With which company, Ministry of Rural Development signed MoU for training rural youth for skill development?

Ans:-Maruti Suzuki

Explanation:-Ministry of Rural Development (MoRD) signed MoU with Maruti Suzuki India Ltd for training rural youth for skill development in presence of Minister of Rural Development Narendra Singh Tomar. This partnership between the government and the

SSCE 8981426494/8296260082 Downloaded from - www.onlinessce.com Page 96

leading industry from the automotive sector will provide training to atleast 5000 candidates in two years with assured placement to the rural youth of our country. Deen Dayal Upadhyaya Grameen Kaushalya Yojana (DDU-GKY) is the flagship placement linked skill- training programme under the MoRD.

4)Which state has emerged as the best-performing state in the Startup Ranking 2018?

Ans:-Gujarat

Explanation:-Gujarat has emerged as the best-performing state in the Startup Ranking 2018. The National Report on States' Startup Ranking was unveiled by Department of Industrial Policy and Promotion, Ministry of Commerce and Industry in New Delhi. Karnataka, Kerala, Odisha and Rajasthan, with a score of more than 85 percentile, have been adjudged the top performers.

5)How many schemes are announced by the Assam Government for farmers, recently?

Ans:-3

Explanation:-Assam government has announced three schemes for the welfare of farmers. The schemes are the Assam Farmer's Credit Subsidy Scheme (AFCSS), Assam Farmer's Interest Relief Scheme (AFIRS) and Assam Farmers' Incentive Scheme (AFIS). The state Finance Minister Himanta Biswa Sarma stated that all three schemes will cover five lakh farmers in the first phase. Under AFCSS, whatever amount of loan the farmers took from banks and whatever amount they repaid, 25% of the money will be reimbursed by the state government. On AFIRS, the Minister Stated that currently 3% of the 7% interest against farm loan is paid by the Central government and the remaining 4% is required to be paid by farmers. The state government has decided to pay the remaining 4% loan.

6)Which state launched Health Insurance Scheme(MHIS-IV) merging it with Ayushman Bharat scheme?

Ans:-Meghalaya

Explanation:-Chief Minister of Meghalaya Conrad K Sangma launched the Meghalaya Health Insurance Scheme(MHIS-IV) in convergence with that of the Centre's Ayushman Bharat scheme. The MHIS-IV targets to cover more than 8 lakh households offering over 2300 medical and surgical packages and up to Rs 5 lakh per family. The new scheme will be functional from February 1,2019.MHIS-IV covers all citizens in the state while the Ayushman Bharat(Centre's Scheme) targeted the bottom 40 percent of lower income families.

7)Who was appointed as the next Ambassador of India to the Republic of Djibouti?

Ans:-Shri Ashok Kumar

SSCE 8981426494/8296260082 Downloaded from - www.onlinessce.com Page 97

Explanation:-Ashok Kumar was appointed as the next Ambassador of India to the Republic of Djibouti. He is presently Joint Secretary in the Ministry of External Affairs.

8)Who launched the Integrated Government Online Training Programme and RTI Portal on Judgments in New Delhi, recently?

Ans:-Shri Jitendra Singh

Explanation:-The Union Minister of State (Independent Charge) Development of North Eastern Region (DoNER), MoS PMO, Personnel, Public Grievances & Pensions, Dr. Jitendra Singh launched the iGOT (Integrated Government Online Training Programme) developed by Department of Personnel and Training, Ministry of Personnel, Public Grievances & Pensions. The Right to Information (RTI) Portal on Judgments/Orders of Supreme Court, High Courts and CIC has been developed by the Institute of Secretariat Training and Management (ISTM), New Delhi. This Portal will provide a learning environment for all stakeholders under which a repository on the landmark cases on the RTI will be available at one place.

9)Govt forms new council National Medical Devices Promotion Council (NMDPC) to tackle patient safety and pricing on Medical Devices. How many members are there in the council?

Ans:-14

Explanation:-The central government set up a new National Medical Devices Promotion Council (NMDPC) to plug loopholes in India's regulatory framework for medical devices to ensure safety of patients, pricing and restrictions on import of pre-owned equipment. The council will operate under Department of Industrial Policy and Promotion (DIPP) in the Ministry of Commerce and Industry, and will have 14 members, including representatives from the Health Ministry and NITI Aayog. The NMDPC has also appointed the Association of Indian Medical Device Industry (AiMeD) as a member of the Council. Currently India imports 70-90% of its medical devices with an import bill of over Rs. 31,000 crore. The move to form the council was announced by Suresh Prabhu, Minister of Commerce and Industry during the WHO Global Forum on Medical Devices at Visakhapatnam.

10)Nilanjan Roy was sworn as the CFO of this company.

Ans:-Infosys

Explanation:-Infosys Ltd has appointed Bharti Airtel Ltd's global CFO Nilanjan Roy as chief financial officer, effective 1 March 2019. He replaces interim CFO Jayesh Sanghrajka, who will resume his responsibilities as deputy CFO. Roy spent 13 years with Airtel, before which he worked for 15 years with Unilever across their global operations.

11)Who among the following was appointed as the first full-time Chief Vigilance Officer (CVO) of SEBI for 3 years?

SSCE 8981426494/8296260082 Downloaded from - www.onlinessce.com Page 98

Ans:-Arti Chhabra Srivastava

Explanation:-The Prime Minister's Office (PMO) cleared the name of Arti Chhabra Srivastava as the first full-time Chief Vigilance Officer (CVO) of SEBI for an initial period of 3 years. She is a 1990-batch Indian Ordinance Factory Services officer, and her appointment was approved by the Central Vigilance Commission (CVC). CVOs are considered an extended hand of the CVC, which is tasked with detecting graft by officials holding public office. Functions of the CVO include: Mainly keeping tabs on its officials and detect and punish corruption and other malpractices, to examine existing rules and procedures of the organisation to eliminate or minimise scope for corruption and malpractices, To ensure observance of rules by SEBI officials relating to annual property returns, gifts accepted, benami transactions and regarding their relatives employed in private firms or businesses.

12)Where was the three-day conference on Soft Power helds?

Ans:-New Delhi

Explanation:-The Vice President, Shri M. Venkaiah Naidu inaugurated a three-day conference on Soft Power in New Delhi. The Center for Soft Power is the first center in India that is dedicated to the study of Indian soft power. It was established in collaboration with India Foundation.

13)Mohinder Pratap Singh was appointed as the next Ambassador of India to ______.

Ans:-Mongolia

Explanation:-Mohinder Pratap Singh was appointed as the next Ambassador of India to Mongolia. He is presently Director at Headquarters of the Ministry of External Affairs.

14)Who has been appointed the new coach of the Indian women's cricket team?

Ans:-W V Raman

Explanation:-W V Raman has been appointed the new coach of the Indian women's cricket team. Raman is a vastly-experienced coach having been in charge of Tamil Nadu and India U-19 teams in the past. W V Raman will have to now step down as coach of the Bengal Ranji Trophy team to take up the India job. Former India captain Kapil Dev and Anshuman Gaekwad, along with Shanthan Rangaswamy, who are all part of the three-member selection panel.

15)Who sets a limit for External Commercial Borrowings at 6.5% of GDP at current market prices?

Ans:-Reserve Bank of India

SSCE 8981426494/8296260082 Downloaded from - www.onlinessce.com Page 99

Explanation:-The Reserve Bank of India, in consultation with the government, has decided to have a rule-based dynamic limit for outstanding stock of External Commercial Borrowings (ECB) at 6.5% of GDP at current market prices. As per the statement based on the GDP figures, the soft limit works out to $160 billion for the current financial year. The outstanding stock of ECBs as on September 30, 2018, stands at $126.29 billion.

16)International Human Solidarity Day was observed on?

Ans:-20th December

Explanation:-International Human Solidarity Day is organized across the world on 20th December. It is a day to celebrate our unity in diversity and also a day to remind governments to respect their commitments to international agreements. The General Assembly, on 22 December 2005, by resolution 60/209 identified solidarity as one of the fundamental and universal values.

17)With which country India hold a first-ever meeting on Cultural and People-to- People Exchanges in New Delhi?

Ans:-China

Explanation:-External Affair Minister Sushma Swaraj and her Chinese counterpart Wang Yi co-chaired the first-ever meeting of India-China High-Level Mechanism on Cultural and People-to-People Exchanges in New Delhi. India and China had decided to set up people to people mechanism during the meeting between Prime Minister Narendra Modi and Chinese President Xi Jinping on the sidelines of the Shanghai Cooperation Organization Summit in China.

18)Union Minister of State for Civil Aviation Jayant Sinha launched an official mobile application for Global Aviation Summit 2019. GAS 2019 is scheduled to be held in ______.

Ans:-January 2019

Explanation:-Union Minister of State for Civil Aviation Jayant Sinha launched an official mobile application for Global Aviation Summit 2019. The Global Aviation Summit (GAS) 2019 is scheduled to be held in January 2019 in Mumbai. The application was launched in the presence of R.N. Choubey, Secretary, Ministry of Civil Aviation, Dr. Guruprasad Mohapatra, Chairman, Airports Authority of India (AAI), Dr. Shefali Juneja, Joint Secretary, Ministry of Civil Aviation, and other officials of AAI.

19)Pranab Kumar Das has been appointed as chairman for which of the following?

Ans:-CBIC

SSCE 8981426494/8296260082 Downloaded from - www.onlinessce.com Page 100

Explanation:- Senior bureaucrat Pranab Kumar Das has been appointed as chairman of the Central Board of Indirect Taxes and Customs (CBIC), the apex policy-making body for indirect taxes. Das, special secretary and member (customs) at CBIC, will succeed S Ramesh. Das, a 1983-batch IRS officer of Customs and Central Excise Cadre was appointed as a member in the Central Board of Excise and Customs (CBEC) in 2017.

Daily Current Affairs 22th Dec,2018

1)Indian player, Anup Kumar announced his retirement on 19 December 2018. He belongs to which sports?

Ans:-Kabaddi

Explanation:-Kabaddi player Anup Kumar announced his retirement on 19 December 2018. He began his international career in 2006 at the South Asian Games in Sri Lanka. He led India to victory at the 2016 Kabaddi World Cup. It was during his tenure that the Indian team won 2 gold medals at Asian Games in 2014. In season 2 of the Pro Kabaddi League, Anup captained U Mumba and led the team to victory.

2)Who among the following will lay the foundation of 'Paradip Hyderabad Pipeline Product' Project (PHPL) on 24 December 2018?

Ans:-Narendra Modi

Explanation:-The Prime Minister, Narendra Modi will lay the foundation of 'Paradip Hyderabad Pipeline Product' Project (PHPL) on 24 December 2018. He will also lay the foundation of Bokaro-Angul section of 'Jagdishpur-Haldia & Bokaro-Dhamra' Gas Pipeline Project (PM Urja Ganga) and is being built by the Gas Authority of India Limited. PHPL is being built by Indian Oil Corporation Limited.

3)How many technology centres have launched by the MSME Ministry recently?

Ans:-20

Explanation:-The government of India will set up 20 additional technology centres to help Micro, Small and Medium Enterprises (MSMEs). These centres support MSMEs by giving them access to advanced manufacturing technologies, skilling manpower and providing technical and business advisory support. Currently, 10 such centres are operational in different states of the country.

4)NASA's first flight to moon that marked its 50thanniversary is;

Ans:-Apollo 8

Explanation:-NASA's first flight to the moon Apollo 8 is marking its 50th anniversary. On Dec. 21st, 1968, three men flew to the moon for the first time in human history. Apollo 8

SSCE 8981426494/8296260082 Downloaded from - www.onlinessce.com Page 101

commander Frank Borman, Jim Lovell and Bill Anders orbited the moon on Christmas Eve, reading from the Book of Genesis. To this day, the mission is considered NASA's boldest and perhaps most dangerous undertaking ever. NASA whipped it together in four months, in order to beat the Soviets to the moon.

5)Which ministry developed an Integrated Government Online Training Programme along with the Department of Personnel and Training?

Ans:-Ministry of Personnel, Public Grievances and Pensions

Explanation:-'iGOT' (Integrated Government Online Training Programme) was launched on 20 December 2018. It was developed by Department of Personnel and Training, Ministry of Personnel, Public Grievances & Pensions. It will augment the existing training mechanism, provided to government servants, with online module-based training coupled with certification.

6)For which country, US has regulator Federation of Aviation Administration (FAA) retained the highest aviation safety rank?

Ans:-India

Explanation:-US regulator Federation of Aviation Administration (FAA) retained the highest aviation safety ranking for India. According to FAA, India's International Aviation Safety Assessment (IASA) rating remains 'Category 1'. This ranking was based on an audit of the Directorate General of Civil Aviation (DGCA) in July 2018.

7)Which of the following outdoor mural got into Guinness World Records?

Ans:-Dreary grain silo

Explanation:-A dreary grain silo that was transformed into an enormous colourful artwork in South Korea has been named by Guinness World Records as the largest outdoor mural in the world. The painting depicts a young boy's journey into adulthood covers the outside of giant storage containers in the port city of Incheon takes up 23,688 sq m. The mural was commissioned by the city's government and port authority. It aims at improving the negative view of aged industrial facilities. Twenty-two artists used more than 850,000 litres of paint to tell the story which reflects the seasons and resembles 16 individual book covers, at a cost of 550 million won.

8)Name the company who has partnered with self-drive rental car firm Zoomcar to offer the electric version in Pune?

Ans:-Tata Motors

Explanation:-Tata Motors has partnered with self-drive rental car firm Zoomcar to offer the electric version of its compact sedan Tigor in Pune city as part of the shared mobility plans.

SSCE 8981426494/8296260082 Downloaded from - www.onlinessce.com Page 102

Under the partnership, Zoomcar expects to deploy 500 electric vehicles in over 20 cities with Tata Motors in the next one year. Tata Motors is committed to the Centre's vision of driving electric mobility in the country.

9)How many workers have been selected for PM Shram Awards 2017?

Ans:-40

Explanation:-The Government of India has decided to confer the Prime Minister's Shram Awards for the year 2017 to be awarded to 40 workers. They are employed in the Departmental Undertakings & Public Sector Undertakings of the Central and State Governments and Private Sector Units. The awards are in recognition of their distinguished performances in the field of productivity.

10)President's rule is set to be imposed on which state, based on union cabinet's ruling and the governor's report?

Ans:-Jammu and Kashmir

Explanation:-Six months after the Governor's rule in Jammu and Kashmir, President's rule is set to be imposed on J&K, as per the union cabinet's ruling and the governor's (Satya Pal Singh) report, after a span of 22 years. President's rule was first imposed in the state in 1986, whereas the previous instance of President Rule was in 1996, 22 years ago. Under Article 92 of the Jammu and Kashmir Constitution, the state will be under the governor's rule for 6 months under political crisis situations. The Governor has to dissolve the Legislative Assembly after six months after which it directly comes under President's rule for another six months after which elections have to be held in the state. President's rule can be extended by another six months. Also, no Presidential proclamation shall in any case remain in force for more than three years except for the intervention of Election Commission of India.

11)Name the app launched by the Civil aviation ministry and Airport Authority of India (AAI) ahead of Global Aviation Summit.

Ans:-GAS 2019

Explanation:-The Civil aviation ministry along with the Airport Authority of India (AAI) launched an app named 'GAS 2019'. This app will enable users to interact with the co- delegates and speakers besides providing a platform for networking through personal and group chats. This was launched ahead of the global aviation summit to be held in Mumbai in January 2019. The Global Aviation Summit would take place on January 15-16, 2019 at Mumbai.

12)All-India annual conference of the state Directors General and Inspectors General of Police will be held in ______.

Ans:-Gujarat

SSCE 8981426494/8296260082 Downloaded from - www.onlinessce.com Page 103

Explanation:-The All-India annual conference of the state Directors General and Inspectors General of Police will be held from 20 to 22 December 2018 in Gujarat. The conference will be held in the presence of Union Home Minister Rajnath Singh. The agenda of the annual conference is militancy in Jammu and Kashmir, cross-border terrorism, and attempts to radicalise youth.

13)What is the name of the new autonomous body that has been recommended by NITI Aayog's 'Strategy for New India @75'?

Ans:-Arbitration Council of India

Explanation:-The NITI Aayog unveiled its comprehensive national Strategy for New India dubbed as 'Strategy for New India @75', defining clear objectives for 2022-23. The document aims for establishing a New India by 2022 and to propel India towards a USD 5 trillion economy by 2030. It has set up a new autonomous body, viz., the Arbitration Council of India to make the arbitration process cost effective and speedy, and to pre-empt the need for court intervention.

14)Where was the International Training Centre for Operational Oceanography (ITCOocean) Complex inagurated?

Ans:-Hyderabad

Explanation:-International Training Centre for Operational Oceanography (ITCOocean) Complex will be inaugurated in Hyderabad on 22 December 2018. It was established by the Ministry of Earth Sciences. It will provide training to scientists/researchers/government officers/disaster managers/decision makers in the areas of ocean science and management.

15)This state plans to set up Herbal Corporation to promote Ayurveda and herbal medicines in the state.

Ans:-Haryana

Explanation:-Haryana's Chief Minister Manohar Lal Khattar announced the setting up of the 'Haryana Herbal Corporation'. The aim of the corporation is to promote Ayurveda and herbal medicines in the state. It will also act as a catalyst for developing area of the district as an organic cluster and setting up of a world-class nursery for the herbal plant.

16)Which International organization has officially adopted the global framework on International migration?

Ans:-UNGA

Explanation:-The UN General Assembly (UNGA) has officially adopted the Global Compact for Safe, Orderly and Regular Migration. The document is the first-ever negotiated global framework on a common approach to international migration in all its dimensions. It

SSCE 8981426494/8296260082 Downloaded from - www.onlinessce.com Page 104

is meant to address issues that concern the world's 258 million people on the move and countries of origin, transit and destination. The non-binding agreement was adopted by the General Assembly with 152 votes in favour in Marrakesh, Morocco on 10th December 2018. India voted in favour of the resolution while, the Czech Republic, Hungary, Israel, Poland, and US voted against it. The document reaffirms the foundational principles of global community, including national sovereignty and universal human rights. Though non-legally binding, the Compact is the outcome of a long negotiation process and provides a strong platform for cooperation on migration, drawing on best practices and international law.

17)Qualcomm won a patent dispute case against Apple, which could lead to a ban on sales of iPhones in which of the following country?

Ans:-Germany

Explanation:-A German court found in favour of US chipmaker Qualcomm in a patent dispute case against Apple, which could lead to a ban on sales of iPhones in Germany. The ruling affects iPhone 7 through iPhone X models, which contain a particular component that violated a Qualcomm patent.

18)The first US State which have female majority in its legislature is;

Ans:-Nevada

Explanation:-Nevada became the first state in the US with an overall female majority in its legislature on 18 December 2018. The appointments of the Democrats Rochelle Thuy Nguyen and Beatrice Angela Duran to two Las Vegas-area legislative seats give women 51% of the 63 seats in the legislature. No state has previously had a female-majority or even a 50%-female legislature.

19)National Balrang-2018 was started in;

Ans:-Bhopal

Explanation:-To promote the artistic talent of the students, the National Balrang-2018 has started at the Rashtriya Manav Sangrahalya (IGRMS) in Bhopal on 19th of December. The 3-day cultural festival is being organised by the School Education Department of Madhya Pradesh in collaboration with the IGRMS. Its purpose is to introduce children with various cultural diversities and traditional lifestyles so as to instill the spirit of social harmony, human values, and brotherhood. 'Balrang' is a unique programme that has been held in Bhopal since the past 23 years. As many as 520 students from various schools in 18 states and two union territories will present folk dances, songs and other cultural programmes.

20)Which company conferred with National Energy Conservation Award 2018 recently?

SSCE 8981426494/8296260082 Downloaded from - www.onlinessce.com Page 105

Ans:-C.R.I. pumps

Explanation:-C.R.I. pumps, one of the Leading Manufacturer of energy efficient pumps won the National Energy Conservation Award 2018 in the pumps category for the fourth time. C.R.I. Group joint Managing Director, Mr. G. Selvaraj receives the award. C.R.I. is the highest contributor in the country for the projects of EESL (Energy Efficiency Services Limited) to replace the old inefficient pumps with new 5 star rated smart pumps.

Daily Current Affairs 23th Dec,2018

1) Name of the Genetically Modified (GM) indoor plant to remove pollutants inside the house.

Ans:-Devil's ivy

Explanation:-Researchers have Genetically Modified (GM) a common indoor plant, golden pothos or Devil's ivy (Epipremnum aureum), to remove pollutants inside the house. It has been modified to produce a liver enzyme called cytochrome p450 2e1, which is taken from rabbits, that breaks down a wide range of pollutants. The researchers put the modified plant and normal pothos ivy in glass tubes and added either benzene or chloroform gas into each tube. They found that the concentration of either gas did not change for normal plants. However, for the modified plants, the concentration of chloroform dropped by 82%, and it was almost undetectable by day six.

2)Which Indian city will host the first ever India China Film Festival strengthening the cultural understanding amidst?

Ans:-New Delhi

Explanation:- A three day India China Film Festival is set to begin in New Delhi from 22nd of December this year culminating on 24th of December. In order to strengthen and deepen cultural understanding between the two countries, the film festival is being organized during the first India-China High-Level Mechanism on Cultural and People-to-People Exchanges. Jackie Chan's 2012 action movie "CZ12" will open the India-China Film Festival here. The three-day Festival will be organized at Siri Fort Auditorium II, New Delhi by the Directorate of Film Festivals, Ministry of Information & Broadcasting and the Indian Government. The duo cultural and people-to-people exchanges will probably be encouraged by way of this initiative. A total of seven films including four Chinese and three Indian Films will be screened during the Festival. The opening film of the Festival will be 'CZ12' also known as 'Chinese Zodiac'.

3)NFAI and ______sign MoU to Transfer the land for the construction of storage facility for films and film material.

Ans:-FTII

SSCE 8981426494/8296260082 Downloaded from - www.onlinessce.com Page 106

Explanation:-National Film Archive of India (NFAI) and Film & Television Institute of India (FTII) signed Memorandum of Understanding (MoU) for transfer of three acres of land for the construction of storage facility for films and film material. The land belonging to FTII at their Kothrud campus in Pune will be transferred to NFAI for this purpose.

4)What is the position of India in the Stock market?

Ans:- 7th

Explanation:-India achieved another milestone when Indian Stock market overtakes Germany for the first time in seven years to become the world's seventh largest stock market. This move reflects India's positive returns this year as companies dependency on domestic demand enabled them to avoid meltdown in other emerging markets spurred due to U.S. - China trade war. South Asian Giant, India is projected to grow at 7.5 percent in 2018 and 7.3 percent in 2019 whereas Germany managed to achieve a growth rate of only 1.6 percent in 2018. After the exit of United Kingdom from European Union, now European Union has only one country in top seven economies of the world - France.

5)Sahitya Akademi award winner Prabhanjan passed away recently. He belongs to which state?

Ans:-Tamil Nadu

Explanation:-Noted Tamil writer and Sahitya Akademi award winner Prabhanjan died due to Cancer at the age of 83.Prabhanjan started his career as a Tamil Teacher and got Sahitya Akademi Award for his historic novel "Vaanam Vasapadum" in 1995.

6)Which country tops in the FIFA global ranking 2018?

Ans:-Belgium

Explanation:-Federation Internationale de Football Association (FIFA) released the Global football ranking topped by Belgium who have 1727 points as per FIFA calculations. So Belgium is going to end the year as No. 1 ahead of World Champion France. France ranked 2nd with 1726 points and Brazil ranked third with 1676 points. India ranked at 97th Position in the latest FIFA ranking released on 20th December 2018.

7)The CM of this state opened a World Herbal Forest Project in a collaboration of Patanjali.

Ans:-Haryana

Explanation:-Haryana Chief Minister Manohar Lal Khattar inaugurated the World Herbal Forest Project, a collaboration of Patanjali and Haryana Government, at Morni Hill, near Chandigarh. The project covers over 52,000 acres of land in Morni Hills and is aimed at developing a forest of medicinal plants which will provide a huge boost to the Ayurvedic

SSCE 8981426494/8296260082 Downloaded from - www.onlinessce.com Page 107

industry across the country. Yoga guru and Patanjali CEO Acharya Balkrishna were also present during the inaugural function. About 25 different varieties of plants found in the country will be housed in this forest. 50,000 plants each of 20 medicinal varieties are being sown in the first phase of the project.

8)Which app has been launched by the Ministry of Aviation for Global Aviation Summit 2019?

Ans:-GAS-2019

Explanation:-The Ministry of Aviation along with the Airport Authority of India (AAI) has launched an official mobile app "GAS-2019" for the Global Aviation Summit (GAS) for 2019, which is scheduled to be held in Mumbai in January 2019. The app enables users to interact with the co-delegates and speakers besides providing a platform for networking through personal and group chats. The summit is being hosted by the civil aviation ministry along with the AAI and industry lobby FICCI.

9)Which ministry launched the "Asiatic Lion Conservation Project" to protect and conserve the population of Asiatic Lion?

Ans:-Ministry of Environment, Forest and Climate Change

Explanation:-The Union Ministry of Environment, Forest and Climate Change launched the "Asiatic Lion Conservation Project" to protect and conserve the population of Asiatic Lion and its associated ecosystem. The Asiatic Lion Conservation Project is aimed at the conservation and recovery of Asiatic Lion with the help of up to date techniques, instruments, regular scientific research studies, disease management, modern surveillance and patrolling techniques.

10)On which date, the National Mathematics Day was observed?

Ans:-December 22

Explanation:-The National Mathematics Day (NMD) is celebrated every year in India on December 22 to commemorate the birth anniversary of great mathematical genius Srinivasa Ramanujan. The day also recognizes his contribution to the world of mathematics, which includes mathematical analysis, number theory, infinite series, and continued fractions. The NMD was declared by the then Prime Minister Manmohan Singh in 2012 to mark the 125th birth anniversary of Ramanujan. Born in 1887 in Erode, Madras Presidency (now Tamil Nadu), Ramanujan's affinity for Mathematics was such that he failed in college exams because of his negligence for non-mathematical subjects. However, his talents were recognized by a mathematician at Madras Port Trust were Ramanujan had started working as a clerk in 1912. In 1917, Ramanujan was elected to be a member of the London Mathematical Society.

SSCE 8981426494/8296260082 Downloaded from - www.onlinessce.com Page 108

11) Name of the Indian CM who has been conferred the Skoch Chief Minister of the Year Award for 2018?

Ans:-Mamata Banerjee

Explanation:-Mamata Banerjee, the Chief Minister of West Bengal , has been conferred the Skoch Chief Minister of the Year Award for 2018 for her contribution in the overall development of the state and emerging as the best chief minister in the country. Banerjee has been conferred with the prestigious award on the basis of the state's performance in the field of governance, culture, finance, urban and rural development. In 2018, The state has bagged 31 Skoch Awards for its performance across different fields besides implementing unique projects. As per Skoch, West Bengal has also won the top position in terms of the number of man-days created and the amount spent in paying people for their work in the 100 days' work scheme. Besides Banerjee, the Skoch Group has also awarded Andhra Pradesh Chief Minister N Chandrababu Naidu with the 'SKOCH Sprinter of the Year', and Commerce and Industry and civil aviation minister Suresh Prabhu as the 'Steady Reformer'.

12) This country launched the first satellite to provide space-based broadband as part of Hongyun project.

Ans:-China

Explanation:-China launched its first communication satellite, from a Long March 11 carrier rocket, to provide broadband internet services worldwide to rival Google and other firms. It is the first in the Hongyun project planned by China Aerospace Science and Industry Corp (CASIC). It weighs 247 kilogrammes and works in a sun-synchronous orbit about 1,100 kilometres above earth. Powered by solar arrays, the satellite would verify basic designs of Hongyun satellite and demonstrating low-orbit broadband communications technologies with a life span of more than 1 year.

13)For what purpose, the Union Government has launched iGOT programme?

Ans:-Training program for Govt employees

Explanation:-Union Minister Dr Jitendra Singh launched the iGOT (Integrated Government Online Training Programme) for government employees. The iGOT will augment the existing training mechanism with online module-based training coupled with certification. The programme, developed by the Department of Personnel and Training (DoPT), Ministry of Personnel, will make training inputs available to a government servant on-site and on flexitime basis. The initiative aims at "competent civil services for good governance". Dr. Singh also launched Right to Information (RTI) Portal on Judgments/Orders of Supreme Court, High Courts and CIC that will provide a learning environment for all stakeholders under which a repository on the landmark cases on the RTI will be available at one place. He also launched a hybrid course on administrative laws developed by the DoPT in collaboration with premier law institute National Law School of India University (NLSIU), Bengaluru. SSCE 8981426494/8296260082 Downloaded from - www.onlinessce.com Page 109

14)Which metro plans to launch mobile app to book a ride for travelling to areas around stations?

Ans:-Chennai Metro Rail Limited

Explanation:-Chennai Metro Rail Limited (CMRL) is to soon offer its commuters a facility to book a ride through its mobile app, for travelling to areas around stations. For this project, CMRL has tied up with a firm to offer the service. The firm's icon for this service will be created in the Chennai Metro Rail mobile app and when commuters touch the icon, it will direct them to the firm's vehicle booking page and they can hire a car.

Daily Current Affairs 24th Dec,2018

1)How many IT companies has shortlisted by the Reserve Bank of India to set up a wide-based digital Public Credit Registry (PCR) for capturing details of all borrowers and wilful defaulters?

Ans:-6

Explanation:-The Reserve Bank of India has shortlisted six major IT companies, including TCS, Wipro and IBM India, to set up a wide-based digital Public Credit Registry (PCR) for capturing details of all borrowers and wilful defaulters. The proposed PCR will also include data from entities like market regulator Sebi, the corporate affairs ministry, Goods and Service Tax Network (GSTN) and the Insolvency and Bankruptcy Board of India (IBBI) to enable banks and financial institutions to get a 360-degree profile of the existing as well as prospective borrowers on a real-time basis.

2)Prime Minister Narendra Modi releases a 100 rupees commemorative coin in the honour of;

Ans:-Atal Bihari Vajpayee

Explanation:-Prime Minister Narendra Modi releases a 100 rupees commemorative coin in honour of former Prime Minister and Bharat Ratna Atal Bihari Vajpayee. Atal Bihari Vajpayee's birth anniversary, which falls on 25th December, is celebrated as 'Good Governance Day'. He was conferred with the Bharat Ratna was in 2014. PM Modi stated that the voice of Atal Ji was the voice of common people of India for four decades.

3)How many Indian news firms selected for Google News Initiative Funding?

Ans:- 10

Exolanation:-Google has selected 87 news organisations from 23 countries for funding under its Global News Initiative (GNI). Of these 87 organisations selected, 10 are from India. The funding aims at helping the organisations in building video capabilities and experiment with new formats for video journalism. The news firms selected from India are

SSCE 8981426494/8296260082 Downloaded from - www.onlinessce.com Page 110

Factly, Asianet News Media and Entertainment, NDTV, India Today, Bharatiya Digital Party, Nyooz and Video Volunteers. Google will aid these organisations in expanding newsroom video operations and trying out new ways of reporting news through video from reaching younger audiences online to exploring live and fact-checking formats.

4)Nirmala Sitharaman inaugurated the Indian Navy's Information Fusion Centre (IFC) in Which Indian City?

Ans:-Gurugram

Explanation:-Defence Minister Nirmala Sitharaman inaugurated the Indian Navy's Information Fusion Centre (IFC) in Gurugram that aims to collaborate with partner countries and multi-national agencies to develop maritime awareness and share information on vessels. The IFC has been established at the Navy's Information Management and Analysis Centre (IMAC) at Gurugram which is the single point centre interlinking all the coastal radar chains to generate a seamless real-time picture of the nearly 7,500-km long coastline.

5)Which state launched a KALIA scheme and announced a Rs10,000 crore scheme for the overall development of farmers?

Ans:-Odisha

Explanation:-Odisha CM Naveen Patnaik announced a Rs10,000 crore scheme for the overall development of farmers. Named Krushak Assistance for Livelihood and Income Augmentation (KALIA), the scheme will provide farmers with ₹10,000 a year at the rate of Rs5,000 each for Kharif and Rabi season. Additionally, crop loans of up to Rs50,000 will be interest-free under the scheme.

6)Which country stood at 12th rank in the "misperception index"?

Ans:-India

Explanation:-Indians placed at 12th rank in the "misperception index" according to the MORI Perils of Perception Survey 2018, done by Ipsos India, an independent market research company. It behind several countries like Italy, Singapore, Hong Kong, South Korea, Chile, and Argentina, among others. Thailand, Mexico, Turkey, Malaysia, and Brazil were ranked first in five and performed poorer than India. "misperception index" is based on questions which asked to people and it revealed the wide chasm between what people perceived and what was real.

7)Indian Railways bagged ______awards at the 'National Energy Conservation Awards (NECA) 2018' held in New Delhi.

Ans:-17

SSCE 8981426494/8296260082 Downloaded from - www.onlinessce.com Page 111

Explanation:-Indian Railways bagged 17 awards at the 'National Energy Conservation Awards (NECA) 2018' held in New Delhi. Under 'transport' category, Indian Railways obtained 10 awards for the subsector railway station. Vidisha railway station in Madhya Pradesh won the first prize. Under 'building' category, Indian Railways got three awards for subsector Railway hospitals. Railway hospitals of Izzatnagar division won the first prize. Under 'institution' category, Indian Railways bagged four awards for subsector state PWD, CPWD and PHED. Passenger reservation system complex building of Secunderabad division won the first prize. The awards are given by the Bureau of Energy Efficiency under the Ministry of Power to various institutions for adopting energy efficiency measures.

8)Assamese singer Dipali Borthakur Passes Away on Dec 24. She was popularly known as;

Ans:-Nightingale of Assam

Explanation:-Legendary Assamese singer Dipali Borthakur popularly referred to as the "Nightingale of Assam" passed away. She was 77. Borthakur had been suffering from a motor neuron disease for the last four decades. For her outstanding contributions to the field of arts, Borthakur was honored with the Padma Shri award in 1998. The Silipi Bota conferred on her by the state government in 2010 and the Aideu Handique Silpi Award in 2012 are among the numerous accolades she received.

9)15th Global SME Business Summit was held in ______.

Ans:-New Delhi

Explanation:-The 15th edition of a two day Global SME Business Summit was organised by the Ministry of MSME and Confederation of Indian Industry (CII) in association with Government e-marketplace in New Delhi. The theme of the event was 'Building Partnerships through Global Value Chains'. The agenda of the summit was to bring together renowned speakers with unbounded experience and expertise, a global audience, and various national and international SME stakeholders to integrate Indian MSMEs into the Global Value Chains (GVCs).

10)Which team won the 2018 Club World Cup and record fourth in total?

Ans:-Team Wellington

Explanation:-Spain's Real Madrid won the Club World Cup for a third consecutive year, and record fourth in total, by beating Abu Dhabi side Al Ain in the final. Real now hold the record for most victories in the competition, which was first held in 2000, moving one clear of Spanish rivals Barcelona.

11)Who received South Indian Education Society (SIES) recently?

Ans:-Shri Manmohan Singh

SSCE 8981426494/8296260082 Downloaded from - www.onlinessce.com Page 112

Explanation:-Former Prime Minister Dr Manmohan Singh received South Indian Education Society (SIES) Sri Chandrasekharendra Saraswati National Eminence Award as part of the Kanchi Mahaswami festival.He was awarded for his contribution to the field of public leadership. Professor Dr , Niti Aayog member Dr V K Saraswat and Swami Chidanand Saraswati were awarded for their work in the fields of community leadership, science and technology and social thinkers, respectively.

12)When was the National Farmer's Day observed?

Ans:-23rd December

Explanation:-National Farmer's Day (Kisan Diwas) 2018 was celebrated all over India on 23rd of December. It is celebrated in the honour of the former Prime Minister of India Chaudhary Charan Singh with the focus on the upliftment of the status of farmers. In Lucknow, a program was organised by Yogi Adityanath, Chief Minister of Uttar Pradesh. He also unveiled a statue of Sri Chaudhary Charan Singh in Ghaziabad and announce the allocation of Rs. 325 crore for the development of the city and rural areas.

13)Which was named as new capital of Gitega?

Ans:-Burundi

Explanation:-Burundi confirmed the small central city of Gitega the country's new political capital.

14)India successfully test-fired its nuclear-capable long-range ballistic missile Agni-IV, with a strike range of;

Ans:-4,000 km

Explanation:-India successfully test-fired its nuclear-capable long-range ballistic missile Agni-IV, with a strike range of 4,000 km, as part of a user trial by the Army. The strategic surface-to-surface Agni-IV missile was flight tested from launch complex-4 of the Integrated Test Range (ITR) at Dr Abdul Kalam Island, earlier known as Wheeler Island. This was the 7th trial of Agni-IV missile. The last trial conducted by the strategic force command (SFC) of the Indian Army from the same base on January 2, 2018 was successful.

15)Which state Assembly passes Anti-mob Violence Bill recently?

Ans:-Manipur

Explanation:-The Manipur Assembly has passed a bill which recommends life imprisonment for those involved in mob violence if it results in the death of a person. Chief Minister N Biren Singh, who also holds charge of the Home department, moved the 'The Manipur Protection from Mob Violence Bill, 2018' in the state Assembly. The bill was passed by the Assembly unanimously. It recommends a rigorous life imprisonment term if

SSCE 8981426494/8296260082 Downloaded from - www.onlinessce.com Page 113

the crime results in the death of a victim. The bill was introduced in the Assembly in the wake of incidents of mob lynchings in the state.

16)Who amoung the following has been elected as the president of the Archery Association of India?

Ans:-B P Rao

Explanation:-B P Rao has been elected as the president of the Archery Association of India in the polls held under a High Court-appointed administrator. Rao, who represented Archery Association of Assam, beat Rupak Debroy of Tripura at the elections held at the Jawaharlal Nehru Stadium in New Delhi.A retired IAS officer of Assam cadre, Rao earlier served as vice president of the archery association of the northeastern state. He was also earlier associated with Sports Authority of India in various capacities.

17)Name the Indian cyclist who becomes the fastest Asian to cycle the globe?

Ans:-Vedangi Kulkarni

Explanation:-The 20-year old Indian woman Vedangi Kulkarni becomes the fastest Asian to cycle the globe. She had completed 29,000 kilometres' distance required to qualify as bicycling across the globe. Vedangi spent 159 days peddling up to 300 kilometers a day in 14 countries. Starting off from Perth in July, she will now be flying back to the Australian city to complete the record by cycling a 15-kilometre distance to reach the same place from where she started. Vedangi hails from Pune.

18)Which country launched its first communication satellite of the Hongyun project?

Ans:-China

Explanation:-China launched its first communication satellite, from a Long March 11 carrier rocket, to provide broadband internet services worldwide to rival Google and other firms. It is the first in the Hongyun project planned by China Aerospace Science and Industry Corp (CASIC). It weighs 247 kilogrammes and works in a sun-synchronous orbit about 1,100 kilometres above earth. Powered by solar arrays, the satellite would verify basic designs of Hongyun satellite and demonstrating low-orbit broadband communications technologies with a life span of more than 1 year.

Daily Current Affairs 25th Dec,2018

1)According to the National Crime Records Bureau, This state tops in atrocities by police.

Ans:-Uttar Pradesh

SSCE 8981426494/8296260082 Downloaded from - www.onlinessce.com Page 114

Explanation:-According to the National Crime Records Bureau (NCRB), Uttar Pradesh registered 236 cases against police personnel for human rights violations between 2014 and 2016, the highest in the country's total of 411 such cases during the same period. Uttar Pradesh was followed by Delhi where 63 cases of atrocities by police personnel were registered in the same period. The cases of human rights violations by police personnel include disappearance of persons, illegal detention or arrests, fake encounter killings, extortion, assault on women with intent to outrage her modesty, atrocities on scheduled castes, scheduled tribes, torture among others.

2)Who launched the First Global Positioning System (GPS) III satellite nicknamed as Vespucci US Military Satellite?

Ans:-SpaceX

Explanation:-SpaceX launched the U.S. Air Force's newest 'GPS III' satellite on a Falcon 9 rocket from Cape Canaveral Air Force Station in Florida, USA. The solar-powered spacecraft, dubbed "Vespucci," is the company's first official "National Security Space" mission. Vespucci's nickname honours Italian cartographer and explorer Amerigo Vespucci, after whom North and South America were named. It was competed through the Air Force's Evolved Expendable Launch Vehicle program. The successful GPS III SV01 mission marked SpaceX's 21st and final launch of 2018, beating the company's 2017 flight rate of 18 launches.

3)Veteran singer died at the age of 91. He belongs to the state of;

Ans:-West Bengal

Explanation:-Veteran Rabindra Sangeet singer Dwijen Mukhopadhyay died at his residence in Salt Lake,West Bengal following various age-related ailments at the age of 91. Dwijen Mukhopadhyay was known for his rendition of 'Jagoo Durga' on All India Radio and he was honoured with Padmabhushan and Banga Bibhusan. He was born in 1927 and recorded over 1000 songs of Tagore during his lifetime.

4)Prime Minister Narendra Modi released a stamp and a coin to commemorate the Paika rebellion of 1817 in which of the following state?

Ans:-Odisha

Explanation:-On 24 December 2018, Prime Minister Narendra Modi released a stamp and a coin to commemorate the Paika rebellion of 1817 in Odisha. The Paika Rebellion was the first war of Independence against the British. In the early 1800s, the farming community of Odisha had risen in rebellion against British's revenue model. The rebellion of 1817 came to be known as Paika Bidroh.

SSCE 8981426494/8296260082 Downloaded from - www.onlinessce.com Page 115

5)National Consumer Day was celebrated on 24th December of Every Year. It was celebrated with the theme of;

Ans:-Timely Disposal of Consumer Complaints

Explanation:-Every Year 24th December is observed as National Consumer Day with a specific theme in India. This year the National Consumer Day is celebrated with the theme "Timely Disposal of Consumer Complaints". On this day the Consumer Protection Act, 1986 had received the assent of the president. The enactment of this Act is considered as a historic milestone in the consumer movement in the country.

6)Name the new exotic planet discovered by researchers outside our solar system in the constellation Cassiopeia.

Ans:-HD219134 b

Explanation:-21 light years away on december 22, 2018 in the constellation Cassiopeia, a shimmering "super-Earth", five times the mass of Earth, dubbed as HD219134 b was discovered. Unlike the Earth, however, it most likely does not have a massive core of iron, but is rich in calcium and aluminium alongside magnesium and silicon. It shimmers red to blue like rubies and sapphires, because these gemstones are aluminium oxides which are common on the . According to the researchers it is one of three candidates likely to belong to a new exotic class of . The other two exoplanets studied were 55 Cancri e and WASP-47 e. The study was published in the journal Monthly Notices of the Royal Astronomical Society: Letters.

7)Where was the 6th edition of Yuva Natya Samaroh, a theatre festival of five young directors held?

Ans:-New Delhi

Explanation:- Sahitya Kala Parishad, Department of Art, Culture and Languages, Government of Delhi, organized the sixth edition of five-day long Yuva Natya Samaroh at Kamani Auditorium in New Delhi. The event will conclude on 26th December. It is a theatre festival of five young directors with some of the best modern, mythological as well as Parsi style drama. The festival will start with the famous and well-discussed story by Mithileshwar titled Babuji and directed by Rajesh Singh while the Parsi play Safed Khoon directed by Samiruddin (Javed Sameer), will be presented on the last day of the festival.

8)Who will replace United States Defence Secretary Jim Mattis as acting Pentagon chief?

Ans:-Patrick Shanahan

Explanation:-United States President Donald Trump announced through a tweet that Defence Secretary Jim Mattis will be replaced by his deputy Patrick Shanahan as acting

SSCE 8981426494/8296260082 Downloaded from - www.onlinessce.com Page 116

Pentagon chief on January 1, 2019 the date which is two months earlier than planned. Earlier Jim Mattis, had announced his resignation on 21st December 2018 due to strong policy differences with Mr Trump especially on Trump's Syria troop with drawl move but offered to stay on for two months in order to facilitate a smooth transition. Mr Shanahan, is a former executive at the aerospace giant Boeing, joined the Pentagon in July 2017 after Mr Trump nominated him and was reportedly a vocal supporter of the president's plan to establish a sixth branch of the armed forces, known as the "space force".

9)Where was the statue of Indian farmer's leader and 5th Prime Minister of India Chaudhary Charan Singh, inaugurated on National Farmer's Day?

Ans:-Ghaziabad, Uttar Pradesh

Explanation:-National Farmer's Day (NFD) was celebrated throughout the country to acknowledge the role of farmers in India's socio-economic development and to mark the birth anniversary of Indian farmer's leader and 5th Prime Minister of India Shri Chaudhary Charan Singh. This day which is also observed as KisanDiwas is celebrated across the nation every year with the focus on the upliftment of the status of farmers. On this occasion Yogi Adityanath, Chief Minister of Uttar Pradesh unveiled a statue of Sri Chaudhary Charan Singh in Ghaziabad, Uttar Pradesh and addressed a public meeting and announced the allocation of Rs. 325 crore for the development of the city and rural areas.

10)ICICI Bank has projected the GDP (Gross Domestic Product) to increase to what percent in 2019-20 and 7.2% in 2018-19?

Ans:-7.4 %

Explanation:-ICICI Bank projected the GDP to increase to 7.4% in 2019-20. Its growth forecast for 2018-2019 is at 7.2 mainly due to slow consumption. It states that one of the biggest factors restricting growth will be the real estate and small scale units as they are yet to recover from effects of GST and demonetisation.

11)Punjab National Bank and this state launched a special card called PNB Rupay card, for Kumbh Mela 2019.

Ans:-Uttar Pradesh

Explanation:-State-owned Punjab National Bank (PNB) in collaboration with Uttar Pradesh government launched a special card called PNB Rupay card, for Kumbh Mela 2019. It is aimed to create a model for digitization at this edition of Kumbh Mela for convenient and hassle-free transactions for the participating 12 crore devotees. This product can be used even in absence of internet.

12)India's first music museum will be set up in ______, Tamilnadu.

Ans:-Thiruvaiyaru

SSCE 8981426494/8296260082 Downloaded from - www.onlinessce.com Page 117

Explanation:-The State Minister for Culture and Tamil Development Shri K Pandiarajan announced setting up of the country's first music museum in Thiruvaiyaru, Tamilnadu. The venue chosen for the project is the birth place of Saint Tyagaraja, one of the Trinities of Carnatic music. The museum will be set up with assistance from the Central government.

13)Andhra Pradesh, Himachal Pradesh and ______were the top three performing states under the maternity benefit programme Pradhan Mantri Matru Vandana Yojana (PMMVY).

Ans:-Madhya Pradesh

Explanation:-Andhra Pradesh, Himachal Pradesh and Madhya Pradesh were the top three performing states under the maternity benefit programme Pradhan Mantri Matru Vandana Yojana (PMMVY). Minister of State for the Women and Child Development Ministry Virendra Kumar stated PMMVY is a centrally-sponsored scheme under which the cost- sharing ratio between the Centre, states and the Union Territories with the legislation is 60:40. For the eight northeastern states and the three Himalayan states it is 90:10, and 100 per cent central assistance for the Union Territories without the legislation. About 51.70 lakh beneficiaries are covered annually under the PMMVY and grants-in-aid amounting to Rs 2,048.59 crores and Rs 369.31 crores have been sanctioned to the states and the Union Territories during the year 2017-18 and 2018-19 respectively.

14)Ram Nath Kovind inaugurate the Centre of Excellence for Genetic Blood Disorders in which of the following city?

Ans:-Karimnagar, Telangana

Explanation:-The President of India, Shri Ram Nath Kovind, on his first visit to Karimnagar, inaugurated the Centre of Excellence for Genetic Blood Disorders at the Prathima Institute of Medical Sciences in Karimnagar, Telangana. This centre will work on Sickle Cell Anaemia, Thalassemia and Other Genetic Blood Disorders. It was set up by Prathima Institute of Medical Sciences.

15)Jharkhand CM announced Rs ______crore scheme for farmers.

Ans:-2,250

Explanation:-Jharkhand CM Raghubar Das announced a Rs.2,250 crore scheme to help 22.76 lakh small and marginal farmers of the state. Named Mukhyamantri Krishi Ashirwad Yojana, the scheme will provide farmers with a yearly financial assistance of Rs.5,000 per acre for Kharif crops. The scheme will start from 2019-20 financial year.

16)Who has conferred the Skoch Golden Jubilee Challenger Award at the 55th Skoch Summit in New Delhi?

Ans:-Suresh Prabhu

SSCE 8981426494/8296260082 Downloaded from - www.onlinessce.com Page 118

Explanation:-Commerce and Industry Minister Suresh Prabhu has conferred the Skoch Golden Jubilee Challenger Award at the 55th Skoch Summit in New Delhi. The Skoch Challenger Awards are the highest independently instituted civilian honours. The Commerce and Industry Minister was awarded for his contribution to reforms, particularly in the power sector reforms. A book titled 'India 2030' was also released on the occasion. For its inclusive banking and expansion in rural areas with the establishment of 35 easy payment units in Himalayan district of Leh in Jammu and Kashmir, J&K Bank was conferred with SKOCH Award in 'Banking and Finance (B&F) Silver Category'.

17)Sahitya Kala Parishad, Government of Delhi and this department organized the sixth edition of five-day long Yuva Natya Samaroh at New Delhi.

Ans:-Department of Art, Culture and Languages

Explanation:-Sahitya Kala Parishad, Department of Art, Culture and Languages, Government of Delhi, organized the sixth edition of five-day long Yuva Natya Samaroh at Kamani Auditorium in New Delhi. The event will conclude on 26th December. It is a theatre festival of five young directors with some of the best modern, mythological as well as Parsi style drama. The festival will start with the famous and well-discussed story by Mithileshwar titled Babuji and directed by Rajesh Singh while the Parsi play Safed Khoon directed by Samiruddin (Javed Sameer), will be presented on the last day of the festival.

18)Who was awarded Driver's Driver of the Year, recently?

Ans:-Lewis Hamilton

Explanation:- Lewis Hamilton was awarded Drivers' Driver of the Year after claiming the 5th World Drivers' Championship. After this he became the 3rd man in history to secure the 5th world title. This was a part of vote given by the F1 management for the first time among 17 of the 20 drivers participating in the event. He was followed by Max Verstappen in the second place, Fernando Alonso in the third place.

Daily Current Affairs 26th Dec,2018

1)This country launched first satellite for space-based broadband to rival Google.

Ans:-China

Explanation:-The satellite was launched from a Long March 11 carrier rocket from the Jiuquan Satellite Launch Centre in north-western China. The satellite is the first in the Hongyun project planned by China Aerospace Science and Industry Corp (CASIC). The Hongyun project, started in September 2016, aims to build a space-based communications network to provide broadband internet connectivity to users around the world, especially those in the underserved regions.

2)For which bank, P.V. Bharathi was Appointed as MD and CEO?

SSCE 8981426494/8296260082 Downloaded from - www.onlinessce.com Page 119

Ans:-Corporation Bank

Explanation:-P V Bharathi has been appointed as managing director and chief executive officer of the Corporation Bank. Bharathi is at present Executive Director, Canara Bank. She will replace Jai Kumar Garg. She will take over the charge on or after February 1, 2019. Birupaksha Mishra and Balakrishna Alse S have been appointed as executive director in the Corporation Bank and the Oriental Bank of Commerce, respectively.

3)Which two countries agreed on avoidance of power vacuum in Syria?

Ans:-US and Turkey

Explanation:-US president Donald Trump and Turkish President Tayyip Erdogan agreed to prevent a power vacuum in Syria after the pullout of American forces. According to an official statement by Turkish Presidency, the both of the leaders in a conversation yesterday, agreed to ensure coordination between countries' military and other officials to avoid a power vacuum which could result into abuse of the withdrawal and transition phase in Syria.

4)Kalu police station placed in the list of best performing police stations across the nation. It was located in ______.

Ans:-Rajasthan

Explanation:-Union Home Minister Rajnath Singh released the list of best performing police stations across the nation during the ongoing DGP conference. On the top of the rank- list stands Kalu police station of Rajasthan. Singh awarded trophies to the police officers of the top three police stations of India in 2018 at the conference. The top three police stations in India for 2018 are Kalu (Bikaner, Rajasthan), Campbell Bay ( Andaman & Nicobar Islands). Farakka (Murshidabad, West Bengal).

5)Which state launched a AAUY scheme, where the each household in the state will be able to avail medical treatment of up to 5 lakh rupees annually?

Ans:-Uttarkhand

Explanation:-Atal Ayushman Uttarakhand Yojana' has been launched in Uttarakhand. Under the scheme, each household in the state will be able to avail medical treatment of up to 5 lakh rupees annually. The scheme will benefit 23 lakh households and cover 1,350 critical diseases. Launching the Yojna in Dehradun, Chief Minister Trivendra Singh Rawat distributed scheme-related golden cards to beneficiaries. Mr. Rawat stated that all the households have been included under the scheme as the government wants that everyone should be able to afford medical treatment.

6)Where was the first meeting of the Follow-up Committee held?

Ans:-Iran

SSCE 8981426494/8296260082 Downloaded from - www.onlinessce.com Page 120

Explanation:-The first meeting of the Follow-up Committee for implementation of the Trilateral Chabahar Agreement between India, Afghanistan and Iran was held in the port city of Chabahar in Iran. India Ports Global Limited company opened its office and took over operations at Chabahar. They agreed on the routes for the trade and transit corridors between the three countries. It was agreed to finalize at the earliest Protocol to harmonize transit, roads, customs, and consular matters.

7)Which become the first state in the country to appoint out-of-state trade representatives.

Ans:-Maharashtra

Explanation:-Maharashtra has become the first state in the country to appoint out-of-state trade representatives. The Maharashtra State Agriculture Marketing Board (MSAMB) appointed six trade representatives to promote trade in semi-perishable commodities. They will also collect relevant market information. They have been posted in Punjab, Haryana, Delhi, Rajasthan, West Bengal, Assam, and Tamil Nadu.

8)Tsunami stuck coastal towns in ______on 22 December 2018.

Ans:-Indonesia

Explanation:-Tsunami stuck coastal towns on Indonesia's Sunda Strait on 22 December 2018. It is thought undersea landslides from the Anak Krakatau volcano caused them. The Sunda Strait, between the islands of Java and Sumatra, connects the Java Sea to the Indian Ocean. Indonesia is prone to tsunamis because it lies on the Ring of Fire, the line of frequent earthquakes and volcanic eruptions.

9)Mission Indradhanush was launched for a targeted approach to immunisation in India. It was related with ______.

Ans:-Child Vaccination

Explanation:-Mission Indradhanush was launched for a targeted approach to immunisation in India, in a bid to change the tardy annual growth rate of 1 per cent. Between 2014 and 2018, India's annual immunisation growth rate has risen to 4 per cent, with an unprecedented 16 per cent rise in the number of fully immunised children. The word Indradhanush was chosen to represent the seven vaccines that were subsequently included in the Universal Immunization Program against these seven diseases - tuberculosis, poliomyelitis, hepatitis B, diphtheria, pertussis, tetanus and measles.

10)Who was appointed as the next High Commissioner of India to Bangladesh?

Ans:-Riva Ganguly Das

SSCE 8981426494/8296260082 Downloaded from - www.onlinessce.com Page 121

Explanation:-Riva Ganguly Das was appointed as the next High Commissioner of India to Bangladesh. She is currently the Director General of Indian Council of Cultural Relations (ICCR).

11)PM Narendra Modi inaugurated 4.94-kilometer long bridge in Assam recently, It was named as;

Ans:-Bogibeel bridge

Explanation:-PM Narendra Modi inaugurated the Bogibeel bridge in Assam. The 4.94- kilometer long Bogibeel bridge on Brahmaputra river will connect the two northeast states of Assam and Arunachal Pradesh. This Bogibeel Bridge on Brahmaputra river marks a historic milestone towards achieving the goal of New India. It was in 2002, the then Prime Minister Atal Bihari Vajpayee commenced the construction of the double-decker bridge in Dibrugarh of Assam. The 4.9 km-long Bogibeel Bridge on the Brahmaputra river is India's only fully welded bridge for which European codes and welding standards were adhered to for the first time in the country

12)Turkmenistan launched its first messaging app on 24 December 2018. It was named as;

Ans:-BizBarde

Explanation:-Turkmenistan launched its first messaging app on 24 December 2018. The name of the messaging app is 'BizBarde'. The app will allow the exchange of messages, files, photos, and videos. The application is available via the Google Android platform as well as Apple's iOS and Microsoft Windows.

13)Bharat Ratna and former Prime Minister Atal Bihari Vajpayee's birthday is celebrated as Good Governance Day across the country on ______.

Ans:-25 December

Explanation:-Bharat Ratna and former Prime Minister Atal Bihari Vajpayee's birthday is celebrated as Good Governance Day across the country on 25 December. The Samadhi of Bharat Ratna and Former Prime Minister Atal Behari Vajpayee Sadaiv Atal was dedicated to the Nation. The Samadhi has been completed by CPWD at a cost of over ten crore rupees and the entire cost of construction has been borne by the Atal Smriti Nyas Society.

14)Researchers have developed the world's smallest ______board game by using dynamic DNA origami tiles.

Ans:-Tic-tac-toe

Explanation:-The researchers from California Institute of Technology, United States of America (USA) made world's smallest tic-tac-toe game board using dynamic DNA

SSCE 8981426494/8296260082 Downloaded from - www.onlinessce.com Page 122

microscopic organic structures that can be programmed to transform into predesigned patterns. Using this technique they fashioned a microscopic game of tic-tac-toe in which players place their X's and O's by adding special DNA tiles to the board. They had chosen to make the world's smallest version of the Italian polymath Leonardo Da Vinci's iconic painting Mona Lisa. DNA consists of a backbone and four types of molecules known as bases. These bases are adenine, guanine, cytosine, and thymine.

15)Which state is the venue of the 106th Indian Science Congress?

Ans:-Punjab

Explanation:-The 106th Indian Science Congress (ISC-2019) will be held at Lovely Professional University in Jalandhar, Punjab on January 3-7 with theme ''Future India: Science and Technology' . It will be inaugurated by Prime Minister Narendra Modi. The Indian Science Congress Association (ISCA) has been organising the event annually to bring together science fraternity from across the world to discuss innovations and researches. This year, six Nobel laureates from countries like Germany, Hungary, England and others will be participating in the science congress to make it the largest such gathering of its kind in the country. Eminent scientists from Indian Space Research Organisation (ISRO), Department of Science and Technology (DST), University Grants Commission (UGC) and All India Council for Technical Education (AICTE) are also slated to take part. The science congress will attract over 20,000 delegates including 3000 acclaimed scientists and researchers.

16)Who bagged the Indian-style prestigious wrestling championship 'Maharashtra Kesari 2018'?

Ans:-Bala Rafique Shaikh

Explanation:-Bala Rafique Shaikh clinched the 62nd Maharashtra Kesari 2018 title by defeating Abhijeet Katke of Pune in the wrestling duel held at Azad ground, Jalna, Maharashtra. He defeated his opponent 11-3 in the one-sided match. Shaikh was honoured with a silver mace and cash prize of Rs 1 lakh. Maharashtra Kesari is an Indian-style prestigious wrestling championship, established in 1961. 'Kesari' is a Sanskrit word for 'Lion'.

Daily Current Affairs 27th Dec,2018

1)The 7th National Photography Awards 2019 was organised by the ministry of ______.

Ans:-Ministry of Information and Broadcasting

Explanation:-The photo division of Ministry of Information & Broadcasting (I&B) is organizing the 7th National Photography Awards in February 2019. Entries for all the awards have been invited from across the country. The photographers will be awarded in mainly three categories - life time achievement award, awards for professional photographers

SSCE 8981426494/8296260082 Downloaded from - www.onlinessce.com Page 123

and the awards for the amateur photographers. The award for professional photographer has two categories- professional photographer of the year award with cash prize of Rs. 1 lakh and special mention awards with cash prize of Rs. 50 thousand. The Life Time Achievement award carries a cash prize of Rs 3 lakh. The entries can be sent to the photo division by 31st of December 2018. Those interested can visit the division's website for further details.

2)Name the Indian born who won the Junior Debate Championship at the World Scholar's Cup held at Yale University

Ans:-Aadi Sai Vijaykaran

Explanation:-Aadi Sai Vijaykaran created history by winning the Junior Debate Championship at the World Scholar's Cup held at Yale University, the USA. The theme for the cup was 'An Entangled World: diplomacy, human relationships, the science of memory, and literature, art'. It is the first time an individual from India has won the coveted title.

3)The first country in Southeast Asia to legalize medical marijuana is;

Ans:-Thailand

Explantion:-The junta-appointed parliament voted 166-0 to amend the Narcotic Act of 1979 to legalise the medicinal use of marijuana. Marijuana was used in Thailand to relieve pain and fatigue until its use was criminalised in the country in the 1930s. In Singapore, Malaysia and Indonesia, marijuana traffickers who are caught, face capital punishment.

4)Sulagitti Narasamma, who passed away recently, was the renowned social worker of which state?

Ans:-Karnataka

Explanation:-Sulagitti Narasamma (97), the renowned social worker and Padma Shri awardee, has passed away in Bengaluru on December 26, 2018. Born in 1920, she hailed from a community that was at the time referred to as 'untouchable'. She is best known for extending free of cost midwife services to women at a time when medical help was hard to come by in rural areas. It is believed that she learned the art from her grandmother, Marigemma, at a very young age. She had helped deliver more than 15,000 babies in Krishnapura, a remote village in Pavagada taluk in Karnataka. Her work received special recognition in Krishnapura & was conferred an honorary doctorate by the Tumkur University in 2014. She was also conferred with the Padma Shri by President Ram Nath Kovind on March 20, 2018.

5)This country formally Announces IWC Withdrawal to resume Commercial Whaling.

Ans:-Japan

SSCE 8981426494/8296260082 Downloaded from - www.onlinessce.com Page 124

Explanation:-In a landmark policy shift, Japan formally announced that it would withdraw from the International Whaling Commission (IWC) and resume commercial whaling in its territorial waters next year for the first time in more than 30 years. Chief Cabinet Secretary Yoshihide Suga announced Japan will restart commercial whaling in July 2019 - its first such excursion since 1988 - but limit its hunting activities to its own territorial waters and exclusive economic zone.

6)Who was appointed as the Executive Director of Allahabad Bank?

Ans:-K Ramachandran

Explanation:-K Ramachandran was appointed as the Executive Director of Allahabad Bank from the current post of General Manager. Ramachandaran will hold office up to the date of his superannuation, which is June 30, 2021.

7)Reserve Bank of India appointed whom as a chairman of the Economic Capital Framework (ECF) committee?

Ans:-Bimal Jalan

Explanation:- The Reserve Bank of India (RBI) appointed former governor Bimal Jalan as the chairman of the Economic Capital Framework (ECF) committee formed to address the issue of excess reserves with the central bank. Former RBI deputy governor Rakesh Mohan has been appointed the vice-chairman of the committee. Other members include Bharat Doshi, Sudhir Mankad, Subhash Chandra Garg, and N.S. Vishwanathan. The committee will decide whether RBI is holding provisions, reserves, and buffers in the surplus or deficit of the required levels. It will submit its report within 90 days of the first meeting.

8)Who organised a Swachch Bharat Grand Challenge in New Delhi, recently?

Ans:-DIPP

Explanation:-As part of the Swachhta Pakhwada held from, Department of Industrial Policy and Promotion (DIPP) organized a Swachch Bharat Grand Challenge. Four areas of sanitation, waste management, water and wastewater management and air management were selected for the grand challenge. The 1st Prize money for the Swachch Bharat Grand Challenge was Rs. 2 lakh. The awards were given away by Secretary DIPP, Ramesh Abhishek in New Delhi.

9)Name the American woman who became the youngest woman to win 50 World Cup ski races with victory in the slalom in the French Alps.

Ans:-Mikaela Shiffrin

Explanation:-American Mikaela Shiffrin became the youngest woman to win 50 World Cup ski races with victory in the slalom in the French Alps. She finished 0.29 sec ahead of

SSCE 8981426494/8296260082 Downloaded from - www.onlinessce.com Page 125

Slovakia's Petra Vlhova and 0.37sec ahead of Sweden Frida Hansdotter in third place to win in Courchevel victory.

10)Who among the following was inducted into ICC Cricket Hall of Fame?

Ans:-Ricky Ponting

Explanation:-Former Australia captain Ricky Ponting was formally inducted into the ICC Cricket Hall of Fame. Ponting received his commemorative cap from compatriot ICC Cricket Hall of Famer, Glenn McGrath at the MCG. Ponting was named in the ICC Cricket Hall of Fame along with former India captain and England woman wicketkeeper-batter Claire Taylor during the ICC Annual Conference in Dublin.

11)Which state government renamed Child Care Institutes as Jagannath Ashrams recently?

Ans:-Haryana

Explanation:-The Haryana government has recently decided to renamed Child Care Institutes as Jagannath Ashrams. This was announced by the Chief Minister Manohar Lal Khattar while addressing the people during a dance competition organised by women wing of All India Vaish Federation (AIVF) in Karnal on December 25, 2018.

12)Who has been conferred with the 'Tansen Samman' for 2018 by the Madhya Pradesh government?

Ans:-Manju Mehta

Explanation:-Noted sitar player Manju Mehta has been conferred with the 'Tansen Samman' for 2018 by the Madhya Pradesh government for her contribution in the field of music. Union minister Narendra Singh Tomar presented the award to Mehta, 74, at the inaugural ceremony of the Tansen Music Festival. The 'Raja Mansingh Tomar Honour' was given to the Sankat Mochan Pratishthan in Varanasi for 2017 and the Natrang Pratishthan of New Delhi for 2018. The award is given to institutions for nurturing good music.

13)The third Dwijing Festival begins in the state of;

Ans:-Assam

Exolanation:-In Assam, the third Dwijing Festival has begun on the bank of Aye river at Chirang district. Around 15 lakh tourists will participate in the festival. Assam PHE minister Rihon Daimary inaugurated the festival in presence of several dignitaries. Chief Minister Sarbanand Sonowal will attend the event. Adventure sports, food mart, cultural events, traditional games are being organized in the 12-day long event. Participants from Thailand, Bhutan and Bangladesh have reached the festival.

SSCE 8981426494/8296260082 Downloaded from - www.onlinessce.com Page 126

14)Vice Chancellor of the University of Kashmir Dr. Hamidi Kashmiri passed away recently. He is famous in which of the following language?

Ans:-Urdu

Explanation:-Dr. Hamidi Kashmiri, the tallest literary figure in the world of Urdu language, literature and literary criticism passed away. He was a prolific creative writer, an acclaimed critic and propounder of "Iktishaafi Tanqueed". He has also been the Vice Chancellor of the University of Kashmir for years.

15)Who has been appointed as India's Chief Boxing Coach?

Ans:-C A Kuttappa

Explanation:-Dronacharya awardee C A Kuttappa has taken over as India's chief boxing coach at the ongoing national camp. He took over from veteran coach SR Singh at the beginning of the camp. The 39-year-old Kuttappa is credited with shaping some of the country's most successful boxers such as Vijender Singh, M Suranjoy Singh, and Shiva Thapa, among others.

16)'Bharat Ratna Atal Bihar Vajpayee International Schools' for students in rural areas was launched by;

Ans:-Maharashtra

Explanation:-The Maharashtra government has launched Bharat Ratna Atal Bihari Vajpayee International Schools in Mumbai on the occasion of 94th birth anniversary of the late Prime Minister. The school is affiliated to the Maharashtra International Education Board (MIEB) with an aim to achieve higher education standards in rural areas.

17)Name the first female speaker of Fiji's Parliament, who died on 22nd December 2018.

Ans:-Dr. Jiko Luveni

Explanation:-On 22nd December 2018, Dr. Jiko Luveni, the first female speaker of Fiji's Parliament, died in office at the age of 72. Luveni first served for eight years as minister for women, social welfare and poverty alleviation in the interim government of Prime Minister Voreqe Bainimarama, who seized power in a 2006 coup.

18)Andhra Pradesh gets it new High Court in ______.

Ans:-Amaravati

Explanation:-The separate High Court for Andhra Pradesh will start functioning from January 1. Ministry of Law and Justice notified the constitution of the High Court in

SSCE 8981426494/8296260082 Downloaded from - www.onlinessce.com Page 127

Amaravati. Andhra Pradesh High Court will be the 25th High Court in the country. The Hyderabad High Court has been functioning as common high court till now after bifurcation of undivided Andhra Pradesh.

19)How many islands will be renamed in the Andaman and Nicobar to mark the 75th anniversary of Netaji's visit to the islands?

Ans:-3

Explanation:-The Centre has reportedly decided to rename Ross Island, Neil Island and Havelock Island in Andaman and Nicobar as Netaji Subhas Chandra Bose Island, Shaheed Dweep and Swaraj Dweep respectively. PM Narendra Modi is likely to announce it during his upcoming visit to Port Blair, on 30 December. The occasion is to mark the 75th anniversary of Netaji's visit to the islands.

Daily Current Affairs 28th Dec,2018

1)26th edition of National Children's Science Congress begins in ______.

Ans:-Bhubaneswar

Explanation:-Odisha Chief Minister Naveen Patnaik inaugurated 26th edition of National Children's Science Congress in Bhubaneswar in which as many as 800 students from ten Asian and seven Gulf countries are participating. This year's theme is 'Science, Technology and Innovations: For clean, green and healthy nation'. It is for the second time that the National Children's Science Congress is being organized in Odisha. Earlier, the mega event was held in 2015.

2)Which company emerged as the most profitable Public Sector Undertakings (PSUs) in 2017-18?

Ans:-Indian Oil Corporation

Explanation:-Indian Oil Corporation, Oil and Natural Gas Corporation (ONGC) and NTPC emerged as the top three most profitable Public Sector Undertakings (PSUs) in 2017-18. Coal India and Power Grid Corporation occupied the fourth and fifth spot in the list of top 10 profit making CPSEs (Central Public Sector Enterprises) in the 2017-18 fiscal. BSNL, Air India and MTNL incurred the highest losses for the second consecutive year.

3)Who was appointed the deputy managing director of Infrastructure Leasing & Financial Services Ltd?

Ans:-Bijay Kumar

Explanation:-Infrastructure Leasing & Financial Services Ltd appointed retired Indian Administrative Service official Bijay Kumar as its deputy managing director.

SSCE 8981426494/8296260082 Downloaded from - www.onlinessce.com Page 128

4)Which has been awarded Best Capital City in the Solid Waste Management in the Swachh Survekshan 2018 rankings?

Ans:-Hyderabad

Explanation:-The Greater Hyderabad Municipal Corporation (GHMC) has been awarded Best Capital City in the Solid Waste Management in the Swachh Survekshan 2018 rankings. Swachh Survekshan was launched by the Ministry of Housing and Urban Affairs (MoHUA) under Swachh Bharat Mission (Urban). The Swachh Survekashan survey ranks cities on various sanitation parameters. The GHMC was also at the 27th place among 4,041 cities. The GHMC said that as a part of the Swachh Survekshan 2019, maintaining the timing of toilets, i.e., from 4 a.m. to 10 p.m. and proper signage will be an important parameter. As a part of this, the GHMC has requested the Hyderabad Metro Rail to ensure that public toilets in all metro corridors are open from 4 a.m. to 10 p.m.

5)Reserve Bank of India (RBI) launched a 'Survey on Retail Payment Habits of Individuals (SRPHi)' for how many cities?

Ans:-6

Explanation:-The Reserve Bank of India (RBI) launched a 'Survey on Retail Payment Habits of Individuals (SRPHi)' in 6 cities including four metropolitan cities. The survey will cover a sample of 6,000 individuals from various socio-economic backgrounds across six cities of: Delhi, Mumbai, Kolkata, Chennai, Bengaluru and Guwahati. The fieldwork for the survey was carried out by Sigma Research & Consulting Pvt Ltd. The survey would capture payment habits of individuals and this will provide some idea about awareness and usage habits of digital payment products.

6)The 41st meeting of the Central Sanctioning and Monitoring Committee was held in ______.

Ans:-New Delhi

Explanation:-The 41st meeting of the Central Sanctioning and Monitoring Committee was held in New Delhi. In the meeting, the Ministry of Housing & Urban Affairs has approved the construction of another 3,10,597 more affordable houses for the benefit of urban poor under Pradhan Mantri Awas Yojana (Urban). A total of 864 projects with a project cost of Rs 14,662 crore with central assistance of Rs 4,658 crore has been approved in the meeting held under the Chairmanship of Durga Shankar Mishra, Secretary, Ministry of Housing and Urban Affairs. The cumulative number of houses sanctioned under PMAY(U) now is 68,54,126. Uttar Pradesh has been sanctioned 1,08,135 houses while the sanction for Karnataka is 1,05,502 affordable houses.

7)Which state cabinet announced the approval of an online system for mutation of property?

SSCE 8981426494/8296260082 Downloaded from - www.onlinessce.com Page 129

Ans:-West Bengal

Explanation:-The West Bengal cabinet chaired by Chief Minister Mamata Banerjee announced approval of an online system for mutation of property. Its objective is to ease the process of property mutation and check illegal practices.

8)Who released the book 'Prince, Patron and Patriarch Maharaja Jagatjit Singh of Kapurthala'?

Ans:-Amarinder Singh

Explanation:-Captain Amarinder Singh, Chief Minister of Punjab, released the book 'Prince, Patron and Patriarch Maharaja Jagatjit Singh of Kapurthala', co-authored by the Maharaja's grandson, Brigadier (Retd) Jagjit Singh, and Cynthia Frederick in Chandigarh. Punjab Governor VP Singh Badnore was also present on the occasion of the launch of the book. The book which is published by Roli Books is a candid tribute by a grandson to the erstwhile exceptional royal king of Kapurthala. Maharaja Jagatjit Singh is considered as one of the scripters of modernization of Kapurthala who brought Patiala and Kapurthala on the world map through modernization.

9)Russia planned to introduce the new type of strategic weapon. It was named as;

Ans:-Avangard system

Explanation:-Russia has conducted a final test of a nuclear-capable glider that flies at 20 times the speed of sound. The missile was launched from the Dombarovskiy missile base in the southern Ural Mountains and hit its target on a test site in Kamchatka, about 6,000km away. Avangard is designed using new composite materials to withstand temperatures of up to 2,000 degrees Celsius that come from a flight through the atmosphere at hypersonic speeds.

10)The centre Railways of this Division has developed the AI-powered robot named USTAAD to check for faults in trains.

Ans:-Nagpur Division

Explanation:-The mechanical branch of Central Railway's Nagpur division developed a new AI-powered robot named USTAAD (Undergear Surveillance through Artificial Intelligence Assisted Droid) which examines parts of the coach in real time with an HD camera and transmits them over WiFi. The new Artificial Intelligence (AI) powered robot will click photos and record videos of the train, under the gears and then transmits them over WiFi to engineers for maintenance and repair. Since USTAAD can watch and examine areas which cannot be seen and approached by humans usually, due to the narrow spaces between under- gear parts, it will reduce chances of human error while examining undergear equipment on Indian Railways trains. The special features of the robot include HD (high-definition) camera

SSCE 8981426494/8296260082 Downloaded from - www.onlinessce.com Page 130

that can be rotated in any direction as per commands are given by the engineer and can capture video as well as still photographs with zoom in or out facility.

11)An Indian-American, Rajesh Subramaniam was appointed as the President and Chief Executive Officer of

Ans:-FedEx

Explanation:-FedEx Express, a U.S. multinational courier delivery giant, has named Rajesh Subramaniam, an Indian-American, as the President and Chief Executive Officer of the firm. He will assume his new role on January 1, 2019. He replaces David L Cunningham. Mr. Subramaniam currently holds the executive vice- president, chief marketing and communications officer position of FedEx Corporation. He is an IIT-Bombay graduate from Thiruvananthapuram and has been with FedEx for more than 27 years. He has held various executive-level positions in the company. He began his career in Memphis and subsequently moved to Hong Kong where he oversaw marketing and customer service for the Asia Pacific region.

12)Union Ministry for Electronics and Information Technology has announced draft rules of ______to end the complete encryption system to trace 'unlawful' activities for better national security.

Ans:-Information Technology (Intermediary Guidelines) Rules 2018

Explanation:-The Union Ministry for Electronics and Information Technology (MeitY) announced draft rules of the Information Technology (Intermediary Guidelines) Rules 2018. Its objective is to end the complete encryption system to trace 'unlawful' activities for better national security. These would amend and replace the Information Technology Act, 2000 and would include the inter-ministerial consultations, followed by discussions with major social media platforms like Facebook, Google, Twitter, Yahoo and WhatsApp, and associations representing intermediaries. According to the amended rules under Section 79 of the IT Act, It will be mandatory for online platforms to deploy technology to enable access to content seen as "unlawful". Information about "unlawful acts" can be sought from the intermediaries by "court order". The parameter to judge unlawful acts would be Article 19(2) of the Constitution, which places restriction on freedom of speech and expression. The measures proposed include breaking end-to-end encryption so that the origin of messages can be traced. Online platforms will keep a record of "unlawful activity" for a period of "180 days", double the 90 days in the older version.

13)This ministry signs MoU to Develop A Mobile Audio Guide App For 5 Iconic Sites.

Ans:-Minister of State for Tourism

Explanation:-K J Alphons, Minister of State (I/C) for Tourism, handed over a Memorandum of Understanding (MoU) under the 'Adopt a Heritage' scheme to Resbird Technologies for development of a mobile Audio Guide App for five iconic sites. The five

SSCE 8981426494/8296260082 Downloaded from - www.onlinessce.com Page 131

iconic sites for which the mobile Audio Guide App is to be developed are Amer Fort (Rajasthan), Kaziranga (Assam), Colva Beach (Goa), Kumarakom (Kerala) and Mahabodhi Temple (Bihar).

14)Who has been conferred with the 'Champions of Change' award by Vice President M Venkaiah Naidu in Delhi?

Ans:-N. Biren Singh

Explanation:-Manipur Chief Minister N Biren Singh conferred the 'Champions of Change' award by Vice President M Venkaiah Naidu in Delhi. He was recognized for his leadership role in transforming the governance in the north-eastern state. At a programme to identify the progress made in 115 'aspirational' districts, Manipur was chosen for its all-round development on various parameters in all its 'aspirational' districts. Other such awardees are: 1. Union Minister of State for Food Processing Sadhvi Niranjan Jyoti, 2.Union Minister for Tribal Affairs Sudarshan and 3.Member of Parliament Nishikant Dubey. The award selection jury was led by former Chief Justice of India K G Balakrishna and former Supreme Court judge Sudha Mishra.

15)Divya Patidar Joshi was crowned as Mrs. India My Identity Beauty Pageant 2018. She belongs to the state of;

Ans:-Madhya Pradesh

Explanation:-The Mrs. India My Identity Beauty Pageant 2018 finale was held in Delhi and Divya Patidar Joshi was crowned with the prestigious honor. Hailing from Madhya Pradesh's Ratlam, Divya won the crown of Mrs India this year from among 24 promising contestants. She won all the rounds of Beauty with Brain contest along with Best Cat Walk title.

16)Which country imposed a monthly limit on the amount of Indian currency?

Ans:-Nepal

Explanation:-Nepal imposed a monthly limit on the amount of Indian currency its citizen can spend in India. A spokesperson of the Nepal Rastra Bank (NRB) stated that a Nepali citizen would be unable to spend more than INR 1 lakh per month while paying for goods and services in India. The decision was taken to address the country's current account deficit. The policy applicable to prepaid, credit and debit cards of Nepali banks came into effect.

17)SPJIMR hosted the 12th Annual Indian Subcontinent Decision Sciences Institute (ISDSI). The theme of this international conference is;

Ans:-Data-Driven Decision Making in the Digital Age

Explanation:-SPJIMR hosted the 12th Annual Indian Subcontinent Decision Sciences Institute (ISDSI) Conference at SPJIMR, Mumbai. The theme of this international

SSCE 8981426494/8296260082 Downloaded from - www.onlinessce.com Page 132

conference is "Data-Driven Decision Making in the Digital Age". The main objective of this conference is to provide a platform for academicians, researchers and practitioners to disseminate research, providing insights into the challenges, opportunities, emerging strategies and analytical tools in the domain of decision making.

Daily Current Affairs 28th Dec,2018

1) Who among was named as the Balkan Athlete of the Year for 2018?

Ans:-Luka Modric

Explanation:-Croatia captain Luka Modric was named Balkan Athlete of the Year, becoming only the second soccer player to scoop the prize after Bulgaria's former European Footballer of the Year Hristo Stoichkov in 1994. Ballon d'Or winner Modric won the Champions League and the FIFA Club World Cup with his Spanish club Real Madrid as well as being instrumental in helping Croatia reach the World Cup final in Russia. He was awarded the tournament's Golden Ball Award.

2)The Scientist from this Institute have developed world's Thinnest Material by making usage of Magnesium Diboride.

Ans:-IIT Gandhinagar

Explanation:-Scientists from Indian Institute of Technology, Gandhinagar have developed world's Thinnest Material by making usage of Magnesium Diboride. It has its utility in next- generation materials and UV absorbing films.

3)2019 International Camel festival will be held in ______.

Ans:-Rajasthan, India

Explanation:-International Camel festival will be held at Bikaner in Rajasthan from 12 Jan 2019. Activities such as fur cutting, camel decoration, designing among others would be organised at the twenty-sixth International camel festival. Competitions like Mr. Bikana and Miss Marwan, cultural performances by folk artists from different regions would also be held during the festival.

4)Which bank pipped Tata Group as India's largest business house worth Rs. 10.40 lakh crore?

Ans:-HDFC Bank

Explanation:-According to Bloomberg data, Deepak Parekh-led HDFC group overtook Tata Group become the country's largest conglomerate in terms of market capitalization. The total market capitalization (MACP) of the group's five listed companies is currently valued at Rs 10.40 lakh crore, which is Rs 1,185 crore more than that of Tata Group. HDFC Bank, a part

SSCE 8981426494/8296260082 Downloaded from - www.onlinessce.com Page 133

of the group, is only the third company in Indian corporate history to cross the Rs 5-lakh crore market capitalization threshold, after TCS (Rs 7.16 trillion) and RIL (Rs 7.09 trillion).

5)The government of India has decided to infuse Rs 2,159 crore in the bank of ______.

Ans:-United Bank of India

Explanation:-State-owned United Bank of India announced that the government has decided to infuse Rs 2,159 crore in the bank as part of Rs 28,615 crore capital infusion to be done in about half a dozen banks. The government has decided to pump Rs 28,615 crore into seven public sector banks (PSBs) through recapitalization bonds soon. Out of these seven PSBs, Bank of India is likely to get the highest amount of Rs 10,086 crore.

6)Name the person who become the first one to complete a solo trek across Antarctica.

Ans:-Colin Timothy O'Brady

Explanation:-A 33-year-old man Colin O'Brady from the United States has become the first person to complete a solo trek across Antarctica without any assistance. He finished the 1,500km journey across the frozen continent in 54 days, lugging his supplies on a sledge as he skied in bone-chilling temperatures from north to south.

7)Who has been appointed as the Chief Justice of new Andhra HC?

Ans:-Chagari Praveen Kumar

Exp;anation:-Chagari Praveen Kumar appointed as Chief Justice of Amaravati High Court, Andhra. He will become acting Chief Justice of AP High Court, with effect from Jan 1, 2019. Earlier, the Union Law Ministry issued an order signed by President Ram Nath Kovind, which said Andhra Pradesh will have its own High Court at its capital Amaravati and the Hyderabad High Court shall become High Court for Telangana.

8)Where was the 41st meeting of the Central Sanctioning and Monitoring Committee held?

Ans:-New Delhi

Explanation:-The 41st meeting of the Central Sanctioning and Monitoring Committee under the Ministry of Housing & Urban Affairs to sanction more affordable houses for the benefit of urban poor under Pradhan Mantri Awas Yojana (Urban) was held at New Delhi. A total of 3,10,597 more affordable houses for the benefit of urban poor under Pradhan Mantri Awas Yojana (Urban) has been approved by the Ministry of Housing & Urban Affairs. The approval consists of: Beneficiary Led Construction (BLC) vertical- 1,91,556 and Affordable Housing in Partnership (AHP)vertical- 1,18,941.

SSCE 8981426494/8296260082 Downloaded from - www.onlinessce.com Page 134

9)Recently, the National Human Rights Commission (NHRC) launched a toll-free number to file complaints through Common Service Centre (CSC). The toll-free number is;

Ans:-14433

Explanation:-The National Human Rights Commission (NHRC) launched a toll-free number and also a facility to file complaints through Common Service Centre (CSC). The toll-free number is '14433'. These steps had been taken by the NHRC to expand its outreach. Justice (retd) H L Dattu is the Chairperson of the National Human Rights Commission.

10)For which project, The Union Cabinet forms a three-member crew to be sent to space for at least seven days.

Ans:-Gaganyaan

Explanation:-The Union Cabinet has approved the Gaganyaan project under which a three- member crew will be sent to space for at least seven days, Union Minister Ravi Shankar Prasad stated. The project will cost INR 10,000 crore. The Gaganyaan project was announced by Prime Minister Narendra Modi in his Independence Day speech. He had stated that the mission will be undertaken by 2022. India has already inked agreements with Russia and France for assistance in the ambitious project.

11)The 6th International Rice Research Institute will be constructed in the campus of;

Ans:-NSRTC, Varanasi

Explanation:-The PM also dedicated the 6th International Rice Research Institute (IRRI), South Asia Regional Center (ISARC) to the nation. The Institute is built at the campus of National Seed Research and Training Center (NSRTC) in Varanasi.

12)Indian Railways announced ______fare concession for transgender senior citizens.

Ans:-40%

Explanation:-Indian Railways announced 40% fare concession for transgender senior citizens. The concession for a person of the third gender, aged 60 years or above, will be available from January 1, 2019. Till now, only men who are 60 years and above, and women aged 58 years and above got the concession. The rate of concession is 40 per cent for men and 50 per cent for women. Though a column for transgender (T) was introduced in the reservation form, the concession was not admissible to them. As of December of 2018, the Indian Railways offers concessions on ticket prices to 53 different categories ranging from 10 per cent to 100 per cent.

SSCE 8981426494/8296260082 Downloaded from - www.onlinessce.com Page 135

13) India signs two Bilateral MoUs between these two countries in the area of Biotechnology on 28th December 2018.

Ans:-Cuba and Korea

Explanation:-The Union Cabinet chaired by Prime Minister Narendra Modi has given approval for the two Bilateral MoUs between India and Cuba, and India and Korea in the area of Biotechnology on 28th Dec 2018. It also give an approval for Setting up of the National Commission for Homoeopathy (NCH) Bill, 2018. Setting up of the National Commission for Indian System of Medicine Bill, 2018. Submission of India's Second Biennial Update Report (BUR) to United Nations Framework Convention on Climate Change (UNFCCC).

14)Which country approved the issue 'Panda bond' in Chinese currency, recently?

Ans:-Pakistan

Explanation:-Pakistan's cabinet has approved the issuance of first-ever renminbi- denominated 'Panda Bonds'. It will issue the bonds to raise loans from China's capital markets. It expects to raise $500 million to $1 billion in different tranches. The interest rate may be over 5.5% but the final price will be determined at the time of launching the bond.

15)President Ram Nath Kovind approved Early Childhood Care and Education Centers (Registration and Regulation) Bill 2017 for which state?

Ans:-Himachal Pradesh

Explanation:-President Ram Nath Kovind approved two state bills of Tamil Nadu and Himachal Pradesh. They are The Himachal Pradesh Early Childhood Care and Education Centers (Registration and Regulation) Bill 2017. And The Tamil Nadu Vanniyakula Kshatriya Public Charitable Trusts and Endowments (Protection and Maintenance) Bill 2018.

16)Which is the least improved district in the Second Delta Ranking of the Aspirational Districts Programme?

Ans:-Pakur, Jharkhand

Explanation:-Niti Aayog CEO Amitabh Kant released the Second Delta Ranking of the Aspirational Districts Programme. Pakur, Jharkhand least improved district in the Second Delta Ranking of the Aspirational Districts Programme. The ranking shows the incremental progress achieved by 111 aspirational districts (out of 115) during June 1, 2018 to October 31, 2018, across six key development sectors. Three aspirational districts from West Bengal did not participate, while one district of Kerala could not take part because of floods. The 6 developmental sectors are: Health and Nutrition, Education, Agriculture and Water Resources, Financial Inclusion, Skill Development, and Basic Infrastructure.

SSCE 8981426494/8296260082 Downloaded from - www.onlinessce.com Page 136

17)World's busiest single-runway airport after Mumbai, recently sold for USD 3.7 bn is ______.

Ans:- Gatwick Airport, London

Explanation:-French infrastructure giant Vinci has agreed to acquire control of London's Gatwick Airport, the world's second-busiest single runway airport after Mumbai, for $3.7 billion. Vinci, which manages 45 airports in 12 countries including Chile and Cambodia, will own a 50.01% stake in Gatwick Airport. The purchase comes ahead of the UK's departure from the European Union on March 29.

18)Ambika Prasad Panda was appointed as Chairman and Managing Director of ______.

Ans:-SECL

Explanation:-Ambika Prasad Panda was appointed as Chairman and Managing Director of South Eastern Coalfields Limited (SECL). She is at present Director of finance in SECL. She has been appointed the CMD of SECL for a period of five years. SECL is the subsidiary of Coal India Limited and its headquarters is in Bilaspur, Chhattisgarh.

Daily Current Affairs 29th Dec,2018

1)For which country, India announces INR 4,500 crore assistance recently?

Ans:-Bhutan

Explanation:-Prime Minister Narendra Modi announced Rs. 4,500 crore financial assistance to Bhutan for its 12th five-year plan after holding wide-ranging talks with his Bhutanese counterpart Lotay Tshering. The new five-year plan of Bhutan began in 2018 and will continue till 2022.

2)The Scientist from this Institute have developed world's Thinnest Material by making usage of Magnesium Diboride.

Ans:-IIT Gandhinagar

Explanation:-Scientists from Indian Institute of Technology, Gandhinagar have developed world's Thinnest Material by making usage of Magnesium Diboride. It has its utility in next- generation materials and UV absorbing films.

3)Who was named as the Balkan Athlete of the Year for 2018?

Ans:- Luka Modric

Explanation:-Croatia captain Luka Modric was named Balkan Athlete of the Year, becoming only the second soccer player to scoop the prize after Bulgaria's former European SSCE 8981426494/8296260082 Downloaded from - www.onlinessce.com Page 137

Footballer of the Year Hristo Stoichkov in 1994. Ballon d'Or winner Modric won the Champions League and the FIFA Club World Cup with his Spanish club Real Madrid as well as being instrumental in helping Croatia reach the World Cup final in Russia. He was awarded the tournament's Golden Ball Award.

4)Which bank pipped Tata Group as India's largest business house worth Rs. 10.40 lakh crore?

Ans:-HDFC Bank

Explanation:-According to Bloomberg data, Deepak Parekh-led HDFC group overtook Tata Group become the country's largest conglomerate in terms of market capitalization. The total market capitalization (MACP) of the group's five listed companies is currently valued at Rs 10.40 lakh crore, which is Rs 1,185 crore more than that of Tata Group. HDFC Bank, a part of the group, is only the third company in Indian corporate history to cross the Rs 5-lakh crore market capitalization threshold, after TCS (Rs 7.16 trillion) and RIL (Rs 7.09 trillion).

5)The government of India has decided to infuse Rs 2,159 crore in the bank of ______.

Ans:-United Bank of India

Explanation:-State-owned United Bank of India announced that the government has decided to infuse Rs 2,159 crore in the bank as part of Rs 28,615 crore capital infusion to be done in about half a dozen banks. The government has decided to pump Rs 28,615 crore into seven public sector banks (PSBs) through recapitalization bonds soon. Out of these seven PSBs, Bank of India is likely to get the highest amount of Rs 10,086 crore.

6)Name the person who become the first one to complete a solo trek across Antarctica.

Ans:-Colin Timothy O'Brady

Explanation:-A 33-year-old man Colin O'Brady from the United States has become the first person to complete a solo trek across Antarctica without any assistance. He finished the 1,500km journey across the frozen continent in 54 days, lugging his supplies on a sledge as he skied in bone-chilling temperatures from north to south.

7)Who has been appointed as the Chief Justice of new Andhra HC?

Ans:-Chagari Praveen Kumar

Explanation:-Chagari Praveen Kumar appointed as Chief Justice of Amaravati High Court, Andhra. He will become acting Chief Justice of AP High Court, with effect from Jan 1, 2019. Earlier, the Union Law Ministry issued an order signed by President Ram Nath Kovind, which said Andhra Pradesh will have its own High Court at its capital Amaravati and the Hyderabad High Court shall become High Court for Telangana.

SSCE 8981426494/8296260082 Downloaded from - www.onlinessce.com Page 138

8)Where was the 41st meeting of the Central Sanctioning and Monitoring Committee held?

Ans:-New Delhi

Explanation:-The 41st meeting of the Central Sanctioning and Monitoring Committee under the Ministry of Housing & Urban Affairs to sanction more affordable houses for the benefit of urban poor under Pradhan Mantri Awas Yojana (Urban) was held at New Delhi. A total of 3,10,597 more affordable houses for the benefit of urban poor under Pradhan Mantri Awas Yojana (Urban) has been approved by the Ministry of Housing & Urban Affairs. The approval consists of: Beneficiary Led Construction (BLC) vertical- 1,91,556 and Affordable Housing in Partnership (AHP)vertical- 1,18,941.

9)Recently, the National Human Rights Commission (NHRC) launched a toll-free number to file complaints through Common Service Centre (CSC). The toll-free number is;

Ans:-14433

Explanation:- The National Human Rights Commission (NHRC) launched a toll-free number and also a facility to file complaints through Common Service Centre (CSC). The toll-free number is '14433'. These steps had been taken by the NHRC to expand its outreach. Justice (retd) H L Dattu is the Chairperson of the National Human Rights Commission.

10)For which project, The Union Cabinet forms a three-member crew to be sent to space for at least seven days.

Ans:-Gaganyaan

Explanation:-The Union Cabinet has approved the Gaganyaan project under which a three- member crew will be sent to space for at least seven days, Union Minister Ravi Shankar Prasad stated. The project will cost INR 10,000 crore. The Gaganyaan project was announced by Prime Minister Narendra Modi in his Independence Day speech. He had stated that the mission will be undertaken by 2022. India has already inked agreements with Russia and France for assistance in the ambitious project.

11)The 6th International Rice Research Institute will be constructed in the campus of;

Ans:-NSRTC, Varanasi

Explanation:-The PM also dedicated the 6th International Rice Research Institute (IRRI), South Asia Regional Center (ISARC) to the nation. The Institute is built at the campus of National Seed Research and Training Center (NSRTC) in Varanasi.

12)Indian Railways announced ______fare concession for transgender senior citizens.

SSCE 8981426494/8296260082 Downloaded from - www.onlinessce.com Page 139

Ans:-40%

Explanation:-Indian Railways announced 40% fare concession for transgender senior citizens. The concession for a person of the third gender, aged 60 years or above, will be available from January 1, 2019. Till now, only men who are 60 years and above, and women aged 58 years and above got the concession. The rate of concession is 40 per cent for men and 50 per cent for women. Though a column for transgender (T) was introduced in the reservation form, the concession was not admissible to them. As of December of 2018, the Indian Railways offers concessions on ticket prices to 53 different categories ranging from 10 per cent to 100 per cent.

13)Which country approved the issue 'Panda bond' in Chinese currency, recently?

Ans:-Pakistan

Explanation:-Pakistan's cabinet has approved the issuance of first-ever renminbi- denominated 'Panda Bonds'. It will issue the bonds to raise loans from China's capital markets. It expects to raise $500 million to $1 billion in different tranches. The interest rate may be over 5.5% but the final price will be determined at the time of launching the bond.

14)As per the RBI data, which bank topped in the List Of Banks Penalising Staff For Fraud in 2015-2017?

Ans:-ICICI

Explanation:-According to data furnished by the Reserve Bank of India, between 2015 and 2017, private lender ICICI Bank penalized the highest number of employees for defrauding bank assets in the last three year out of 60 scheduled commercial banks operating in India. It is followed by State Bank of India and Punjab National Bank. In all, banks took action against a staggering 13,949 officials, of which 16%, or 2,236, belonged to ICICI.

15)Which is the least improved district in the Second Delta Ranking of the Aspirational Districts Programme?

Ans:-Pakur, Jharkhand

Explanation:-Niti Aayog CEO Amitabh Kant released the Second Delta Ranking of the Aspirational Districts Programme. Pakur, Jharkhand least improved district in the Second Delta Ranking of the Aspirational Districts Programme. The ranking shows the incremental progress achieved by 111 aspirational districts (out of 115) during June 1, 2018 to October 31, 2018, across six key development sectors. Three aspirational districts from West Bengal did not participate, while one district of Kerala could not take part because of floods. The 6 developmental sectors are: Health and Nutrition, Education, Agriculture and Water Resources, Financial Inclusion, Skill Development, and Basic Infrastructure.

SSCE 8981426494/8296260082 Downloaded from - www.onlinessce.com Page 140

16)India signs two Bilateral MoUs between these two countries in the area of Biotechnology on 28th December 2018.

Ans:-Cuba and Korea

Explanation:-The Union Cabinet chaired by Prime Minister Narendra Modi has given approval for the two Bilateral MoUs between India and Cuba, and India and Korea in the area of Biotechnology on 28th Dec 2018. It also give an approval for Setting up of the National Commission for Homoeopathy (NCH) Bill, 2018. Setting up of the National Commission for Indian System of Medicine Bill, 2018. Submission of India's Second Biennial Update Report (BUR) to United Nations Framework Convention on Climate Change (UNFCCC).

17)Ambika Prasad Panda was appointed as Chairman and Managing Director of ______.

Ans:-SECL

Explanation:-Ambika Prasad Panda was appointed as Chairman and Managing Director of South Eastern Coalfields Limited (SECL). She is at present Director of finance in SECL. She has been appointed the CMD of SECL for a period of five years. SECL is the subsidiary of Coal India Limited and its headquarters is in Bilaspur, Chhattisgarh.

18)President Ram Nath Kovind approved Early Childhood Care and Education Centers (Registration and Regulation) Bill 2017 for which of the state?

Ans:-Himachal Pradesh

Explanation:-President Ram Nath Kovind approved two state bills of Tamil Nadu and Himachal Pradesh. They are The Himachal Pradesh Early Childhood Care and Education Centers (Registration and Regulation) Bill 2017. And The Tamil Nadu Vanniyakula Kshatriya Public Charitable Trusts and Endowments (Protection and Maintenance) Bill 2018.

19)World's busiest single-runway airport after Mumbai, recently sold for USD 3.7 bn is ______.

Ans:-Gatwick Airport, London

Explanation:-French infrastructure giant Vinci has agreed to acquire control of London's Gatwick Airport, the world's second-busiest single runway airport after Mumbai, for $3.7 billion. Vinci, which manages 45 airports in 12 countries including Chile and Cambodia, will own a 50.01% stake in Gatwick Airport. The purchase comes ahead of the UK's departure from the European Union on March 29.

20)2019 International Camel festival will be held in ______

SSCE 8981426494/8296260082 Downloaded from - www.onlinessce.com Page 141

Ans:-Rajasthan, India

Explanation:-International Camel festival will be held at Bikaner in Rajasthan from 12 Jan 2019. Activities such as fur cutting, camel decoration, designing among others would be organised at the twenty-sixth International camel festival. Competitions like Mr. Bikana and Miss Marwan, cultural performances by folk artists from different regions would also be held during the festival.

Daily Current Affairs 30th Dec,2018

1)Which of the following has recently resumed its diplomatic service in Syria after six years of suspension?

Ans:-United Arab Emirates

Explanation:-The United Arab Emirates (UAE) has recently resumed its diplomatic service in Syria after six years of suspension, signaling a thaw in the relations between the two countries, as also with the Arab world. The diplomatic thaw comes after the U.S. announced plans to pull its troops from Syria. This move will also prevent the dangers of regional interference in Syrian affairs. Hence, this marks the first official acknowledgment by regional adversaries of Syrian President Bashar Assad that he has won the almost 8-year civil war to dislodge him. Earlier, the UAE broke ties with Syria in February 2012 when "Arab Spring"-inspired protests began against Assad's rule and metastasized into a sectarian conflict that killed hundreds of thousands, made millions of others refugees and laid waste to vast areas of the country. Syria was also expelled from the 22-member Arab League in 2011. Arab countries sanctioned Damascus and condemned its use of military force against civilians.

2)Which ministry has recently brought a booklet on cyber safety for school children?

Ans:-Ministry of Home Affairs

Explanation:-The Union Ministry of Home Affairs (MHA) has brought out a booklet on cyber safety for school children to inform them about different aspects of cyber safety. The booklet, titled 'A Handbook for Students on cyber Safety', also deals with the problems of cyber bullying, cyber grooming and email fraud. It advises teenagers against accepting friend requests from unknown people on social media. The booklet explains various kinds of cyber crimes like identity theft, job fraud, email spoofing and how children can overcome them. The information provided in the handbook is intended to create awareness among citizens especially students about various cyber threats that can impact them and ways to safeguard themselves against cyber crimes. The booklet also provides details about various safeguards that can be adopted by children to protect themselves from cyber grooming.

3)How many Central Public Sector Enterprises are approved by the Union Cabinet recently?

SSCE 8981426494/8296260082 Downloaded from - www.onlinessce.com Page 142

Ans:- 7

Explanation:-The Union Cabinet approved the listing of 7 Central Public Sector Enterprises (CPSEs) on the stock exchanges via Initial Public Offering (IPO) or Follow-on Public Offer (FPO). The companies that will go for IPOs include Telecommunication Consultants (India) Ltd., RailTel Corporation India Ltd., National Seed Corporation India Ltd., Tehri Hydro Development Corporation Limited, Water & Power Consultancy Services (India) Limited and FCI Aravali Gypsum and Minerals (India) Limited. Kudremukh Iron Ore Company Limited will go for an FPO.

4)Which short film won the best short documentary award at Asia South East Short Film Festival?

Ans:-Finding Beauty in Garbage

Explanation:-The short film Finding Beauty in Garbage won the best short documentary award at Asia South East Short Film Festival recently held at Phnom Penh in Cambodia. The short documentary is based on issues surrounding mountains of garbage in Dibrugarh town of Assam. The film has also been selected for a screening at the Short and Sweet Film Festival to be held in Utah, U.S.

5)Which district topped in the Second Delta Ranking of the Aspirational Districts Programme, released by NITI Aayog?

Ans:-Virudhunagar, Tamil Nadu

Explanation:-Niti Aayog CEO Amitabh Kant released the Second Delta Ranking of the Aspirational Districts Programme. The ranking shows the incremental progress achieved by 111 aspirational districts (out of 115) during June 1, 2018 to October 31, 2018, across six key development sectors. Three aspirational districts from West Bengal did not participate, while one district of Kerala could not take part because of floods. The 6 developmental sectors are: Health and Nutrition, Education, Agriculture and Water Resources, Financial Inclusion, Skill Development, and Basic Infrastructure. The data has been received from household surveys conducted by NITI Aayog's knowledge partners, namely, TATA Trusts and Bill & Melinda Gates Foundation (IDInsight).

6)Who was elected as the new President of Madagascar, recently?

Ans:-Andry Rajoelina

Explanation:-Andry Rajoelina, a former leader of Madagascar has won the country's presidential vote and in the process defeated Marc Ravalomanana, who was another former President of the country. Rajoelina won nearly 56% of the votes and had got 39% of the vote during the first round of the election, whereas, Mr. Ravalomanana had got 35%.The former president of the country, Henry Rajaonarimampianina was eliminated in the first round of the election as he obtained just 9% of the votes. Both Rajoelina and Ravalomanana are wealthy

SSCE 8981426494/8296260082 Downloaded from - www.onlinessce.com Page 143

men and were barred from the 2013 presidential race and had promised to boost the economy of the country. Although, the elections had been beset by allegations of fraud.

7)Where was the 12th Global Healthcare Summit 2018 held?

Ans:-Mumbai

Explanation:-The 12th Global Healthcare Summit (GHS) was inaugurated by President Ram Nath Kovind in Mumbai, Maharashtra on 29th December. The summit is being organized by the American Association of Physicians of Indian Origin, more commonly known as AAPI. Mr. Kovind acknowledged the professionalism and excellent reputation of doctors of Indian origin in the US.

8)Who laid the foundation stone of 11 projects for Yamuna rejuvenation?

Ans:- Nitin Gadkari

Explanation:-A total of 11 projects have been taken up under Namami Gange Programme by National Mission for Clean Ganga to conserve the River Yamuna in Delhi. Most of the projects are for the sewerage infrastructure and are at various stages of implementation. These projects under Yamuna Action Plan (YAP) III are located in three drainage zones namely Kondli, Rithala and Okhla in Delhi.

9)Union Cabinet approved Framework Agreement between India and which country for Cooperation in the Exploration and Uses of Outer Space for Peaceful Purposes?

Ans:-Sao Tome & Principe

Explanation:-The Union Cabinet, chaired by the Prime Minister Shri Narendra Modi approved Framework Agreement between India and Sao Tome & Principe. The MOU was signed on Cooperation in the Exploration and Uses of Outer Space for Peaceful Purposes. The Framework Agreement was signed in New Delhi, on 7th September 2018.

10)Who won the Gold Medal by the Systems Society of India (SSI) during the 42nd National Systems Conference?

Ans:-K.Sivan

Explanation:-ISRO chairman K.Sivan was presented the National Systems Gold Medal by the Systems Society of India (SSI) during the 42nd National Systems Conference held in Thiruvananthapuram. The medal acknowledges his outstanding contributions to system science and engineering. B.N. Suresh, the Chancellor, Indian Institute of Space Science and Technology (IIST), presented the medal at a function held at the VSSC.

11)Which bank will issue the Seventh Phase of Electoral Bonds to Sale from 1 Jan?

SSCE 8981426494/8296260082 Downloaded from - www.onlinessce.com Page 144

Ans:-State Bank of India

Explanation:-The State Bank of India (SBI), in the VII Phase of sale of Electoral Bonds, has been authorised to issue and encash Electoral Bonds through its 29 Authorised Branches w.e.f. January 01 - 10, 2019. Electoral bonds are alternatives to cash donations made to political parties that were introduced in 2018 as part of the government's efforts to bring transparency in political funding. These are short-term bonds that are valid for 15 calendar days, where the holders do not earn any interest (coupon rate). In the last six phases till November 2018, electoral bonds aggregated up to Rs 1,056.73 crore from purchases made by citizens or entities.

12)Name the person who has been named as the most admired man by Americans for the 11th year in a row according to a gallup poll?

Ans:-Barack Obama

Explanation:-Former President of United States of America (USA), Barack Obama has been named as the most admired man by Americans for the 11th year in a row according to a gallup poll. Present President of USA, Donald trumps ranked 2nd for the fourth consecutive year. Former First Lady of USA Michelle Obama was also voted as the most admired women by American. Hillary Clinton missed the top of the list for the list time in 17 years.

13)The unique Chapparam (Chariot) festival 2018 has started in which of the following state?

Ans:-Tamil Nadu

Explanation:-The unique Chapparam (Chariot) festival was celebrated with traditional rituals and festivities in the famous Madurai Meenakshi temple, in Tamil Nadu. The fest symbolizes the importance of preserving all kinds of food and fodder. The festival is also an auspicious occasion when the God and Goddess measure out food for all living beings. In it, both Goddess Meenakshi and her spouse Lord Sundareswarar circumambulate the four main streets around the temple. Cooked rice, which is distributed among devotees, are taken home and preserved because it signifies prosperity for the whole year. Another specialty of the festival is that the chariot of Goddess Meenakshi is pulled by lady devotees alone.

14)The Union Cabinet approved the submission of India's second Biennial Update Report (BUR) to ______.

Ans:-UNFCCC

Expanation:-The Union Cabinet approved the submission of India's second Biennial Update Report (BUR) to the United Nations Framework Convention on Climate Change (UNFCCC). BUR has been prepared based on a range of studies conducted at the national level. The scope of BUR is to provide an update to India's first BUR to UNFCCC. The Five major components of Biennial Update Report (BUR) are

SSCE 8981426494/8296260082 Downloaded from - www.onlinessce.com Page 145

15) The researchers from this country built a Virtual Reality (VR) 3D model of cancer, recently.

Ans:-UK

Explanation:- The scientists at Cambridge University have built a Virtual Reality (VR) 3D model of cancer that will help increase the understanding of cancer as well as in the search for new treatment. Researchers from the Cancer Research UK (CRUK) Researchers started with a one millimetre cubed piece of breast cancer tissue biopsy, containing around 100,000 cells. They cut wafer thin slices, scanned and then stained them with markers to show their molecular make-up and DNA characteristics. The tumour was then rebuilt using virtual reality (VR), which allows multiple users from anywhere, in the world to examine the tumour. Although the human tissue sample was about the size of a pinhead, using the VR headsets, it could be magnified to appear several metres across. To explore the tumour in more detail, the VR system allows one to 'fly through' the cells.

Daily Current Affairs 31st Dec,2018 1) 11th Bengaluru International Film Festival will start from February 7, 2019. Its theme is: Ans:- Nature's Fury Explanation: The 8-day 11th Bengaluru International Film Festival (BIFFES) will start in February 7, 2019. It will conclude in February 14, 2019. The theme of the Festival would be 'Nature's Fury.' It will be organized by: the Karnataka Chalanachitra Academy and Department of Information and Public Relations of Government of Karnataka. The festival is always held on first Thursday of February every year. 2) The book titled "Early Indians: The Story of Our Ancestors and Where We Came From" was written by whom? Ans:-Tony Joseph Explanation: The book titled "Early Indians: The Story of Our Ancestors and Where We Came From" written by Tony Joseph was released. The book talks about the migration of the First Indians also known as 'Aryans' who arrived here 65,000 years ago. 3) Which country successfully tested Russia's S-400 missile air defence system worth USD 3 billion Ans:- China Explanation: China successfully tested Russia's S-400 missile air defence system. It received the last shipments of the weaponry from Russia in July under a USD 3 billion contract signed in 2015. China was the first foreign buyer to seal a government-to- government deal with Russia in 2014 to procure the S-400 missile. It has a supersonic speed of 3 kilometre per second. 4) The cabinet of which country approved an issuance of "Panda Bonds" in Chinese Currency recently?

SSCE 8981426494/8296260082 Downloaded from - www.onlinessce.com Page 146

Ans:- Pakistan Explanation: Pakistan Cabinet approved the issuance of Pakistan's one-of-its-kind Panda Bonds after a meeting presided by Prime Minister Imran Khan. The approval for issuance of bonds in the capital markets of China was given effect to the delay issuance of US dollar dominated Euro bonds worth 3 billion dollars. Finance Minister, Asad Umar told the interest rate may range above 5.5% but the final price would be determined only at the time of launching the bond. 5) Who of the following was appointed as Chief Information Commissioner (CIC)? Ans: Sudhir Bhargava Explanation: The government has appointed Sudhir Bhargava as Chief Information Commissioner (CIC), along with four information commissioners. Mr. Bhargava was Information Commissioner in the CIC. An alumnus of St Michael's High School in Patna and St Stephen's College in Delhi, Mr. Sinha is a 1981-batch Indian Foreign Service officer who was the High Commissioner of India in the United Kingdom. 6) Which country successfully sent six Yunhai-2 atmospheric environment research satellites and a test communication satellite of Hongyan constellation into orbit? Ans:- China. Explanation: China successfully sent six Yunhai-2 atmospheric environment research satellites and a test communication satellite of Hongyan constellation into orbit. They were launched by a Long March-2D rocket from the Jiuquan Satellite Launch Center in northwest China. It was the 297th mission of the Long March rocket series. The six Yunhai-2 satellites will study atmospheric environment, monitor space environment, prevent and reduce disasters and conduct scientific experiments. The test satellite, the first for the constellation Hongyan, meaning "swan goose" in Chinese, will be used to verify the function of mobile communications in low earth orbit (LEO). 7) An International Seminar to commemorate the 200th Birth Anniversary of Sikh philosopher, reformer and freedom fighter , Sri Satguru Ram Singhji inaugurated in: Ans:- New Delhi Explanation: To commemorate the 200th Birth Anniversary of Sikh philosopher, reformer and freedom fighter , Sri Satguru Ram Singhji, an International Seminar was inaugurated by the Minister of State for Culture (I/c), Dr Mahesh Sharma in New Delhi. The seminar was organised by the Union Ministry of Culture in collaboration with Panjab University, Chandigarh and Kuka Martyrs Memorial Trust. 8) BeiDou Navigation Satellite System (BDS) of this country started to provide a global services. Ans: China Explanation: China's BeiDou Navigation Satellite System (BDS), touted as a rival to the widely-used American GPS, has started providing global services. The positioning accuracy of the system has reached 10 metres globally and five metres in the Asia-Pacific region.

SSCE 8981426494/8296260082 Downloaded from - www.onlinessce.com Page 147

9) Shri Ram Nath Kovind, inaugurated the centenary celebrations of the world's oldest yoga institute in ______that has marked 100 years of its establishment. Ans:- Mumbai Explanation: The President of India, Shri Ram Nath Kovind, inaugurated the centenary celebrations of the world's oldest yoga institute located in Mumbai that has marked 100 years of its establishment. 10) Who has adopted the Russian Resolution on International Information Security? Ans:- UNGA Explanation: The United Nations General Assembly (UNGA) has adopted two Russian Resolutions on International Information Security (IIS) and has been backed by India regarding the same. This has been done for achieving democratic, inclusive and transparent UN negotiations process on security in the use of information and communication technologies. The two resolutions are: "Developments in the field of information and telecommunications in the context of international security" and "Countering the use of information and communications technologies for criminal purposes" and has thus, a new chapter has been opened by UN on IIS. These documents are the world's first code of conduct in the digital paradigm and hence creates the foundation for peaceful interaction and to prevent war, confrontation and other aggressive actions. 11) Which of the following is the first police force in India to digitise all malkhanas across every police station under the e-malkhana project? Ans: Delhi Explanation: Delhi Police became the first police force in country to digitise all malkhanas across every police station under the e-malkhana project. Accordingly, around 3,11,600 case properties have been encrypted in digital form. Also, the e-malkhana project of the remaining 10 districts at Chinamaya Mission was also inaugurated by the Delhi Police Commissioner Amulya Patnaik. 12) For which state, the Central Government has given an extension of 6 months to complete the ongoing exercise for updating the National Register of Citizens (NRC)? Ans: Assam Explanation: The Center has given an extension of six months to complete the ongoing exercise for updating the National Register of Citizens (NRC) in Assam by 30th June, 2019 as the enumeration of citizens in the NRC cannot be completed within the earlier specified date of 31st December, 2018. The Registrar General of India, Mr. Sailesh issued a notification that stated that the Central Government considers the extension necessary in lieu of public interest to complete the update of National Register of Citizens. The first notification regarding the same was issued by the Government on 6th December, 2013 and had set a deadline of 3 years to complete the entire process but since then five extensions have been given but the exercise had not been completed. The draft NRC was published on 30th July, 2018 and excluded 40 lac people from the list which stirred a huge controversy. It included 2.9 crore people out of the total 3,29 crore applications.

SSCE 8981426494/8296260082 Downloaded from - www.onlinessce.com Page 148

13) Who has been named as the International Cricket Council (ICC) Women's Cricketer of the Year? Ans: Smriti Mandhana Explanation: India's stylish opener Smriti Mandhana has been named as the International Cricket Council (ICC) Women's Cricketer of the Year. The 22-year-old, who has also been named as the ICC Women's ODI Player of the Year, achieved the feat by giving some exceptional performances throughout the year. She scored 669 runs at an average of 66.90 in 12 ODIs and 622 runs at a strike-rate of 130.67 in 25 T20Is from January 1 to December 31, 2018. She is currently ranked fourth in the ICC Women's Players Rankings for ODI Batters and 10th in the ICC Women'Players Rankings for T20I Batters. 14) Name the small finance bank that has received the Reserve Bank approval to operate as scheduled bank making it the fifth scheduled bank that operates from Kerala? Ans: ESAF Small Finance Bank Explanation: The Thrissur, Kerala-based ESAF Small Finance Bank has received the Reserve Bank approval to operate as scheduled bank making it the fifth scheduled bank that operates from Kerala after Federal Bank, South Indian Bank, Catholic Syrian Bank, and Dhanlaxmi Bank. The ESAF Small Finance Bank Ltd has received the RBI's approval for inclusion in the second schedule of the RBI Act, 1934 paving its way to operate as scheduled bank. In 2016, RBI gave approval to ESAF MFI along with 9 other applicants to start banking operations as a small finance bank and within two years of its commencement of operations, it has received the scheduled bank tag. On the operation front, the ESAF Small Finance Bank has a network of 432 outlets across 13 states with 2.6 million customers and a loan book of Rs 4,660 crore and deposits if over Rs 3,300 crore. Its capital- to-risk-weighted- asset (CRAR) ratio stands at 27.39 per cent. 15) Who won Bangladesh's election with majority? Ans: Sheikh Hasina Explanation: Prime Minister Sheikh Hasina's alliance won Bangladesh's election with a thumping majority, the country's Election Commission announced. It gave her a third straight term following a vote that the opposition rejected as rigged. The win consolidated Hasina's decade-long rule over Bangladesh. The alliance dominated by her Awami League, seen as close to regional power India, won 287 of the 298 seats for which results have been declared, the commission said. There are 300 constituencies in the country. The main opposition Bangladesh Nationalist Party (BNP), which boycotted the last poll in 2014, won just six seats. 16) Which institute entered Guinness Book of World Records 'for organising the largest men's Health Awareness programme on prostate cancer on a single day? Ans: MNJ Institute of Oncology and Regional Cancer Centre Explanation: Telangana's MNJ Institute of Oncology and Regional Cancer Centre entered Guinness Book of World Records 'for organising the largest men's Health Awareness

SSCE 8981426494/8296260082 Downloaded from - www.onlinessce.com Page 149

programme on prostate cancer on a single day.' The programme was organised on October 26, 2018 and 487 participants took part in it. This is the first Guinness World Records achievement for the state government in the field of medicine. 17) For which team, Archie Schiller was named as the co-captain? Ans: Australia. Explanation: Cricket Australia have added seven-year-old Archie Schiller in the 15-member squad for the third Test of the series against India, at the Melbourne Cricket Ground (MCG) on Boxing Day, December 26. He was co-captain alongside Tim Paine. Archie Schiller, who has endured 13 heart operations and spent half of his life in hospital after being diagnosed with a life-threatening congenital disease, had the dream of captaining the Australian cricket team. Make a Wish Australia and Cricket Australia, worked together to make Archie's cricket dreams come true. 18) Oscar-and Grammy award winner Norman Gimbel was passed away. He was a famous ______Ans: Lyricist. Explanation: Oscar-and Grammy-winning lyricist Norman Gimbel died at 91. He was best known for writing the lyrics of "Killing Me Softly With His Song" and "The Girl From Ipanema". He was also inducted into the Songwriters Hall of Fame in 1984. 19) For which field, the Government of India will award "Subhash Chandra Bose Aapda Prabandhan Puraskar award"? Ans: Disaster management Explanation: The Government of India invites nominations for the "Subhash Chandra Bose Aapda Prabandhan Puraskar" for excellence in the field of disaster management. The winners will be declared on 23rd January 2019, on the occasion of the birth anniversary of Netaji Subhash Chandra Bose. The last date for applying is 7th January 2019 20) Which country announced 2019 as the 'Year of Active Investment and Social Development' in the backdrop of proposed investments from India to boost its economy? Ans: Uzbekistan Explanation: The President of Uzbekistan, Shavkat Mirziyoyev announced 2019 as the 'Year of Active Investment and Social Development' in the backdrop of proposed investments from India to boost economy of the Central Asian country, in his annual address to the Parliament. The President stated that concrete measures will be taken on creating an open economy, radically improving the business environment and investment climate and reducing the state presence in the economy.

Like our face book page: www.facebook.com/onlinessce

SSCE 8981426494/8296260082 Downloaded from - www.onlinessce.com Page 150

Sen’s School of Competitive

Examinations

One of the Best Institutes in Hooghly District

th Our New Batch will start from 11 February 2019 Contact No:-

8981426494/8296260082

Visit us: www.onlinessce.com

SSCE 8981426494/8296260082 Downloaded from - www.onlinessce.com Page 151